Vous êtes sur la page 1sur 170

5

OBIECTIVE DE REFERIN
I EXEMPLE DE ACTIVITI DE NVARE

1. Cunoaterea i nelegerea conceptelor, a terminologiei i a procedurilor de calcul

Obiective de referin
La sfritul clasei a VII-a elevul va fi
capabil

1.1.s utilizeze noiuni de logic i teoria
mulimilor




1.2.s utilizeze metode i principii
adecvate n rezolvarea problemelor








1.3.s rezolve ecuaii prin diferite metode
i s utilizeze ecuaii i inecuaii n
rezolvarea problemelor





1.4.s utilizeze noiuni de divizibilitate



1.5.s efectueze calcule cu numere reale



Exemple de activiti de nvare
Pe parcursul clasei a VII-a se
recomand urmtoarele activiti :

-rezolvarea unor probleme folosind
proprieti comune oricrei partiii
-exerciii de determinare a numrului de
elemente al unor mulimi folosind
principiul includerii i excluderii

-rezolvarea unor probleme de maxim i
minim
-rezolvri de probleme folosind principiul
invariantului
-probleme care se rezolv folosind
principiul lui Diriclet
-probleme de numrare
-probleme folosind principiul paritii
-probleme de ordonare

-rezolvarea ecuaiilor care conin modul i
parte ntreaga
-folosirea inegalitilor mediilor n
rezolvarea unor inegaliti sau a unor
inecuaii
-rezolvri de ecuaii diofantice (n
mulimea numerelor ntregi)

-rezolvarea unor probleme folosind
noiuni de divizibilitate, congruente
modulo n

-exerciii de calcul a unor sume folosind
diferite metode
-compararea, ordonarea i reprezentarea
pe axa a unor numere reale
-calcularea valorii unor expresii



6

1.6.s rezolve problemele puse la alte
discipline folosind metode matematice

1.7.s utilizeze metode, axiome, leme,
teoreme i relaii geometrice n
demonstrarea problemelor









1.8.s recunoasc i s utilizeze n
demonstraii proprietile unor figuri
geometrice sau a unor linii importante n
triunghi
-rezolvarea problemelor de geometrie
plana care i au originea n fizica

-rezolvarea problemelor de concurenta i
coliniaritate folosind teoreme
reprezentative : Menelaos, Van Aubel,
Ceva
-calculul lungimilor liniilor importante n
triunghi
-calculul ariilor unor figuri geometrice i
aplicarea metodei areolare n
demonstrarea unor probleme
-rezolvarea problemelor folosind relaii
trigonometrice

-demonstrarea unor inegaliti geometrice
-construcii de figuri geometrice
-determinarea locurilor geometrice
remarcabile i rezolvarea problemelor de
loc geometric



2.Dezvoltarea capacitii de a emite judeci de valoare pentru rezolvarea proble-
melor inventiv i euristic-creative


Obiective de referin
La sfritul clasei a VII-a elevul va fi
capabil :
2.1.s analizeze, s elaboreze un plan de
rezolvare i s rezolve probleme dificile









2.2.s formuleze probleme echivalente cu
o problema data modificnd pri ale
ipotezei i/ sau ale concluziei

Exemple de activiti de nvare
Pe parcursul clasei a VII-a se recomanda
urmtoarele activiti :
-analizarea ipotezei, concluziei i
nelegerea problemei
-alegerea metodei de abordare a rezolvrii
problemei
-rezolvarea problemei i verificarea
rezultatului
-analiza rezolvarii
-verificarea redundantei unor ipoteze i
ncercri de eliminare ale unor cerine
(pri ) din ipoteza

-formulri de probleme prin extragerea
unor cazuri particulare sau prin
generalizare

7

2.3.s identifice tehnici de lucru pentru
clase de probleme


-identificarea unor tehnici de lucru
valabile pentru clase de probleme
-analizarea eficienei metodei


3.Dezvoltarea capacitii de a face conexiuni cognitive n cadrul disciplinei i a ariei
curriculare


Obiective de referin
La sfritul clasei a VII-a elevul va fi
capabil :

3.1.s utilizeze raionamente inductive n
rezolvarea problemelor din domeniile
studiate
3.2.s-i formeze o gndire creativ i
divergent



Exemple de activiti de nvare
Pe parcursul clasei a VII-a se recomand
urmtoarele activiti

-folosirea intuiiei i perspicacitii n
alegerea modului de abordare a unei
probleme
-combinarea elementelor cunoscute i
crearea altora noi
-rezolvarea unor probleme teoretice
complexe prin stabilirea unor relaii ntre
cunotine


4.Dezvoltarea capacitii de a comunica utiliznd limbajul matematic

Obiective de referin
La sfritul clasei a VII-a elevul va fi
capabil :

4.1.s foloseasc terminologia specifica
matematicii
4.2.s discute avantajele i dezavantajele
utilizrii unei anumite tehnici de abordare
a unei probleme


Exemple de activiti de nvare
Pe parcursul clasei a VII-a se
recomanda urmtoarele activiti :

-redactarea matematica a unui text
folosind scrierea specifica
-citirea unui text scris matematic i
interpretarea lui
-discutarea argumentelor folosirii unei
anume metode de rezolvare
-descrierea etapelor de rezolvare


8
5. Dezvoltarea interesului i a motivaiei pentru studiul i aplicarea matematicii n
contexte variate

Obiective de referin
La sfritul clasei a VII-a elevul va fi
capabil :

5.1.s sesizeze importanta noiunilor de
geometrie n rezolvarea unor probleme
concrete
5.2.s manifeste ingeniozitate pentru
gsirea de soluii noi
5.3.s manifeste interes pentru folosirea
tehnologiilor informaiei n studiul
matematicii
Exemple de activiti de nvare
Pe parcursul clasei a VII-a se
recomanda urmtoarele activiti

-transpunerea unor probleme din limbaj
uzual n limbajul geometriei , rezolvarea
lor i interpretarea rezultatelor
-abordarea unor probleme la moda n
concursurile colare
-utilizarea unor soft-uri pentru nvarea
matematicii; explorarea internetului

9
CONINUTURI


ALGEBR

1.Mulimi
1.1.Partiii
1.2.Principiul includerii i excluderii
2.Congruente. Aplicaii la rezolvarea problemelor de divizibilitate
3.Principiul paritii
4.Principiul invariantului
5.Metoda reducerii la absurd
6.Probleme de numrare. Principiul lui Diriclet
7.Ecuatii n Z. Ecuaii diofantice
8.Modulul unui numr real. Ecuaii cu module
9.Partea ntreag a unui numr real. Ecuaii cu parte ntreag
10.Inegaliti
11.Probleme de ordonare
12.Rezolvarea ecuaiilor i inecuaiilor cu ajutorul inegalitii mediilor
13.Probleme de maxim i minim
14.Puncte laticiale


GEOMETRIE

1.Relatii metrice n triunghi
1.1.Teoreme ale bisectoarelor
1.2.Teorema lui Pitagora generalizata
1.3.Teorema lui Stewart
1.4.Teorema lui Van Aubel n triunghiul dreptunghic
1.5.Lungimile bisectoarelor interioare
1.6.Lungimile nlimilor
1.7. Teorema medianei
1.8. Teorema lui Lebniz
1.9. Teoreme de concurenta i coliniaritate
1.10. Teorema sinusurilor
1.11. Teorema cosinusului
2.Inegalitati geometrice

10
3.Locuri geometrice
4.Patrulatere inscriptibile i circumscriptibile
5.Constructii geometrice
6.Figuri echivalente
7.Metoda areolara
8.Triunghiuri speciale
8.1.Triunghiul ortic
8.2 Triunghiul tangenial

13
ALGEBR

1. Mulimi

1.1. Partiiile unei mulimi

Definiia 1.1.1. Se numete partiie a unei mulimi A o mulime de submulimi
nevide ale lui A, disjuncte dou cte dou, a cror reuniune este mulimea A.


Probleme rezolvate

R1.1.1. Pentru mulimea A={1,2,3} exist partiiile:
1) {1}, {2}, {3}; 2) {1}, {2,3}; 3) {2}, {1,3}; 4) {3}, {1,2}.
R1.1.2. Fie A={a,b,c,d}. Avem partiiile:
1) {a}, {b}, {c}, {d}; 2) {a}, {b,c,d}; 3) {b}, {a,c,d}; 4) {c}, {a,b,d}, 5) {d},
{a,b,c}; 6) {a,b}, {c}, {d}; 7) {a,c}, {b}, {d}; 8) {a,d}, {b}, {c}, 9) {b,c}, {a}, {d};
10) {c,d}, {a}, {b}; 11) {b,d}, {a}, {c}, 12) {a,b}, {c,d}; 13) {a,c}, {b,d}; 14) {a,d},
{b,c}.
Mulimea submulimilor unei mulimi nu reprezint o partiie, deoarece nu
toate submulimile sunt disjuncte.
Mulimea submulimilor unei mulimi A se numete mulimea prilor mulimii
A i se noteaz P(A).
Dac o mulime A are n elemente (nN) atunci P(A) are 2
n
elemente.
R1.1.3. S se determine numrul submulimilor
A={a,b,c,d}{1,2,...,102}
cu proprietatea c a+b=c+d=102.
Soluie. Presupunem c a<b<c<d. Atunci perechile cu suma 102 sunt: (1,101),
(2,100), (3,99),..., (49,53), (50,52). Perechii (1,101) i corespund 49 de perechi:
(2,100), (3,99),..., (50,52). Perechii (2,100) i corespund 48 de perechi: (3,99),
(4,98),..., (50,52). Numrnd analog obinem pentru perechea (49,53) perechea (50,52).
Numrul cerut este:
1225 25 49
2
50 49
1 2 ... 47 48 49 = =

= + + + + + .
R1.1.4. Fie mulimea A={1,2,3,...,998,999}.
Care este cel mai mic numr de submulimi n care poate fi partiionat
mulimea A astfel nct dac ntr-o submulime se afl 2
x
cu xN
*
, n acea submulime
s nu se mai afle (2
x
)
y
cu y natural diferit de zero i unu.
Soluie. Puterile naturale 2
x
cu x0 mai mici ca 999 sunt 2
1
, 2
2
, 2
3
, 2
4
, 2
5
, 2
6
,
2
7
, 2
8
, 2
9
. Dac fiecare din aceste elemente sunt n submulimi disjuncte atunci 2
x
nu
este n aceeai submulime cu (2
x
)
y
, dar nu este satisfcut condiia de minim (de
exemplu 2
2
, 2
3
, 2
5
, 2
7
pot fi n aceeai submulime orice putere cu exponent natural
diferit de zero i unu a acestor numere nu are ca rezultat pe unul din ele).
14
Vom construi un model care s realizeze minimul cerut, avnd n vedere c
submulimea ce-l conine pe 2 s nu mai conin i alt putere a sa; n submulimea n
care se afl 2
2
s nu se mai afle alt putere a lui 2
2
, n submulimea n care se afl 2
3
s
nu se mai afle o alt putere a lui 2
3
, etc. Atunci A
1
={1,2,3}, fiindc 4=2
2
A
1
.
A
2
={4,5,6,7,8,9,10,11,12,13,14,15}. A
2
poate conine elementele 4 i 8
deoarece 8=2
3
(2
2
)
y
dar nu poate s conin pe 16 deoarece 16=(2
2
)
2
.
A
3
={16,17,18,...,254,255}. A
3
nu-l poate conine pe 256=(2
4
)
2
, dac l conine
pe 16=2
4
. i n sfrit A
4
={256,257,...,998,999}.
Deci numrul minim de submulimi care satisfac ipoteza este 4, i am dat un
model de realizare. Evident soluia nu este unic.
R1.1.5. Partiionai mulimea A={1,2,3,4,...,49} n apte submulimi disjuncte,
astfel nct suma elementelor fiecrei submulimi s fie aceeai.
Soluie. Mulimea A are 49 de elemente. Din fiecare element scdem
25
2
1 49
=
+
pentru a obine o mulime ce are suma elementelor 0, adic
} 24 , 23 , 22 ,..., 2 , 1 , 0 , 1 ,..., 22 , 23 , 24 {
*
1
= A . Aceast mulime se poate partiiona.
Din
*
1
A extragem apte submulimi disjuncte de cte trei elemente, cu suma
elementelor zero. Fie aceste submulimi B
1
, B
2
, B
3
, B
4
, B
5
, B
6
, B
7
, cu B
1
={24,-7,-17},
B
2
={-24,7,17}, B
3
={20,-5,-15}, B
4
={-20,5,15}, B
5
={16,-3,-13}, B
5
={-16,3,13}, B
7
={-
1,0,1}.
Elementele rmase sunt apte cvadruple de numere ntregi, egale n valoare
absolut dou cte dou, dar de semne contrare:
(-2,-4,2,4), (-6,-8,6,8), (-9,-10,9,10), (-11,-12,11,12),
(-14,-18,14,18), (-19,-21,19,21), (-22,-23,22,23).
Atam acestea submulimilor B
1
, B
2
, B
3
, B
4
, B
5
, B
6
, B
7
i obinem:
A
1
={24,-7,-17,-2,-4,2,4}, A
2
={-24,7,17,-6,-8,6,8},
A
3
={20,-5,-15,-9,-10,9,10}, A
4
={-20,5,15,-11,-12,11,12}
A
5
={16,-3,-13,-14,-18,14,18}, A
6
={-16,3,13,-19,-21,19,21}
A
7
={-1,0,1-22,-23,22,23}
Adunnd 25 la fiecare element al mulimilor A
1
, A
2
, A
3
, A
4
, A
5
, A
6
, A
7

obinem:
C
1
={49,18,8,23,21,27,29}, C
2
={1,32,42,19,17,31,33},
C
3
={45,20,10,16,15,34,35}, C
4
={5,30,40,14,13,36,37},
C
5
={9,28,38,6,4,44,46}, C
6
={24,25,26,3,2,47,48}.
Mulimile C
1
, C
2
, C
3
, C
4
, C
5
, C
6
, C
7
sunt disjuncte i fiecare are suma
elementelor 175.
R1.1.6. S se cerceteze dac exist numere naturale n astfel nct mulimea
A={n,n+1,n+2,n+3,n+4,n+5} s poat fi mprit n dou submulimi disjuncte cu
proprietatea c produsul tuturor elementelor uneia dintre acestea s fie egal cu produsul
tuturor elementelor celeilalte submulimi.
Soluie. Pentru n=0 problema nu are soluie.
Problema nu are soluie dac cel puin un element al mulimii A se divide cu
un numr prim mai mare sau egal cu 7. Fiindc mulimea are cele ase elemente
15
numere consecutive, dac unul din acestea se divide cu un numr prim mai mare sau
egal cu 7 nu va mai exista n mulimea A un alt element divizibil cu un numr prim mai
mare sau egal cu 7. Cum A conine ase numere naturale consecutive, cel puin unul
este divizibil cu 5. Dac exist numai un element divizibil cu 5 problema nu are
soluie. Trebuie s existe dou elemente divizibile cu 5. Acestea pot fi numai n+5 i n.
Deci n este multiplu de 5. Elementele n i n+5 trebuie s fac parte din submulimi
diferite. S observm c produsul a dou dintre elementele mulimii A este mai mare
dect fiecare dintre celelalte elemente ale mulimii. Este suficient s artm c produsul
cel mai mic este mai mare dect cel mai mare dintre elementele rmase, adic
n(n+1)>n+5 n
2
-5>0. Aceast diferen este pozitiv pentru orice n5, nN.
Rezult c produsele care ar putea ndeplini cerina problemei trebuie cu
necesitate s conin cte trei i numai cte trei factori. Avem situaiile:
a) (n+1)(n+2)(n+5) i n(n+3)(n+4)
b) (n+1)(n+3)(n+5) i n(n+2)(n+4)
c) (n+1)(n+4)(n+5) i n(n+2)(n+3)
d) (n+2)(n+4)(n+5) i n(n+1)(n+3)
e) (n+2)(n+3)(n+5) i n(n+1)(n+4)
f) (n+3)(n+4)(n+5) i n(n+1)(n+2).
n situaia a) avem:
(n+1)(n+2)(n+5)=n(n+3)(n+4) n
3
+7n
2
+12n=n
3
+8n
2
+17n+10
n
2
+5n+10=0,
relaie imposibil pentru n5, nN.
Situaiile b), c), d), e), f) nu sunt posibile pentru c fiecare dintre factorii
primului produs sunt mai mari dect factorul corespunztor din cel de-al doilea produs.
Deci produsul din membrul stng este mai mare dect cel din membrul drept.
Deci nu exist nici un numr natural care s satisfac enunul problemei.


1.2. Principiul includerii i excluderii

Fie A o mulime finit cu n elemente.
Notm cu CardA cardinalul (numrul de elemente) mulimii A.
Teorema 1.2.1 (principiul includerii i excluderii). Fie A, B, C mulimi
finite i CardA, CardB, CardC numrul elementelor acestor mulimi.
Cardinalul mulimilor AB, ABC este dat de relaiile:
Card(AB)=CardA+CardB-Card(AB) (1)
Card(ABC)=CardA+CardB+CardC-Card(AB)-
-Card(AC)-Card(BC)+Card(ABC). (2)
Relaiile (1) i (2) sunt cazuri particulare (pentru n=2 i n=3) ale formulei lui
Boole-Sylvester, care va fi studiat mai trziu.
S artm relaia (1): Numrul elementelor din reuniunea mulimilor A i B
este egal cu suma numerelor elementelor din A i B, din care se scade numrul
elementelor comune mulimilor A i B, care au fost numrate de dou ori.
16
AB este reuniunea mulimilor disjuncte A i B\(AB), iar B este reuniunea
mulimilor disjuncte B\(AB) i AB.
Din Card(AB)=CardA+Card(B\(AB)) i
CardB=Card(B\(AB))+Card(AB) obinem:
Card(AB)=CardA+CardB-Card(AB), adic tocmai relaia (1).



Pentru relaia (2) folosii diagrama i observai c adunnd cardinalele
CardA+CardB+CardC i comparnd cu Card(ABC), trebuie sczute cardinalele
Card(AB), Card(BC), Card(AC), dar atunci se pierde i Card(ABC) i de
aceea trebuie adunat Card(ABC).
Vom prezenta n continuare cteva probleme care folosesc n rezolvare acest
principiu.
R1.2.1. Elevii unei clase joac fotbal sau baschet: 19 joac fotbal, 24 joac
baschet i 16 practic ambele jocuri. Ci elevi sunt n clas?
Soluie. Aplicm principiul includerii i excluderii. Fie F mulimea elevilor ce
joac fotbal, B mulimea elevilor ce joac baschet. Atunci CardF=19, CardB=24,
Card(FB)=16,
Card(FB)=CardF+CardB-Card(AB)=19+24-16=43-16=27.
Deci numrul elevilor din clas este 27.
R1.2.2. Din cei 1000 de elevi ai unei coli 506 particip la olimpiada de limba
romn, 460 particip la olimpiada de fizic i 442 particip la olimpiada de
matematic. Dintre acetia 160 particip la olimpiadele de limba romn i fizic, 162
particip la olimpiadele de fizic i matematic i 131 particip la olimpiadele de limba
romn i matematic. Ci dintre elevii colii particip la toate cele trei olimpiade.
Soluie. Aplicm principiul includerii i excluderii (relaia (2)).
Fie R mulimea elevilor ce particip la olimpiada de limba romn, F
mulimea elevilor ce particip la olimpiada de fizic, iar M mulimea elevilor ce
particip la olimpiada de matematic. Atunci CardR=506, CardF=460, CardM=442,
Card(RF)=160, Card(FM)=162, Card(RM)=131, Card(RFM)=1000.
Folosim relaia (2) i obinem:
Card(RFM)=CardR+CardF+CardM-Card(RF)-
-Card(RM)-Card(FM)+Card(RFM),
deci 1000=506+460+442-160-131-162+Card(RFM), de unde Card(RFM)=45.
Deci 45 de elevi din coal particip la cte trei olimpiade.

A B
A B
B A
17
Principiul includerii i excluderii permite rezolvarea simpl a unor probleme
de divizibilitate.
R1.2.3. Aflai numrul numerelor naturale mai mici sau egale cu 500 care sunt
divizibile cu 2, 3 sau 5.
Soluie. Fie A mulimea numerelor naturale nenule mai mici sau egale cu 500
divizibile cu 2, B mulimea numerelor naturale mai mici sau egale cu 500 divizibile
cu 3, iar C mulimea numerelor naturale mai mici sau egale cu 500 divizibile cu 5.
Numerele divizibile cu 2, mai mici sau egale cu 500 sunt: 2, 4, 6, 8,..., 500.
Vom folosi partea ntreag fiindc la mpriri ne intereseaz numai cturile nu
i resturile. Atunci
250
2
500
CardA =

= , 166
3
500
CardB =

= , 100
5
500
CardC =

= ,
82
6
500
B) Card(A =

= , 33
15
500
C) Card(B =

= ,
50
10
500
C) Card(A =

= , 16
30
500
C) B Card(A =

=
Din Card(ABC)=CardA+CardB+CardC-Card(AB)-
-Card(AC)-Card(BC)+Card(ABC) obinem Card(ABC)=367.
Acum putem afla i numrul numerelor naturale nenule mai mici sau egale cu
500 care nu sunt divizibile cu 2, nici cu 3, nici cu 5. Acestea sunt n numr de 500-
267=233.


18
2. Congruene n Z. Aplicaii la rezolvarea problemelor de divizibilitate

Definiia 2.1. Dou numere ntregi a i b se numesc congruente modulo n, n
fiind un numr ntreg, dac n divide pe a-b, adic n|(a-b).
Numrul n se numete modulul relaiei de congruen. n loc de a scrie n|(a-b),
se scrie ab(mod n) i citim: a este congruent cu b modulo n.
Proprieti ale relaiei de congruen n Z
2.1.1. Relaia de congruen este reflexiv, adic aa(mod n), oricare ar fi
numrul ntreg a, cnd modulul n este dat. (Orice numr ntreg (diferit de zero) este
congruent cu el nsui modulo n).
Avem n|0, dar a-a=0, deci n|(a-a), adic aa(mod n).
2.1.2. Relaia de congruen este simetric, adic dac ab(mod n), atunci
ba(mod n).
Faptul c ab(mod n) nseamn c n|(a-b).
Folosind proprietile relaiei de divizibilitate obinem n(-1)(a-b), deci n|(b-a),
adic ba(mod n).
2.1.3. Relaia de congruen este tranzitiv, adic dac ab(mod n) i bc(mod
n), atunci ac(mod n).
Faptul c ab(mod n) nseamn c n|(a-b), iar bc(mod n) nseamn c m|(b-c).
Folosind proprietile relaiei de divizibilitate obinem n|[(a-b)+(b-c)], deci
n|(a-c), de unde ac(mod n).
Relaia de congruen n raport cu un modul dat n fiind reflexiv, simetric i
tranzitiv este o relaie de echivalen.
2.1.4. i) Dou relaii de congruen n raport cu un acelai modul se adun
membru cu membru, adic: dac ab(mod n) i cd(mod n) atunci:
a+cb+d(mod n).
ii) Dou relaii de congruen n raport cu acelai modul se scad membru cu
membru, adic dac ab(mod n) i cd(mod n) atunci
a-cb-d(mod n).
iii) ntr-o relaie de congruen se poate trece un numr ntreg dintr-un membru
n altul al relaiei de congruen cu semnul schimbat, adic dac ab+c(mod n), atunci
a-cb(mod n).

i) Deci ab(mod n) nseamn c n|(a-b), iar cd(mod n) nseamn
n|(c-d). Folosind proprietile relaiei de divizibilitate obinem n|[(a-b)+(c-d)] sau
n|[(a+c)-(b+d)], de unde a+cb+d(mod n).
ii) Din ab(mod n) i cd(mod n) obinem n|(a-b) i n|(c-d), de unde n|[(a-b)-
(c-d)], sau n|[(a-c)-(b-d)], deci a-cb-d(mod n).
iii) Din ab+c(mod n) obinem n|[a-(b+c)], deci n|[(a-c)-b], adic
a-cb(mod n).
2.1.5. i) Dou relaii de congruen n raport cu acelai modul n se pot nmuli
membru cu membru, adic dac ab(mod n) i cd(mod n), atunci acbd(mod n).
ii) Ambii membri ai unei relaii de congruen se pot nmuli cu orice numr
ntreg diferit de zero, adic:
Dac ab(mod n) atunci acbc(mod n), oricare ar fi numrul ntreg c nenul.
19
iii) Ambii membri ai unei relaii de congruen se pot nmuli cu orice numr
ntreg nenul, nmulind n acelai timp i modulul, adic:
Dac ab(mod n), atunci ac=b(mod nc), oricare ar fi numrul ntreg c nenul.
Demonstraie. i) Din ab(mod n) obinem n|(a-b), iar din cd(mod n) obinem
c n|(c-d). Folosind proprietile relaiei de divizibilitate avem: n|[c(a-b)+b(c-d)], de
unde n|(ac-bd) adic acbd(mod n).
ii) Din ab(mod n) obinem n|(a-b), i folosind proprietile relaiei de
divizibilitate, pentru orice numr ntreg c avem: n|(a-b)c, deci n|(ac-bc), adic
acbc(mod n).
iii) Din ab(mod n) rezult c n|(a-b). Folosind proprietile relaiei de
divizibilitate obinem: nc|(a-b)c, pentru orice numr ntreg c.
Deci nc|(ac-bc), adic acbc(mod nc).
2.1.6. Orice relaie de congruen n raport cu un modul dat este n acelai
timp o relaie de congruen n raport cu orice modul care este un divizor al modulului
iniial, adic dac ab(mod n) i k|n, atunci ab(mod k).
Demonstraie. Din ab(mod n) obinem n|(a-b). Din k|n i n|(a-b) obinem k|(a-
b), deci ab(mod k).
2.1.7. Dac un membru al relaiei de congruen ntre dou numere ntregi se
divide cu modulul, atunci i cellalt membru al relaiei de congruen se divide cu
modulul, adic
dac ab(mod n) i n|a, atunci n|b sau
dac ab(mod n) i n|b, atunci n|a.
Demonstraie. Din ab(mod n) obinem n|(a-b).
Dac n|a atunci n|[(-1)(a-b)+a], deci n|b.
Dac n|b atunci n|[(a-b)+b], deci n|a.
2.1.8. Dou numere ntregi oarecare sunt ntotdeauna congruente n raport cu
modulul 1 i (-1), adic ab(mod 1) i ab(mod (-1)) oricare ar fi numerele ntregi a i
b nenule.
Demonstraie. Fiindc a,bZ i atunci 1|(a-b), deci ab(mod 1), iar din -1|(a-b)
obinem ab(mod (-1)).
Relaia de egalitate este un caz particular al relaiei de congruen n cazul
cnd modulul este zero i reciproc.


Probleme rezolvate

R2.1. S se afle restul mpririi numrului A=9851275+970 la 11.
Soluie. Avem: 9856(mod 11), 127510(mod 11), 9702(mod 11).
Atunci 9851275+970(610+2)(mod 11)=627(mod 11).
Restul mpririi numrului A la 11 este 7.
R2.2. S se afle restul mpririi numrului B=4690
7
157
8
-151
5
la 13.
Soluie. Avem relaiile:
469010(mod 13), 1571(mod 13), 1518(mod 13).
20
Atunci
B=4690
7
157
8
-151
5
10
7
1
8
-8
5
(mod 13)=(10
7
-8
5
)(mod 13).
Fiindc 10
3
-1(mod 13), 8
2
-1(mod 13)
10
6
1(mod 13), 8
4
1(mod 13)
10
7
10(mod 13), 8
5
8(mod 13)
obinem B=(10
7
-8
5
)(mod 13)(10-8)(mod 13)2(mod 13).
Deci restul mpririi lui B la 13 este 2.
R2.3. Rezolvai n Z ecuaiile:
a) 4x3x+2(mod 7); b) 5x+63x+4(mod 6).
Soluie. 4x3x+2(mod 7), rezult c 4x-3x2(mod 7), rezult c x2(mod 7)
adic x=7k+2, unde kZ.
b) 5x+63x+4(mod 6) rezult c 2x+64(mod 6), rezult c
2x-2(mod 6) deci x-1(mod 6), adic x=6k-1, kZ.
R2.4. S se arate c pentru orice numr natural n, numrul 10
3n
-1 se divide cu 37.
Soluie. Fiindc 1000=3727+1, rezult 10001(mod 37).
Deci 10
3n
1
n
(mod 37)1(mod 37), adic 10
3n
-1 se divide cu 37.
R2.5. S se demonstreze c numrul N=7+7
3
+7
5
+7
7
+...+7
95
se divide cu 336.
Soluie. Avem 336=748 (numere prime ntre ele). Dm factor comun pe 7 i
obinem: N=7(1+7
2
+7
4
+7
6
+...+7
94
), deci 7|N. Trebuie s mai artm c 48 divide pe
1+7
2
+7
4
+7
6
+...+7
94
.
Dar 1+7
2
+7
4
+7
6
+...+7
94
=49
0
+49
1
+49
2
+49
3
+...+49
47
. Fiindc 491(mod 48), obinem:
) 48 (mod 0 1 ... 1 1 49 ... 49 49 49
ori de 48
47 2 1 0
+ + + = + + + +
43 42 1

Deci N se divide cu 7 i 48 (prime ntre ele) atunci se divide cu 748=336.
R2.6. S se gseasc nN, astfel nct numrul A=n
n+1
+(n+1)
n
s fie divizibil
prin 3.
Soluie. Un numr mprit la 3 poate da unul din resturile: 0, 1, 2. Avem
situaiile:
i) n0(mod 3); ii) n1(mod 3); iii) n2(mod 3).

i) n primul caz din n0(mod 3) obinem n
n+1
0(mod 3). Din n0(mod 3)
obinem n+11(mod 3) i atunci (n+1)
n
1
n
(mod 3), deci (n+1)
n
1(mod 3).
Deci ) 3 (mod 0 / ) 3 (mod 1 1) (n n
n 1 n
+ +
+
.
ii) Din n1(mod 3), rezult n
n+1
1
n+1
(mod 3) i deci n
n+1
1(mod 3).
Din n1(mod 3) obinem n+12(mod 3), rezult c
(n+1)
n
2
n
(mod 3)(-1)
n
(mod 3). Deci n
n+1
+(n+1)
n
[1+(-1)
n
](mod 3), rezult c n
trebuie s fie impar i cum n1(mod 3) rezult c n=6k+1.
ii) Pentru n2(mod 3) obinem n
n+1
2
n+1
(mod 3)(-1)
n+1
(mod 3).
Din n2(mod 3) rezult c n+10(mod 3) i deci (n+1)
n
0(mod 3).
Atunci ) 3 (mod 0 / ) 3 (mod ) 1 ( 1) (n n
1 n n 1 n
+ +
+ +
.
n concluzie avem n=6k+1.

21
3. Principiul paritii

Multe probleme elementare, care de care mai neateptate, folosesc noiunea de
paritate.
Principiul paritii const n separarea cazurilor pare i impare dintr-o situaie.
Regulile paritii:
- suma a dou numere pare este un numr par
- suma a dou numere impare este un numr par
- suma dintre un numr par i altul impar este un numr impar
- produsul dintre un numr par i un numr impar este un numr par
- produsul a dou numere pare este un numr par
- produsul a dou numere impare este un numr impar.


Probleme rezolvate

R3.1. Fie 2 , > n n N .
Demonstrai c numrul fraciilor ireductibile din mulimea
)
`


n
n
n n n
1
,...,
3
,
2
,
1

este par.
Soluie. S artm c dac fracia
n
k
este ireductibil, atunci i fracia
n
k n

este ireductibil. Dac fracia
n
k
este ireductibil atunci k i n sunt prime ntre ele.
S demonstrm c i n-k i n sunt prime ntre ele. Folosim metoda reducerii la
absurd. Presupunem c exist p1 astfel nct p|n i p|(n-k), deci p|[n-(n-k)] adic p|k.
Deci n p | i k p | , adic n i k nu sunt prime ntre ele, fals fiindc
n
k
este ireductibil.
S artm c fraciile
n
k
i
n
k n
sunt diferite. Dac am avea
n
k n
n
k
= am obine
k n 2 = i atunci
k
k
n
k
2
= nu ar mai fi ireductibil. Deci
n
k n
n
k
. Atunci numrul
fraciilor ireductibile din mulimea dat este par.
R3.2. La olimpiada de matematic s-au ntlnit n elevi, dar nu fiecare a dat
mna cu toi ceilali. S se arate c numrul elevilor care au dat mna de un numr
impar de ori este par.
Soluie. Fie
i
x numrul strngerilor de mn pe care le-a realizat elevul cu
numrul de ordin i. Cnd un elev d mna cu un alt elev se realizeaz dou strngeri de
mn, deci numrul total al strngerilor de mn este par, adic
s x x x
n
2 ...
2 1
= + + + (1)
Printre cei n elevi sunt k elevi care au realizat fiecare cte un numr par de
strngeri de mn i n-k elevi care au realizat fiecare cte un numr impar de strngeri
de mn.
22
Atunci (1) se mai poate scrie:
s x x x x x x
n k k k
2 ... ...
impare numere k - n
2 1
pare numere k
' '
2
'
1
= + + + + + + +
+ +
4 4 4 3 4 4 4 2 1 4 4 3 4 4 2 1

Suma alctuit din k numere pare i (n-k) numere impare este un numr par
dac i numai dac numrul numerelor impare este par, deci (n-k) este numr par.
R3.3. Determinai numerele reale
1 2 2 1
,..., ,
+ n
a a a tiind c
1 2 ...
1 2 2 2 1
+ = + + + +
+
n a a a a
n n
i | | ... | | | |
1 1 2 3 2 2 1
a a a a a a
n
= = =
+

Soluie. Fie k a a a a a a
n
= = = =
+
| | ... | | | |
1 1 2 3 2 2 1
. Atunci
k a a
k a a
k a a
k a a
n
n n
=
=
=
=
+
+
1 1 2
1 2 2
3 2
2 1
....... .......... (1)
Adunnd membru cu membru relaiile (1) obinem:
k k k = ... 0 sau ) 1 ... 1 ( 0 = k (2)
Din (2) rezult k=0 sau numrul din parantez este zero. n parantez avnd un
numr impar de 1 obinem c numai k=0 convine. Atunci obinem
1 ...
1 2 2 1
= = = =
+ n
a a a .
R3.4. Ce condiie trebuie s ndeplineasc numrul natural n pentru ca s
existe n numere
n
a a a ,..., ,
2 1
egale cu +1 sau 1, cu proprietatea c:
0 ...
1 1 3 2 2 1
= + + + +

a a a a a a a a
n n n
(1)
Soluie. Suma coninnd n termeni, pentru a avea loc (1) trebuie ca
2
n
termeni
s fie 1 iar
2
n
termeni s fie 1. Deci este necesar ca n s fie par, adic n=2k,
*
N k .
Condiia nu este suficient (exp. 2 1 = = n k i deci 0 2
2 1 1 2 2 1
= + a a a a a a ).
Vom calcula n dou moduri produsul termenilor din (1) i anume:
1 ... ) )( ( ... ) )( (
2 2
2
2
1 1 1 3 2 2 1
= =
n n n n
a a a a a a a a a a a (2)
innd seama c avem k termeni din (1) egali cu +1, iar
|
.
|

\
|
2
n
k termeni egali
cu 1 obinem:
k k
n n n
a a a a a a a a ) 1 ( ) 1 ( ) )( ( ... ) )( (
1 1 3 2 2 1
+ =

(3)
Din (2) i (3) rezult c
k k
) 1 ( ) 1 ( 1 + = , adic trebuie ca k s fie par. Deci
k=2l,
*
N l . Atunci l l k n 4 2 2 2 = = = .
n concluzie n trebuie s fie multiplu de 4.

23
4. Principiul invariantului

Invariantul este o mrime, o relaie, sau o proprietate care rmne neschimbat
n urma aplicrii sau interveniei unei transformri.
Deci o situaie iniial este supus n mod repetat unor transformri. De obicei
se cere s se demonstreze c n urma acestor transformri nu se poate ajunge la o
anumit form. Aceasta se poate face alegnd caracteristica obiectului care a fost supus
transformrii, adic "invariantul" transformrii. Dac n final obiectul nu posed
"invariantul" atunci el nu poate fi obinut n urma transformrilor descrise.


Probleme rezolvate

R4.1. ntr-un sistem cartezian xOy din punctul ) , ( y x este permis
deplasarea ntr-unul din cele patru puncte: ) 1 , 1 ( y x , ) 1 , 1 ( + y x , ) 1 , 1 ( + x x ,
) 1 , 1 ( + + y x . Demonstrai c din punctul (0,0) nu se poate ajunge prin deplasri
succesive n punctul (2003,2004).
Soluie. Observm c un punct ) , ( y x avnd suma coordonatelor y x + un
numr par se poate deplasa ntr-un punct avnd suma coordonatelor tot par:
I. ) 1 , 1 ( y x are suma coordonatelor 2 + y x
II. ) 1 , 1 ( + y x are suma coordonatelor y x +
III. ) 1 , 1 ( + x x are suma coordonatelor x 2
IV. ) 1 , 1 ( + + y x are suma coordonatelor y x + + 2 .
Punctul (0,0) are suma coordonatelor 0, deci un numr par, el se poate deplasa
succesiv n puncte cu aceeai paritate a sumei coordonatelor, deci nu se poate deplasa
n punctul (2003,2004) care are suma coordonatelor impar.
R4.2. O camer are dimensiunile podelei de 7m i 10m. n cele patru coluri
ale camerei se aeaz cte un dulap avnd baza ptrat cu latura de 1m. S se arate c ce
rmne din suprafaa podelei nu poate fi acoperit cu plci dreptunghiulare de
dimensiuni 3m1m.
Soluie. Se mparte camera ntr-o reea de ptrate cu latura 1m pe care le
vopsim n trei culori: rou, alb, negru ca mai jos:
RANRANRANR
ANRANRANRA
NRANRANRAN
RANRANRANR
ANRANRANRA
NRANRANRAN
RANRANRANR
Obinem 24 de R, 23 de A, 23 de N. Eliminnd colurile rmn 20 de ptrele
roii, 23 de ptrele albe, 23 de ptrele negre. Dar oricum am aeza o plac de 31
ea acoper un ptrel rou, unul alb i unul negru. Dac s-ar putea acoperi suprafaa cu
24
un numr ntreg de plci ar trebui s existe acelai numr de ptrele pentru fiecare
culoare.
R4.3. Pe o tabl scriem numerele de la 1 la 4n-1 (n1, N n ).
Printr-o operaie nelegem c tergem dou numere oarecare de pe tabl i n locul lor
scriem valoarea absolut a diferenei celor dou numere. S se demonstreze c dup
4n-2 operaii pe tabl rmne un singur numr par.
Soluie. La orice pas i, nainte de tergerea a dou numere x i y, fie
1 + i
S suma
numerelor de pe tabl i
i
S suma numerelor de pe tabl dup tergerea numerelor x i
y. Atunci avem:

<
= + =
+
y x y
y x x
y x y x S S
i i
daca , 2
daca , 2
| |
1

Deci
i i
S S
+1
este totdeauna un numr par. Suma:
n n
n n
n 2 ) 1 4 (
2
4 ) 1 4 (
1 4 ... 3 2 1 =

= + + + +
este un numr par, rezult c
1
S este tot un numr par. Dar
1
S este ultimul numr de
pe tabl. Deci pe tabl a rmas un numr par.
R4.4. Se consider numerele 3 1 , 3 1 , 3 + . Dup un pas este permis
scrierea a noi trei numere, nlocuind fiecare din numerele date prin semisuma celorlalte
dou. Se poate efectua de cteva ori aceast operaie, s se obin tripletul
3 2 , 3 2 , 2 + ?
Soluie. n acest caz rmne invariant suma numerelor. Dac x, y, z sunt
numerele, atunci ele se nlocuiesc cu
2
,
2
,
2
y x x z z y + + +
, ce au suma
z y x
z y x z y z x y x
+ + =
+ +
=
+
+
+
+
+
2
) ( 2
2 2 2

Suma numerelor iniiale este 5 3 1 3 1 3 = + + + , iar suma numerelor
finale este: 6 3 2 3 2 2 = + + + . Deci plecnd de la numerele 3 1 , 3 1 , 3 + nu
se poate ajunge la numerele 3 2 , 3 2 , 2 + .
R4.5. Considerm numerele: 1, 2, 3,..., 2001. Alegem dou numere x i y din
cele de mai sus i le nlocuim cu
y x
xy
z
+ +
=
1
. Repetm procedeul cu alte dou
numere, .a.m.d. Demonstrai c dup 2000 de pai se obine ntotdeauna acelai numr
(indiferent de modul n care lum perechile). Care este acest numr?
Soluie. Din
y x
xy
z
+ +
=
1
obinem xy yz xz z = + + , de unde avem
xy xyz yz xz z xyz + = + + + , deci + = + + + ) 1 ( ) 1 ( ) 1 ( z xy x z x yz
25
) 1 ( ) 1 )( 1 ( ) 1 ( ) )( 1 ( + = + + + = + + z xy y x z z xy z yz x , care se mai poate scrie
z
z
xy
y x 1 ) 1 )( 1 ( +
=
+ +
, deci
z y
y
x
x 1
1
1 1
+ =
+

+
sau
z y x
1
1
1
1
1
1 + =
|
|
.
|

\
|
+
|
.
|

\
|
+ .
Dac notm cu
n
x x x ,..., ,
2 1
numerele existente la un moment dat, atunci
expresia
|
|
.
|

\
|
+
|
|
.
|

\
|
+
|
|
.
|

\
|
+ =
n
x x x
E
1
1 ...
1
1
1
1
2 1

este invariant. Dac xeste ultimul numr, avem:
x
1
1
2001
2002
...
2
3
1
2
2001
1
1 ...
2
1
1
1
1
1 + = =
|
.
|

\
|
+
|
.
|

\
|
+
|
.
|

\
|
+
Deci
x
1
1 2002 + = , de unde
2001
1
= x .
R4.6. Pe o tabl sunt scrise numerele: 1, 2, 3,..., 1986, 1987. La fiecare pas se
terg cteva numere i se scrie restul dat de suma numerelor terse la mprirea cu 7.
La un moment dat, au rmas pe tabl dou numere, unul dintre ele fiind 987. Care este
acel numr?
Soluie. n acest caz invariantul este restul dat la mprirea cu 7 de suma:
1+2+3+...+1986+1987=19877142, adic zero.
Dac x este numrul cerut, rezult c x+987 se divide cu 7, de unde x se divide
cu 7. Fiindc 987 nu poate fi restul la mprirea cu 7, atunci x trebuie s fie acest rest
i deci 6 x . Pentru c x se divide cu 7, rezult 0 = x .

26
5. Metoda reducerii la absurd

Metoda reducerii la absurd este o metod specific de demonstraie n
matematic. La baza acestei metode st una din legile fundamentale ale logicii clasice:
legea terului exclus, ce are urmtorul enun:
Din dou propoziii contradictorii una este adevrat, cealalt fals, iar a treia
posibilitate nu exist.
Legea terului exclus nu ne precizeaz care din cele dou propoziii este
adevrat i care este fals.
Cnd la dou propoziii contradictorii aplicm legea terului exclus este
suficient s stabilim c una dintre ele este fals pentru a deduce c cealalt este
adevrat.
Metoda reducerii la absurd const n a admite n mod provizoriu, ca adevrat
propoziia contradictorie propoziiei de demonstrat, apoi pe baza acestei presupuneri se
deduc o serie de consecine care duc la un rezultat absurd, deoarece ele contrazic sau
ipoteza problemei date sau un adevr stabilit mai nainte. Mai departe raionm astfel:
dac presupunerea ar fi fost adevrat, atunci n urma raionamentelor logic corecte ar
fi trebuit s ajungem la o concluzie adevrat, deoarece am ajuns la o concluzie fals,
nseamn c presupunerea noastr a fost fals. Aceasta duce la concluzia c
presupunerea fcut nu este posibil i rmne ca adevrat concluzia propoziiei date.
Metoda reducerii la absurd nu se reduce la propoziia c "a demonstra o
propoziie este acelai lucru cu a demonstra contrara reciprocei ei", deoarece pot aprea
i situaii n care nu se contrazice ipoteza ci o alt propoziie (un rezultat cunoscut, o
axiom, o teorem). Metoda reducerii la absurd se folosete att n rezolvarea
problemelor de calcul (de aflat) ct i la rezolvarea problemelor de "demonstrat".
Metoda este des utilizat n demonstrarea teoremelor reciproce, precum i n
demonstrarea teoremelor de unicitate.


Probleme rezolvate

Vom prezenta cteva exerciii i probleme rezolvate n care folosim metoda
reducerii la absurd.

R5.1. Suma a 12 numere naturale nenule este 77. Artai c printre ele se afl
cel puin dou numere egale.
Soluie. Presupunem c exist 12 numere naturale nenule distincte ce au suma
77. Dac le considerm pe cele mai mici, suma lor este:
78 13 6
2
13 12
12 11 ... 3 2 1 = =

= + + + + +
Cum suma celor mai mici 12 numere naturale nenule distincte este mai mare
dect suma dat, 77, rezult c presupunerea fcut este fals. Deci printre numerele
considerate exist cel puin dou numere egale.
27
R5.2. Suma a trei numere naturale este 139. Demonstrai c cel puin unul
dintre ele este mai mare sau egal cu 47.
Soluie. Folosim metoda reducerii la absurd, presupunem concluzia fals, adic
nici unul dintre numere nu este mai mare sau egal cu 47. Fie a,b,c numerele. Deci
a<47, b<47, c<47. Fiindc a,b,c sunt numere naturale rezult c 46 a , 46 b ,
46 c . innd seama c 139 = + + c b a obinem: 138 + + c b a sau 139138,
ceea ce este absurd. Atunci presupunerea fcut este fals i deci concluzia este
adevrat, adic cel puin unul dintre numere este mai mare sau egal cu 47.
R5.3. S se arate c pentru orice numr natural diferit de zero fracia
1 2
1 2
+

n
n

este ireductibil.
Soluie. Presupunem c fracia dat nu este ireductibil, atunci exist un numr
natural d diferit de unu astfel nct ) 1 2 ( | n d i ) 1 2 ( | + n d , de unde
)] 1 2 ( ) 1 2 [( | + n n d adic 2 | d . Fiindc 1 d , rezult d=2. Atunci rezult c
) 1 2 ( | 2 n ceea ce este absurd. Deci presupunerea fcut este fals, i deci fracia este
ireductibil.
R5.4. S se determine numrul elementelor mulimii:
)
`

=
+
+
= 2003 ,..., 3 , 2 , 1 ,
1 2
1
n
n
n
M
Soluie. Mulimea are attea elemente cte valori distincte are fracia
1 2
1
+
+
n
n
,
n=1,2,...,2003. Presupunem prin absurd c exist
1
n i
2
n , cu
2 1
n n pentru care
fracia are aceeai valoare, adic:
+ + = + +
+
+
=
+
+
) 1 2 )( 1 ( ) 1 2 )( 1 (
1 2
1
1 2
1
1 2 2 1
2
2
1
1
n n n n
n
n
n
n

2 1 1 2 2 1 2 1 2 1
1 2 2 1 2 2 n n n n n n n n n n = + + + = + + + .
Am ajuns la o contradicie pentru c
1
n a fost presupus diferit de
2
n . Deci
mulimea M are 2003 elemente.
R5.5. tiind c x, y, z sunt numere reale, s se arate c urmtoarele inegaliti
nu pot fi simultan adevrate:
1 2 > z x
1 z y
1 2 x y
Soluie. Folosim metoda reducerii la absurd. Presupunem c toate inegalitile
sunt adevrate. nmulim a doua inegalitate cu 2 i adunndu-le obinem:
0 2 2 2 2 > + + x y y z z x ,
adic 0>0, ceea ce este absurd. Deci presupunerea fcut este fals. Deci inegalitile
considerate nu pot fi simultan adevrate.
R5.6. S se arate c numrul 5 3 + este iraional.
28
Soluie. Aplicm metoda reducerii la absurd. Presupunem c 5 3 + este
raional; rezult c exist xQ astfel nct x = + 5 3 , de unde obinem c
2
15 2 5 3 x = + + sau
2
15 2 8 x = + , de unde Q

=
2
8
15
2
x
, fals. Deci
presupunerea fcut este fals i deci 5 3 + este iraional.
R5.7. Se consider un ptrat cu latura 1cm i 10 puncte n interiorul su.
Demonstrai c printre cele 10 puncte date exist dou puncte astfel nct distana
dintre ele s nu depeasc
3
2
cm.
Soluie. Cu metoda reducerii la absurd, presupunem c nu exist astfel de 2
puncte cu distana dintre ele s nu depeasc
3
2
cm. mprim ptratul n 9 ptrate
mai mici cu latura
3
1
cm. Diagonala unui astfel de ptrat va avea lungimea
3
2
9
2
3
1
3
1
2 2
= =
|
.
|

\
|
+
|
.
|

\
|
cm calculat cu teorema lui Pitagora. Cele 10 puncte fiind
situate n interiorul ptratului "mare" nseamn c putem aeza 2 puncte n dou ptrate
"mici" alturate astfel nct distana dintre ele s fie mai mare dect
3
2
cm. Pentru ca
problema s fie rezolvat trebuie s existe attea ptrate "mici" cte puncte (zece). Am
ajuns la o contradicie, rezult c presupunerea fcut este fals, rezult c exist 2
puncte la care distana dintre ele nu depete
3
2
cm.
1
3
2
3
1
29
R5.8. Fie ABC i punctele M, N, P diferite de vrfurile triunghiului cu
MBC, NAC, PAB astfel nct 1
PB
PA
NA
NC
MC
MB
= , atunci punctele M, N, P sunt
coliniare. (Reciproca teoremei lui Menelaus)
Soluie. Folosim metoda reducerii la absurd, presupunem c punctele M, N, P
nu sunt coliniare. Unind M cu P printr-o dreapt ce taie pe AC ntr-un punct N' diferit
de N i conform teoremei directe a lui Menelaus avem:
1
PB
PA
A N'
C N'
MC
MB
= (1)
Din ipotez avem:
1
PB
PA
NA
NC
MC
MB
= (2)
Din relaiile (1) i (2) obinem
NA
NC
A N'
C N'
= i deci N'N. Deci presupunerea
c punctele M, N, P nu sunt coliniare este fals. Deci are loc relaia din ipotez.
Reciproca teoremei lui Menelaus constituie una din principalele metode de
demonstrare a coliniaritii multor triplete de puncte.
R5.9. Se consider triunghiul ABC i punctele K, M, L situate pe laturile
(AB), (BC), (AC) i diferite de vrfurile triunghiului. S se demonstreze c cel puin
una din ariile triunghiurilor MAL, KBM, LCK nu depete un sfert din aria
triunghiului ABC.
Soluie. Avem relaiile:
2
sinA AC AB
S
[ABC]

= ,
2
sinA AL AM
S
[AML]

=
Deci
AC AB
AL AM
S
S
[ABC]
[AML]

= (1)
Analog obinem:
BC BA
BK BM
S
S
[ABC]
[BMK]

= (2)
i
A
M
B K C
L
30
CB CA
KC CL
S
S
[ABC]
[CLK]

= (3)
Folosim metoda reducerii la absurd. Presupunem c
4
1
S
S
,
4
1
S
S
,
4
1
S
S
[ABC]
[CLK]
[ABC]
[BMK]
[ABC]
[AML]
> > >
innd seama de relaiile (1), (2), (3), obinem
64
1
CB CA
KC CL
BC BA
BK BM
AC AB
AL AM
>


sau
64
1
BC BC
CK BK
AB AB
AM BM
AC AC
CL AL
>

(4)
Cu inegalitatea dintre media aritmetic i geometric avem:
2
AC
2
CL AL
CL AL =
+
, deci
4
1
AC AC
CL AL


i analog
4
1
AB AB
BM AM

,
4
1
BC BC
CK BK

.
nmulind membru cu membru aceste ultime trei inegaliti obinem:
64
1
BC BC
CK BK
AB AB
BM AM
AC AC
CL AL

(5)
Din (4) i (5) rezult c presupunerea fcut este fals, adic gsim un triunghi
cu aria ce nu depete
4
S
[ABC]
.
R5.10. ntr-un triunghi ascuitunghic neechilateral, printr-un vrf este dus
nlimea, prin altul mediana, iar prin cel de-al treilea bisectoarea. Artai c aceste linii
nu pot forma prin intersecie un triunghi echilateral.
Demonstraie. Considerm triunghiul ABC cu nlimea AA', mediana BB' i
bisectoarea CC'.
Aplicm metoda reducerii la absurd. Presupunem c triunghiul DEL este
echilateral. Din triunghiul dreptunghic CDA' obinem:
m(DCA')=90-60=30
A
C'
B A' C
B'
D
E
L
31
Deci m(C)=60 i atunci m(B'CE)=30. Dar DELB'EC (opuse la
vrf), atunci m(B'EC)=60. n triunghiul B'EC avem
m(EB'C)=180-(30+60)=90
Fiindc BB' este median rezult c (BA)(BC). Dar m(C)=60 i atunci
rezult c triunghiul ABC este echilateral. dar din ipotez triunghiul ABC nu este
echilateral. Atunci presupunerea fcut (c triunghiul DEL este echilateral) este fals i
deci triunghiul DEL nu este echilateral.
32
6. Probleme de numrare

Multe probleme din viaa cotidian cer numrarea elementelor unor mulimi
finite, ale prilor unei mulimi, etc. i de aici importana aprofundrii operaiei de
numrare prin probleme care conduc la numrarea elementelor unor mulimi diverse.
Domeniul matematicii n care se studiaz astfel de probleme se numete combinatoric.
Pentru a aborda diverse probleme de numrare un rol important l joac noiunea de
partea ntreag, numrul divizorilor naturali ai unui numr natural, forma canonic a
unui numr natural n (descompunerea n mod unic n produs de factori primi), etc.
1) Prin partea ntreag a unui numr x nelegem cel mai mare numr ntreg
care nu l depete pe x i se noteaz [x].
Avem x x x < ] [ 1 .
Folosim partea ntreag, de exemplu cnd numrm multiplii unui numr
natural p cuprins n mulimea: {1,2,3,...,n}.
2) Orice numr natural n, diferit de zero, se descompune n mod unic ntr-un
produs de factori primi:
k
k
p p p n

= ...
2 1
2 1
, (1)
unde
k
p p p ,..., ,
2 1
sunt numere prime, iar
k
,..., ,
2 1
sunt numere nenule. Relaia
(1) se numete forma canonic a lui n.
Numrul divizorilor naturali ai lui n este: ) 1 )...( 1 )( 1 (
2 1
+ + +
k
.
Vom prezenta n continuare cteva probleme de aritmetic, teoria numerelor,
geometrie, care se ncadreaz la aceast problematic.
R6.1. Care este exponentul lui 3 n descompunerea n factori primi a
numrului 100! (100!=123...100).
Soluie. Dintre numerele 1,2,3,4,...,100 fiecare al treilea este divizibil cu 3.
Fiindc 100=333+1, rezult c de la 1 la 100 sunt 33 de numere divizibile cu 3. Dintre
aceste 33 de numere fiecare al treilea este divizibil cel puin cu puterea a 2-a a lui 3.
Fiindc 33:3=11, rezult c sunt 11 numere divizibile cu 3
2
. Dintre cele 11 fiecare al 3-
lea este divizibil cu 3
3
. Fiindc 11=33+1, rezult c sunt 3 astfel de numere. Dintre
aceste 3 numere unul este divizibil cu 3
4
. Nu exist nici un numr dintre primele 100,
divizibil cu 3
5
pentru c 3
5
>100. Atunci exponentul lui 3 din descompunerea n produs
de factori primi a numrului 100! este: 33+11+3+1=48.
Fiindc la mpririle efectuate am reinut numai cturile, acestea reprezint de
fapt prile ntregi ale numerelor:
4 3 2
3
100
,
3
100
,
3
100
,
3
100
. Deci exponentul lui 3 din
descompunerea n factori primi a numrului 100! este:
48 1 3 11 33
3
100
3
100
3
100
3
100
4 3 2
= + + + =

.
Cu acelai raionament se arat c exponentul numrului prim p din
descompunerea n factori primi a lui ! n (n!=123...n) este:
33
...
3 2
+

p
n
p
n
p
n

R6.2. Se consider ntr-un plan 5 puncte, oricare trei necoliniare.
a) Cte drepte determin aceste puncte?
b) Cte triunghiuri determin aceste puncte?
c) Dac avem n puncte (oricare trei necoliniare), cte drepte i cte triunghiuri
determin?
Soluie. a) Fie A
1
,A
2
,A
3
,A
4
,A
5
punctele din ipotez. Punctul A
1
determin cu
celelalte 4 puncte un numr de 4 drepte. Din cele 5 puncte pleac 45=20 semidrepte.
Fiecare dreapt a fost numrat de dou ori (de exemplu A
1
A
2
i A
2
A
1
). Atunci
numrul dreptelor care trec prin cele 5 puncte este 20:2=10.
n general, dac avem n puncte (n3) i oricare trei sunt necoliniare atunci ele
determin
2
) 1 ( n n
drepte.
Fie punctele A
1
,A
2
,...,A
n
. Fixnd punctul A
i
, acesta va determina cu celelalte
puncte n-1 drepte. Avnd n puncte, din ele pleac n(n-1) semidrepte. Fiecare dreapt
este numrat de dou ori: A
i
A
k
i A
k
A
i
. Atunci n puncte (oricare trei necoliniare)
determin
2
) 1 ( n n
drepte.
b)-c) Pentru numrul de triunghiuri considerm cazul cnd avem n puncte
(oricare trei necoliniare). Fixm un vrf A
i
de exemplu, fapt ce poate fi realizat n n
moduri. Fixm al doilea vrf A
j
, realizabil n n-1 moduri (dup prima fixare). Fixm al
3-lea vrf A
k
, realizabil n n-2 moduri. innd seama cum au fost alese vrfurile,
obinem ) 2 )( 1 ( n n n variante. Fiecare triunghi A
i
A
j
A
k
a fost numrat de 6 ori:
A
i
A
j
A
k
, A
i
A
k
A
j
, A
j
A
i
A
k
, A
j
A
k
A
i
, A
k
A
j
A
i
, A
k
A
i
A
j
. Atunci numrul de triunghiuri
determinat de n puncte (oricare trei necoliniare) este
6
) 2 )( 1 ( n n n
.
R6.3. Determinai numrul diagonalelor unui poligon convex cu n laturi
(n4).
Soluie. Din fiecare vrf pleac n-3 diagonale pentru c un vrf i cu dou
adiacente nu determin diagonale. Fiind n vrfuri avem n(n-3) segmente. Dar fiecare
diagonal a fost numrat de dou ori, deci numrul diagonalelor unui poligon convex
cu n laturi este
2
) 3 ( n n
.
Altfel. Dac avem n puncte distincte (oricare trei necoliniare), ele determin
2
) 1 ( n n
drepte. Pentru a afla numrul diagonalelor trebuie s scdem numrul
laturilor. Obinem:
2
) 3 (
2
3
2
2
2
) 1 (
2 2

=

=
n n n n n n n
n
n n
.
34
R6.4. S se determine numrul minim de monede de 1, 3, 5 euro de care avem
nevoie pentru a plti orice sum ntreag cuprins ntre 1 i 5n euro.
Soluie. i) Pentru a plti orice sum ntreag cuprins ntre 1 i 5n euro avem
nevoie de cel puin 2 + n monede: pentru a plti sumele de 2 euro i 5n euro avem
nevoie de dou monede de un euro i nc cel puin n monede.
ii) Pentru a plti toate sumele ntregi ntre 1 euro i 5n euro sunt suficiente
2 + n monede: dou monede de 1 euro, o moned de trei euro i 1 n monede de 5
euro.
Orice numr natural cuprins ntre 1 i 5n inclusiv are una din formele
5 5 , 4 5 , 3 5 , 2 5 , 1 5 + + + + + k k k k k unde 1 0 n k .
Sumele de forma 1 5 + k pot fi pltite cu k din cele 1 n monede de 5 euro
) 1 ( k n i una dintre monedele de 1 euro. Sumele de forma 2 5 + k pot fi pltite cu
k monede 5 euro i monedele de 1 euro. Sumele de forma 3 5 + k i 4 5 + k le putem
plti cu orice monede de 5 euro i moneda de 3 euro respectiv k monede de 5 euro o
moned de 3 euro i una de 1 euro. Sumele de forma 5 5 + k ) 1 ( k n pot fi pltite
cu k monede de 5 euro i cu monede de 1 i 3 euro. Deci numrul minim cerut este
2 + n .


6.2. Principiul cutiei sau principiul lui Dirichlet

Sunt multe probleme de matematic cu enunuri inedite ce pot fi abordate cu
mijloace ale gndirii cotidiene, fr a fi nevoie de metode rafinate. Un exemplu
elocvent este principiul cutiei sau principiul lui Dirichlet. Ceea ce caracterizeaz
problemele n care acest principiu se folosete este dificultatea de a le aborda pe ci
cunoscute. ntr-o formulare fr pretenii acest principiu revine la observaia c dac
avem dou cutii n care trebuie puse trei obiecte, ntr-una din ele va trebui s aezm
cel puin dou obiecte. Mai general, dac repartizm un numr mai mare de n obiecte
n n clase, atunci cel puin ntr-o clas vor fi cel puin dou obiecte. Deci avem
Teorema 6.2.1. Considerm o mulime nevid A i A
1
,A
2
,...,A
n
o partiie a
mulimii A (adic A
1
A
2
...A
n
=A i A
i
A
j
=, pentru ij. Dac avem n+1 elemente
din A: a
1
,a
2
,...,a
n
,a
n+1
atunci exist o submulime A
i
a partiiei care s conin cel puin
dou elemente ale mulimii {a
1
,a
2
,...,a
n
,a
n+1
}.
n general principiul cutiei este un principiu de numrare. n ultimul timp acest
principiu a cptat o mare popularitate, fiind pus la baza unui mare numr de probleme,
dintre care unele deosebit de dificile. Vom prezenta cteva exemple n care se folosete
acest principiu n aritmetic, geometrie.
R6.2.1. Considerm mulimea } ,..., , {
2 1 n
a a a A = cu elementele numere
ntregi. S se demonstreze c A are cel puin o parte nevid cu proprietatea c suma
elementelor sale se divide cu n.
Soluie. Dac a este numr ntreg i n numr natural, exist numerele q i r
unice astfel nct r q n a + = cu Z q i } 1 ,..., 2 , 1 , 0 { n r . Vom aplica principiul
35
cutiei. Considerm urmtoarele n submulimi ale lui A: } {
1 1
a A = , } , {
2 1 2
a a A = ,
} , , {
3 2 1 3
a a a A = ,..., } ,..., , {
2 1 n n
a a a A = .
Dac notm cu
i
S cu n i , 1 = suma elementelor fiecrei mulimi avem:
1 1
a S = ,
2 1 2
a a S + = ,
3 2 1 3
a a a S + + = ,...,
n n
a a a S + + + = ...
2 1
.
Dac unul din numerele
i
S cu n i , 1 = se divide cu n problema este rezolvat.
Dac nu, cele n resturi obinute prin mprirea cu n a numerelor
i
S aparin mulimii
{1,2,3,...,n-1} ce are n-1 elemente diferite. Deci exist cu siguran dou numere
i
S i
j
S care dau acelai rest la mprirea cu n. Fie
i i
a a a S + + + = ...
2 1
i
j j
a a a S + + + = ...
2 1
cele dou numere. Fie j i < . Fiindc n divide pe
i j
S S
rezult c submulimea cutat este } ,..., , {
2 1 j i i
a a a B
+ +
= .
R6.2.2. S se arate c oricum am alege cinci numere ntregi, exist dou dintre
acestea, care au suma sau diferena divizibile cu 7.
Soluie. La mprirea cu 7 a unui numr se obine unul din resturile
0,1,2,3,4,5,6. Ptratul su va da la mprirea cu 7 unul din resturile 0,1,2,4. Deoarece
avem cinci numere a,b,c,d,e, cele cinci ptrate ale lor nu pot da la mprirea cu 7 dect
unul din cele patru resturi: 0,1,2,4. Conform principiului cutiei cel puin dou din
aceste cinci ptrate dau la mprirea cu 7 acelai rest. Deci exist } , , , , { , e d c b a y x
astfel nct
2 2
y x se divide cu 7. Deci 7 divide pe ) )( (
2 2
y x y x y x + = , dar
fiind i prim rezult c 7 divide pe y x + sau 7 divide pe y x .
R6.2.3. Patru drepte distincte situate ntr-un plan, l mpart n mai multe
regiuni distincte. S se arate c oricum s-ar aeza 12 puncte n acest plan astfel nct
nici unul s nu aparin dreptelor date, cel puin dou dintre ele se afl n aceeai
regiune.
Soluie. Dreptele fiind distincte pot fi amplasate n felul urmtor:



a)
b)
c) d)
36


Numrul maxim de regiuni este 11 i se obine n cazul i). Regiunile n care a
fost mprit planul vor fi "csuele" din principiul cutiei. Dac am aeza cte un punct
n fiecare regiune am avea nevoie de 11 puncte. Avnd ns 12 puncte, rezult c n cel
puin o regiune vor fi dou puncte.
R6.2.4. Considerm nou puncte ntr-un ptrat cu latura de lungime 1. S se
demonstreze c exist un triunghi cu vrfurile n trei din cele nou puncte a crui arie
s fie cel mult egal cu
8
1
.
Soluie. Unind dou cte dou mijloacele laturilor opuse n ptratul dat,
obinem o mprire a acestuia n patru ptrate de arie
4
1
. Oricum am plasa cele nou
puncte, ntotdeauna trei se vor afla n interiorul sau pe laturile aceluiai ptrat. Fie A,
B, C cele trei puncte situate n ptratul EFGH. S artm c aria triunghiului ABC este
mai mic sau egal cu
8
1
. Ducem prin A, B, C paralele la EH. Una dintre acestea se va
afla ntre celelalte dou, deci va intersecta latura opus vrfului prin care trece. Fie AQ
aceast paralel la EH, Q[BC]. Ducem BNAQ i CPAQ (N,PAQ). Atunci avem:
+ =

= + = CP) (BN
2
AQ
2
CP AQ
2
BN AQ
S S S
[ACQ] [ABQ] [ABC]

e) f)
g) h)
i)
37
8
1
4
1
2
1
2
1
2
1
] [
= = =
EHGF
S HG EH .
Deci
8
1
] [

ABC
S . Egalitatea are loc dac i numai dac
HG EH CP) AQ(BN = + , deci NQ=HF i BN+CP=HG. Deci egalitatea se obine
cnd o latur a triunghiului coincide cu o latur a ptratului i cellalt vrf al
triunghiului se afl pe latura opus.
R6.2.5. Considerm 17 drepte care mpart un ptrat n dou patrulatere care au
raportul ariilor
6
1
. S se arate c cel puin 5 dintre aceste drepte trec prin acelai punct.
Soluie. Fiecare din cele 17 drepte nu poate tia dou laturi consecutive ale
ptratului, deoarece atunci ptratul ar fi mprit ntr-un triunghi i un pentagon. Deci
fiecare dreapt mparte ptratul n dou trapeze dreptunghice care au aceeai nlime.
Fie a una din aceste drepte care taie laturile AB i CD n punctele T i R. Dac L i K
sunt mijloacele laturilor AD respectiv BC, iar E
1
este punctul de intersecie al dreptelor
Q i TR, avem:
6
1
L E
KE
2
AD DR) (AT
2
BC RC) (TB
S
S
1
1
[ATRD]
[BTRC]
= =
+
+
=
Am inut seama c trapezele au nlimile egale i liniile mijlocii au lungimile:
2
RC TB
K E
1
+
= ,
2
DR AT
LE
1
+
= .
Pe dreapta LK exist dou puncte care mpart segmentul LK n raportul
6
1
. Fie
al doilea punct E
2
. Atunci avem:
6
1
L E
K E
L E
KE
2
2
1
1
= = .
Fiindc ntr-un ptrat exist numai dou segmente care unesc mijloacele
laturilor opuse rezult c n interiorul ptratului exist exact patru puncte: E
1
,E
2
,E
3
,E
4

care mpart liniile mijlocii ale ptratului n raportul
6
1
. deci oricare din cele 17 drepte
E F
L
H
G
R
M T K
E
H G
F
P
B
Q
C
N
A
38
trece prin unul din punctele E
1
,E
2
,E
3
,E
4
. Fiindc avem 17 drepte care trec prin patru
puncte, conform principiului "cutiei" cel puin 5 drepte trec prin acelai punct.
R6.2.6. S se arate c oricum am aeza 37 puncte n interiorul unui triunghi
echilateral cu latura de lungime 1, exist cel puin dou puncte astfel nct distana
dintre ele s nu depeasc 0,1(6).
Soluie. mprim fiecare latur a triunghiului n 6 segmente cu lungimea
6
1
.
Prin punctele de diviziune ducem paralele la laturile triunghiului i obinem
1+3+5+7+9+11=36=6
2
triunghiuri echilaterale cu latura de lungime
6
1
. Considernd
37 puncte n triunghiul iniial, cel puin dou dintre acestea se vor afla n interiorul (sau
pe laturi) unui triunghi cu latura ) 6 ( 1 , 0
6
1
= , i deci distana dintre acestea va fi cel
mult
6
1
.
R6.2.7. S se arate c, oricum am aeza 1
2
+ n puncte n interiorul unui
triunghi echilateral cu latura de lungime 1, exist cel puin dou puncte astfel nct
distana dintre ele s nu depeasc
n
1
.
Soluie. mprim fiecare latur a triunghiului n n segmente cu lungimea
n
1
.
Ducnd paralele la laturile triunghiului prin punctele de diviziune, triunghiul se
descompune n
2
) 1 2 ( ... 7 5 3 1 n n = + + + + + triunghiuri echilaterale cu latura de
lungime
n
1
. Csuele din principiul cutiei sunt acum cele
2
n triunghiuri echilaterale.
Considernd 1
2
+ n puncte n interiorul triunghiului iniial evident cel puin dou
D
L
A
a
T
B
E
2
E
1
K
R
C
39
dintre acestea se vor afla n interiorul unui triunghi (sau pe laturi) cu latura de lungime
n
1
i distana dintre acestea va fi cel mult
n
1
.

B C
A

40
7. Ecuaii n Z. Ecuaii diofantice

7. 1 Consideraii teoretice

Ecuaiile n Z pot fi cu coeficieni n Z, sau cu coeficieni ntr-o alt mulime, dar n
ambele cazuri se cer soluii n Z. Restricia impus de soluii n Z conduce la discuia
ecuaiei pe cazuri n funcie de coeficieni. Cea mai cunoscut ecuaie n care
necunoscutele nu sunt liniare este cea pitagoric, adic: x
2
+y
2
=z
2
. Ecuaia a fost
studiat de Pitagora n legtur cu triunghiurile dreptunghice cu laturi numere naturale.
Pentru ecuaia dat, dac tripletul (x
0
, y
0
, z
0
) este soluie a ecuaiei atunci (kx
0
, ky
0
, kz
0
)
cu kZ este de asemenea soluie, deci este suficient s determinm triplete n care
elementele componente sunt prime ntre ele, n acest sens dm urmtoarea:
Teorema 7. 1. 1. Orice soluie (x, y, z) a ecuaiei x
2
+y
2
=z
2
cu componente prime ntre
ele este de forma :x=m
2
-n
2
;y=2mn;z=m
2
+n
2
, cu (m, n)=1;m>n.
Demonstraie:(m
2
-n
2
)
2
+(2mn)
2
=(m
2
+n
2
)
2
deci este soluie.
S artm c(x, y, z)=1. Numerele x, y nu pot fi ambele impare, deoarece dac sunt
impare atunci:x
2
+y
2
=4k+2 iar z par z
2
=4p(contradicie), ca atare exact unul din
numerele x, y este par. Dac d=(x, y, z) i d 2 d|(m
2
+n
2
)+(m
2
-n
2
) d|2m
2
i d|2n
2

i cum (m, n)=1 d|2 m
2
+n
2
par, dar m, n sunt de pariti diferite, deci d=1.
Reciproc, dac (x, y, z)=1 i este soluie, y=2a x, z sunt impare z+x, z-x sunt
pare, atunci:z+x=2b;z-x=2c. Avem (b, c)=1 deoarece (z, x)=1. Mai avem :
4a
2
=y
2
=z
2
-x
2
=(z-x)(z+x)=4bc a
2
=bc i b=m
2
, c=n
2
deoarece (b, c)=1.
Obinem:x=b-c=m
2
-n
2
z=b+c=m
2
+n
2
y=2mn
care este o soluie cu componentele prime ntre ele.

Definitie 7. 1. 2. O ecuaie de forma a
1
x
1
+a
2
x
2
++a
n
x
n
= b, unde a
1
, a
2
, , a
n
, b sunt
ntregi fixai se numete ecuaie diofantic liniar.
n mod obinuit se cere rezolvarea acestei ecuaii n Z. Vom studia aceast ecuaie
pentru dou necunoscute, i avem:
Teorema 7. 1. 3. Ecuaia ax+by=c are soluii n Z, dac i numai dac (a, b)|c
Demonstraie: Fie d=(a, b) a=da
1
; b=db
1
cu (a
1
, b
1
)=1 i avem: da
1
x+db
1
y=c
d(a
1
x+b
1
y)=c d|c (c=dc)
Dac d=(a, b) atunci () x
1
, x
2
Z pentru care : ax
1
+bx
2
=d (algoritmul lui Euclid) i
nmulind cu c avem a(x
1
c)+b(x
2
c)=dc aX
1
+bX
2
=c, deci are soluie.
Obs. 1. 1. Dac (x
0
, y
0
) este o soluie particular atunci toate soluiile sunt date de :
x
1
= x
0
+a
2
t = x
0
+ bt
x
2
= y
0
a
1
t = y
0
- at

Probleme rezolvate

R7. 2. 1. Rezolvai n ZZZ ecuaia : 3x+4y+5z=6.
Soluie : 3x+4y=5k+1, are soluia de forma :

41
x = -1+3k
y = 1-k
deoarece : 3x+4y= -3+9k+4-4k=5k+1. Din aceasta avem : 5z=6-(5k+1)=5-5k z=1-k
Aceasta este o soluie particular. Avem ns:
x = -1+3k+4t
y = 1-k-3t i z = 1-k, unde k, t Z.

R7. 2. 2. Rezolvai n numere naturale ecuaia:
1/x+1/y+1/z=2/3.
Soluie: Din simetrie putem presupune : 2 x y z 1/x+1/y+1/z 3/x
3/x 2/3 2x 9 x 4 deci x { 2, 3, 4 }
Pentru x=4 1/y+1/z = 2/3 1/4 1/y+1/z=5/12 1/y+1/z 2/y 5/12 2/y deci
5y 24 y 4
Dac y=4 1/z=5/12 =2/12=1/6 z=6, i tripletul (4, 4, 6) este soluie.
Dac y=3 1/3+1/z = 5/12 1/z = 5/12-1/3=1/12 z=12 i tripletul (4, 3, 12) este
soluie.
Dac y=2 +1/z=5/12 1/z = -1/12 (fals).
Pentru x=3 1/y+1/z = 2/3-1/3=1/3 1/3 2/y y 6.
Dac y=6 z=6, tripletul (3, 6, 6) este soluie.
Dac y=5 1/z= 1/3-1/5=2/15 z Z.
Dac y=4 1/z= 1/3-1/4=1/12 z=12 i tripletul (3, 4, 12) este soluie
Dac y=3 1/z= 1/3-1/3=0 (imposibil) i de asemenea pentru y=2 nu avem soluie.
Pentru x=2 1/y+1/z=2/3-1/2 = 1/6, din care putem avea :
y=z=12 i tripletul (2, 12, 12) soluie
y=10, z=15 i tripletul (2, 10, 15) soluie
y=9, z=18 i tripletul (2, 9, 18) soluie
y=8, z=24 i tripletul (2, 8, 24) soluie
y=7, z=42 i tripletul (2, 7, 42) soluie.
Pentru y6 nu avem soluie. Avem n total 9 triplete n soluie.

R7. 2. 3. Determinai numerele prime p, q pentru care :
5p
2
- 2q
2
=6q-9p.

Soluie: Relaia dat conduce la : 5p
2
+9p = 6q + 2q
2
p(5p+9) = q(2q+6) p=q sau
q|5p+9. Pentru p=q avem : 3p
2
= -3p p= -1 (imposibil), rmne c : 5p+9=qn
pqn=q(2q+6) 2q+6 = np i q= (np-6)/2 5p+9 = (np-6)n/2 (n
2
-10)p = 6n+18
6n+18 n
2
-10 (p2) n (n-3) 19 n 6 i (6n+18)/(n
2
-10) N n=4 i p=7
de unde q=11, care este soluia.

42
8. Modulul unui numr real. Ecuaii cu module

Pentru definirea noiunii de modul a unui numr real (sau valoarea absolut a
unui numr real) vom da o exprimare algebric. Modulul folosindu-se adesea n
stabilirea distanei dintre dou puncte situate pe o dreapt, vom da o exprimare
geometric a noiunii de modul.


8.1. Definiia modulului unui numr real

Definiia 1. Modulul unui numr pozitiv este egal cu acel numr.
Definiia 2. Modulul unui numr negativ este egal cu opusul lui.
Definiia 3. Modulul lui zero este egal cu zero.

Putem scrie:

() xR,

<

=

<
=
>
=
0 x dac , x
0 x dac , x
x sau
0 x dac , x
0 x dac , 0
0 x dac , x
x

Dac notm cu E(x) o expresie algebric care conine pe x (i ne propunem s
o studiem n raport cu variabila x) atunci:

<

=
0 ) x ( E dac ), x ( E
0 ) x ( E dac ), x ( E
) x ( E

Considerm punctele M(a), punctul M de abscis a, situat pe axa numerelor reale:

O M(a)

0 a

Definiia 4. Distana, msurat pe axa numerelor reale, de la origine la punctul M(a) se
numete modulul numrului real a.

Deci: d(O,M) = OM = |a| , aR

Considerm punctele A(x
1
) i B(x
2
) pe axa numerelor reale:

A(x
1
) O B(x
2
)


43
Distana dintre aceste dou puncte se exprim astfel:

d(A,B) = AB =
2 1
x x


8.2. Proprietile modulului unui numr real

1. () xR, . 0 x
2. () x,y R, y x y x = .
Aceast proprietate o putem generaliza:
() x
1
, x
2
, ... , x
n
R,
n 2 1 n 2 1
x ... x x x ,..., x , x =
Consecine:
() xR, x x = ,
() xR,
2
2
x x = .
3. () x,yR, y0,
y
x
y
x
=
4. () x,yR, y x y x + +
Aceast proprietate o putem generaliza:
() x
1
,x
2
,...,x
n
R:
n 2 1 n 2 1
x ... x x x ... x x + + + + + +
5. Dac xR i a>0: a x = a x sau a x = =
6. Dac xR, a>0: a x < a x a < <
7. Dac xR, a>0: a x > a x sau a x > <

Probleme rezolvate

R8.1.1. S se calculeze:

a)
5049
5049 1263 2526 100 3 2
2
2 16 4 2 ... 2 2 2 +


b) ( ) { } ( )
5
599
12
111 1332 1998 1998 2997
2 : 5 5 3 3 2
[C.M.2001]

44
Soluie.

a) =
+
=
+
5049
5049 5052 5050 5052
5049
5049 5052 5052 5050
2
2 2 2 2
2
2 2 2 2


( )
1
2
1 2 2
5049
5049
=
+
=

b)
2997 1998 999 999 999 999 1998 1997
2 3 8 9 9 8 3 2 = = = ,

1332 1998 333 333 333 333 1332 1998
5 3 625 729 625 729 5 3 = = =
nlocuind n expresia dat, obinem:

( ) ( ) ( ) ( ) 4 2 : 2 2 : 5 5 3 2 3
2995 2997 2995 1332 1332 1998 1997 1998
= = +

R8.1.2. S se determine numerele reale x
1
, x
2
, ... , x
1997
astfel nct
x
1
+ x
2
+ ... + x
1997
=1997 i

1 1997 1997 1996 3 2 2 1
x x x x ... x x x x = = = =
[C.M 1997]

Soluie.
Notm , k x x
2 1
= atunci
x
1
x
2
= k, x
2
x
3
= k, ... , x
1996
x
1997
= k, x
1997
x
1
= k;
sau
x
1
x
2
= k, x
2
x
3
= k, ... , x
1996
x
1997
= k, x
1997
x
1
= k.
Adunm egalitile i obinem: 0 = k sau 0 = k, deci
1 x .. x x
x x ... x x dar
1997 x ... 2 x 1 x
0 x x ... x x x x
1997 2 1
1997 1997 2 1
1997 1996 3 2 2 1
= = = =
)
`

= + + +
= = =
= = = =


R8.1.3. Fie x,y,zR, x + y + z = 4. Artai c:
a) , 2 3 z 2 y 1 x + +
b) 1
2
2 y y
1 x
+
+
[C.M 2001]
Soluie.
a) Vom folosi inegalitatea R c , b , a ) ( , c b a c b a + + + +
45

2 3 z 2 y 1 x 2 6 4
6 z y x 3 z 2 y 1 x 3 z 2 y 1 x
+ + = =
= + + = + + + +

b) 1
2
2 y y
1 x
+
+ 2 2 y y 1 x 2 + + +
Demonstrm c aceast inegalitate este adevrat folosindu-ne de punctul a).
2 y y 1 x 2 + + + = + + + + + + = 1 y x 2 y 1 x y 1 x 2 y 1 x
= 3 z 2 y 1 x + + 2

R8.1.4. S se rezolve n mulimea numerelor reale ecuaiile:
a) ( ) , 0 2 x 1 x 2 2 x
2 2
= +
b) , 0 3 x 2
1 x
3 x 2
=
+


c) . 4 2 1 x 3 = +

Soluie.
a) Amintim egalitatea a a
2
= , ()aR
Atunci ( ) 1 x 2 2 x
2
( ) 2 x 2 x 2 x 2 2 x
2
2
= = + =
Ecuaia devine
( )( )
( ) { } 2 S , 2 x 0 2 x 0 2 x 1 2 x
0 2 x 2 x 2 x
0 2 x 2 x 2 x 0 2 x 2 x
2
= = = = + +
= + +
= + + = +

b)
2
3
x 0 3 x 2 0 1
1 x
1
3 x 2 0 3 x 2
1 x
3 x 2
= = =
|
|
.
|

\
|

+
=
+


Facem observaia c }. 1 { \ R x ) ( 0 1
1 x
1
<
+


c) 4 2 1 x 3 sau 4 2 1 x 3 4 2 1 x 3 = + + = + + = + +
Rezolvm ecuaia
3
4
x sau 0 x 2 2 x 3 sau 2 2 x 3 2 2 x 3 4 2 1 x 3 = = = + = + = + = + +
Rezolvm a doua ecuaie
46
4 2 1 x 3 = + + , 2 1 x 3 = + ecuaie imposibil
Soluia final este S =
)
`


3
4
, 0

R8.1.5. S se afle numerele x
1
, x
2
, ..., x
n
care verific egalitatea:
( ) 0 n | x | ... | x | | x | 2 x ... x x
n 2 1
2
n
2
2
2
1
= + + + + + + + , nN*.
Soluie.
= + + + + + + 0 1 | x | 2 x ... 1 | x | 2 x 1 | x | 2 x
n
2
n 2
2
2 1
2
1

( ) ( ) ( ) = + + + 0 1 | x | ... 1 | x | 1 | x |
2
n
2
2
2
1

= = = = 0 1 | x | ... 1 | x | 1 | x |
n 2 1

= = = = 1 | x | ... | x | | x |
n 2 1

} 1 , 1 { x ..., , x , x
n 2 1
.

R8.1.6. Determinai valorile naturale ale lui a pentru care ecuaia
3 | a x | a = + are dou rdcini ntregi. Rezolvai n acest caz ecuaia.
Soluie.
3 a | a x | sau 3 a | a x | 3 | a x | a = + = + = + .
Ecuaia 3 a | a x | = + este imposibil dac aN.
Atunci: a 3 | a x | 3 a | a x | = = + , are dou rdcini ntregi dac 3 a > 0
a<3 i aN a{0, 1, 2}.
Dac a = 0, }. 3 , 3 { x 3 | x | 3 | x | = =
Dac a = 1, 4 | 1 x | sau 2 | 1 x | 3 | 1 x | 1 = = = + ecuaie imposibil
} 3 , 1 { x 2 | 1 x | = .
Dac a = 2, 5 | 2 x | sau 1 | 2 x | 3 | 2 x | 2 = = = + ecuaie imposibil
} 3 , 1 { x 1 | 2 x | = .

R8.1.7. S se arate c ecuaia
|x a| + |x b| + |x + a| + |x + b| = |a + b| , a, bR
+

nu are rdcini reale.
Soluie.
Folosim inegalitatea: |a + b| |a| + |b| , ()a, bR
i egalitatea |a| = |a| , ()aR.
|x a| + |x + a| = |x a| + |x a| |x a x a| = |2a| = 2|a|
Analog |x b| + |x + b| 2|b|.
Adunnd inegalitile, obinem:
|x a| + |x + a| + |x b| + |x + b| 2|a| + 2|b| = 2(|a| + |b|) 2|a + b| > |a + b| ,
deci nu poate avea loc egalitate pentru orice xR.
47
9. Partea ntreag a unui numr real. Partea fracionar a unui numr real.
Ecuaii cu parte ntreag

9.1 Definiii i notaii

Considerm un numr real x care are scrierea zecimal x = x
0
,x
1
x
2
... unde
x
0
Z i x
1
, x
2
{0, 1, 2, ...,9}.
Partea ntreag a unui numr real x este cel mai mare numr ntreg mai mic sau
egal cu x. Notm: [x] partea ntreag a lui x i o definim astfel:

<

=
Z x sau 0 x dac 1 x
Z x sau 0 x dac x
] x [
0
0

Dac se ine seama de reprezentarea pe ax a numerelor reale, atunci partea
ntreag a unui numr real o putem considera primul numr ntreg din stnga lui x.
Partea ntreag a unui numr real mai poate fi reprezentat i astfel:
| |

<
+ <
=
N n , n x 1 n dac 1 n
1 n x n dac n
x
Exemple: [3,25] = 3; [3,91] = 3; [5,37] = 6; [5,95] = 6; [ 2 ] = 1; [] = 4;
Partea fracionar a unui numr real o notm prin {x} i o definim prin
egalitatea {x} = x [x].
Exemple: {2,35} = 0,35; {3,51} = 0,49
Observaie: Oricare ar fi xR, x = [x] + {x}, x[0,1).
Exemple: 2,57 = [2,57] + {2,57} = 2 + 0,57
3,72 = [3,72] + {3,72} = 4 + 0,28
Prezentm, n cele ce urmeaz, proprieti ale prii ntregi care se deduc direct
din definiie:
Proprietatea 1. () x[k, k+1), kZ avem egalitatea [x] = k
Proprietatea 2. Dac x,yR, i x,z[k, k+1), kZ atunci are loc egalitatea: [x]
= [y]
Proprietatea 3. Dac xR, x < 0, atunci [x] < 0
Dac xR, x 0, atunci [x] 0
Proprietatea 4. () xR exist egalitatea [[x]] = [x]
Proprietatea 5. () xR sunt adevrate inegalitile:
1) [x] x < [x+1] i 2) x 1 < [x] x
Folosind definiia prii ntregi i aceste proprieti vom afla partea ntreag a
unor expresii algebrice i vom rezolva ecuaii n care necunoscuta este exprimat prin
partea ntreag.

Probleme rezolvate
R9.2.1.Aflai partea ntreag a expresiei

5 4
1
4 3
1
3 2
1
2 1
1
A

=
48
Soluie.
( ) | |
( ) | | 51 , 13 23 , 2 73 , 1 41 , 1 2 5
) 3 2 ( 2 5 5 4 4 3 3 2 2 1 A
= + + +
+ + = + + + + + + + =

[A] = 14

R9.2.2.S se calculeze partea ntreag a numrului: ( )
2
1 n n a + + = , nN,
n 3.
L. Pran
Soluie.
Dup ridicarea la ptrat obinem ) 1 n ( n 2 1 n 2 a + + + = .
Folosim n continuare inegalitatea:

, 3 n 4 a 2 n 4 3 n 4 ) 1 n ( n 2 1 n 2 2 n 4
3 n ) ( , 2 n 2 ) 1 n ( n 2 1 n 2
+ < < + + < + + + < +
+ < + < +

deci [a] = 4n + 1, () nN, n 3

R9.2.3.S se rezolve ecuaia:
4
1 x 3
2
1 x
=
(

+
.
Soluie.
tiind c [a] este un numr ntreg () aR vom face substituia:
4
1 x 3
= k,
kZ

3
1 k 4
x 1 k 4 x 3 k 4 1 x 3 k
4
1 - 3x +
= + = = =
Ecuaia devine:
k
3
2 k 2
k
6
4 k 4
k
2
1
3
1 k 4
=
(

+
=
(

+
=
(
(
(
(

+
+
.
Folosind proprietatea 5 suntem condui la rezolvarea inecuaiei:

} 2 , 1 , 0 { k , 2 k 1 2 k 2 k 3 3 2 k 2
3
2 k 2
k 1
2
2 k 2
< + < +
+
<
+

Deci
)
`

3 ,
3
5
,
3
1
x .

49
R9.2.4S se rezolve ecuaia:

7
3 x 5
4
5 x 3
4
7 x 3 +
=
(


+
(


.
Soluie.
Notm k
7
3 x 5
=
+
, kZ
5
3 k 7
x 3 k 7 x 5

= =
Utilizm proprietatea 5 pentru cele dou pri ntregi ale ecuaiei.

4
7 x 3
4
7 x 3
1
4
7 x 3
<
(



4
5 x 3
4
5 x 3
1
4
5 x 3
<
(


Prin adunarea inegalitilor se obine:

4
12 x 6
4
5 x 3
4
7 x 3
2
4
12 x 6
<
(


+
(


i apoi folosind substituia avem:

.
5
32
, 5 x }, 5 , 4 { k deci ,
11
59
k
11
39
59 k 11 39 39 k 11 59 39 k 21 k 10 59 k 21
10
39 k 21
k
10
59 k 21
2
6
5
9 k 21
k 2
2
6
5
9 k 21
)
`

<
< <

<

<



R9.2.5.S se demonstreze c ecuaia:
6
2
2 x 3
2
1 x 3
=
(

+
+
(

+
nu are soluii ntregi.
[G.M. 2/1988]
Soluie.
Presupunem c x = aZ este o soluie a ecuaiei
I. a = 2k, kZ.
| |
Z
6
5
k 6 1 k 6 6 1 k 3 k 3
6 1 k 3
2
1
k 3 6
2
2 k 6
2
1 k 6
= = + = + +
= + +
(

+ =
(

+
+
(

+

II. a = 2k + 1, kZ
50
| |
. Z
3
1
k 2 k 6 6 2 k 3 2 k 3
6
2
5
k 3 2 k 3 6
2
5 k 6
2
4 k 6
= = = + + +
=
(

+ + + =
(

+
+
(

+

Rezult c ecuaia nu are nici o soluie ntreag.




Bibliografie

Gh. Andrei, I. Cucurezeanu, C. Caragea Probleme de algebr, gimnaziu, liceu,
Ed. GIL, Zalu, 1996.
Gh. Schneider Probleme de algebr volumul 2, Ed. Apollo, Craiova, 1990.
C. Nstsescu, C. Ni, M. Brandiburu, D. Joi, Ed. Rotech Pro, 1997.
L. Pran, C.G. Lazeanu Probleme de algebr i trigonometrie, Ed. Facla,
Timioara, 1983.
51
10. Inegalitile algebrice

Prezentm n cele ce urmeaz cteva proprieti elementare ale inegalitilor
precum i cteva inegaliti remarcabile. Acestea vor constitui baza raionamentului n
soluionarea problemelor care implic inegalitile.

10.1. Proprietile inegalitilor

10.1.1. () xR, xx (reflexivitatea)
10.1.2. Dac x,y,zR astfel nct xy i yz atunci xz (tranzitivitatea)
10.1.3. Dac x,y R astfel nct xy i yx atunci x=z (antisimetria)
10.1.4. () x,y,zR dac xy atunci x+z y+z.
10.1.5. () x,y,zR dac xy atunci 1. xzyz, pentru z0.
2. xzyz, pentru z0.
10.1.6. () x,y,R dac x>0 xy atunci
y
1
x
1
.
10.1.7. () x,y,a,bR dac xy atunci x+ay+a.
10.1.8. Dac x,y,a,bR astfel nct 0ab i 0xy , atunci axby.
10.1.9. () x,y,R dac xy atunci x
2n+1
y
2n+1
, () nN.
10.1.10. () x,y,R, dac 0xy atunci x
n
y
n
, () nN.

10.2 Inegaliti remarcabile

10.2.1 () xR, x
2
0
Generalizare () xR, x
2n
0, () nZ
Observaii referitoare la proprietile 1.9. i 1.10
1. dac -3<2 atunci (-3)
3
<2
3
-27<8 adevrat
2. dac -3<2 atunci (-3)
4
<2
4
81<16 fals

10.2.2. () x
1
,x
2
,...,x
n
R, x
1
2
,x
2
2
,...x
n
2
0
Se obine egalitatea pentru x
1
=x
2
=...=x
n
=0
10.2.3. () x,y,R, x
2
+z
2
2xy
10.2.4. () x,y,R, x>0, y>0 exist inegalitatea:
2
x
y
y
x
+
10.2.5. () x,y,R; x,y>0
y
1
x
1
2
xy
2
y x
+

+
( inegalitatea mediilor)
Observaie. Se poate da o generalizare pentru inegalitatea ce exist ntre media
aritmetic i media armonic a numerelor reale pozitive:
52
() x
1
,x
2
,...,x
n
R
+
:
n 2 1
n 2 1
x
1
...
x
1
x
1
n
n
x ... x x
+ + +

+ + +

10.2.6. () x,y,zR: x
2
+ y
2
+ z
2
xy + yz + zx


Probleme rezolvate

R10.3.1. S se arate c oricare ar fi x,y,z R exist inegalitatea x
2
+ y
2
+ z
2
+
6x 4y + 2z + 140
Soluie.
Cutm s formm o sum de ptrat folosind formulele de calcul prescurtat:
x
2
+ 6x + 9 + y
2
+ 4y + 4 + z
2
+ 2z + 1 0
(x+3)
2
+ (y-2)
2
+ (z+1)
2
0, inegalitate adevrat.

R10.3.2. S se arate c oricare ar fi numerele reale strict pozitive are loc
relaia: a
1
2
+ a
2
2
+ ... + a
n
2
2(a
1
+a
2
2 +...+a
n
0
2
) 1 n ( n
) n
+
+
Soluie.
Observm c ultimul termen al expresiei reprezint suma primelor n numere naturale:
1 + 2 + ... + n
2
) 1 n ( n +
=
Aplicnd proprietile adunrii i nmulirii formm ptrate perfecte cu intenia
de a ajunge la o inegalitate adevrat
a
1
2
-2a
1
+1+a
2
2
-2a
2
2 +2+...+a
n
2
-2a
n
n +n = (a
1
-1)
2
+(a
2
- 2 )
2
+...+(a
n
- n )
2
0
Egalitate se obine pentru a
1
=1, a
2
= 2 ,..., a
n
= n


R10.3.3. S se arate c dac a,b i c sunt numere strict pozitive, atunci

c b a
b
ca
a
bc
c
ab
+ + + +

Soluie.
Dm dou metode pentru justificarea acestei inegaliti. Prima se va referi la reducerea
inegalitii date la o inegalitate cunoscut ( remarcabil), iar n a doua metod vom
justifica inegalitatea dat pornind de la o inegalitate cunoscut.

Metoda I
+ + + + c b a
b
ca
a
bc
c
ab
a
2
b
2
+b
2
c
2
+c
2
a
2
a
2
bc+ab
2
c+abc
2

2a
2
b
2
+2b
2
c
2
+2c
2
a
2
2a
2
bc-2ab
2
c+2abc
2
0 a
2
(b-c)
2
+b
2
(a-c)
2
+c
2
(a+b)
2
0,
53
inegalitate adevrat.

Metoda a II-a
Numerele reale
a
bc
i
b
ac
sunt strict pozitive i verific inegalitatea:
c 2
b
ac
a
bc
b
ac
a
bc
2
b
ac
a
bc
+ +
Analog se obin inegalitile b 2
c
ab
a
bc
+ i a 2
c
ab
b
ac
+ .
Adunnd ultimele trei inegaliti se obine inegalitatea cerut:
( ) ( ) c b a 2
b
ca
a
bc
c
ab
c b a 2
b
ca
a
bc
c
ab
2 + + + + + +

+ +

R10.3.4. S se arate c oricare ar fi numerele strict pozitive a,b,c exist
inegalitatea:
c b
a
+
+
a c
b
+
+ 2
b a
c
>
+


Soluie.
Pentru demonstrarea inegalitii vom folosi inegalitatea mediilor pentru
a c
b
+
i 1,
deci
1
a c
b

+ a c
b
2
1
a c
b
+

+
+

a 2
c b a + +

Analog:
b
c a +

b 2
c b a + +
i
c
b a +

c 2
c b a + +
.
Folosind proprietatea 1.6. , obinem inegalitile:
c b a
a 2
c b
a
+ +

+
,
c b a
b 2
c a
b
+ +

+
,
c b a
c 2
b a
c
+ +

+
, iar prin nsumare se
obine inegalitatea cerut
c b
a
+
+
c a
b
+
+ 2
b a
c
>
+

Inegalitatea este strict pentru c a+b+c>0 i egalitatea b+c=a, a+c=b i a+b=c nu pot
avea loc simultan.
Cu rezultat bun putem folosi n demonstrarea acestei inegaliti i inegalitatea
dintre mediile geometrice i i media Armonic a dou numere strict pozitive.
54
c b a
a 2
c b
a
1
c b
a
c b
a
2
c b
a
+ +

+
+
+

+

Analog:
c a
b
+

c b a
b 2
+ +
,
b a
c
+

c b a
c 2
+ +



R10.3.5. S se arate c oricare ar fi x, y,z, numere reale pozitive, exist
inegalitatea
( ) ( ) ( ) ( ) z y x
2
3
y x z y x y z y x + + + + + + +
Artai c se poate gsi n acest caz o inegalitate mai bun (fin), adic
( ) ( ) ( ) ( ) z y x 2 y x z y x y z y x + + + + + + +
Soluie.
Folosim inegalitatea dintre media geometric i cea aritmetic.
( ) z y x +
2
z y x + +
; ( ) y x y +
2
z y x + +
; ( ) y x z +
2
z y x + +

Adunnd inegalitile, obinem inegalitatea cerut:
( ) ( ) ( ) ( ) z y x
2
3
y x z y x y z y x + + + + + + +
Facem un raionament asemntor pentru a obine o mbuntire a inegalitilor date
( ) ( ) ( )
( )
( )
2 2 2
2 2 2
2 2 2
4
2
2
x y z y x z z x y
x y z y x y z x y
x y z
x y z
+ + + + + +
+ + + + + + + =
+ +
= = + +
deci
( ) ( ) ( ) ( ) z y x 2 y x z y x y z y x + + + + + + +

R10.3.6. S se arate c:
a
4
+b
4
+c
4
abc(a+b+c) [M.A.D.]
Soluie.
a
4
+b
4
+c
4
a
2
b
2
+b
2
c
2
+c
2
a
2
= (ab)
2
+(bc)
2
+(ac)
2
= + + ca ab ca bc bc ab abc(a+b+c)


Probleme propuse

P10.4.1. S se arate c () x,y,zR, are loc inegalitatea
x
2
+19y
2
+6z
2
-8xy-4xz+12yz>0

55
P10.4.2. S se arate c oricare ar fi numerele reale strict pozitive a,b,c exist
inegalitatea:
8
bc
a
c
ab
c
b
ac
b
a

+

P10.4.3. Demonstrai c ()tR i nN, are loc inegalitatea:
(1+t)
n
+(1+t
-1
)2
n+1

P10.4.4. Se dau numerele reale pozitive a,b,c, astfel nct a+b+c=1. Artai c
a) 9
c
1
b
1
a
1
+ + ;
b) ( ) ( ) ( )
2
3
b a c c a b c b a + + + + + . Generalizare
c) 5 1 c 4 1 b 4 1 a 4 < + + + + + . Generalizare

P10.4.5. S se arate c dac a
1
,a
2
,...,a
n
sunt numere reale pozitive cu
proprietatea a
1
a
2
...a
n
=1, atunci au loc inegalitile:

a) ( )( ) ( )
n
n 2 1
2 a 1 ... a 1 a 1 + + + .
b) ( )
2
1 1
a a 1 + + ( ) ( )
n 2
n n
2
2 2
3 a a 1 ... a a 1 + + + + .

P10.4.6. S se arate c dac nN* i a>0, atunci
a) ;
2
n
n a
1 n a
...
2 a
1 a
1 a
a

+
+
+ +
+
+
+
+


b) n 2
1 n a
n a
...
1 a
2 a
a
1 a

+
+
+ +
+
+
+
+




Bibliografie

1. Panaitopol L; Bnolil V, Lascu M. Inegaliti Ed. GIL Zalu 1996.
2. Foaie matematice nr. 6/1996.
3. Revista matematic a elevilor din Timioara nr. 1/1997
56
11. Probleme de ordonare

Dac notm cu ) , max( b a cel mai mare dintre numerele a i b, iar prin
) , min( b a cel mai mic dintre numerele a i b atunci avem:

<
=
<
=
b a b
b a b a
b a a
b a
daca
daca sau
daca
) , min(
iar

<
=
>
=
b a b
b a b a
b a a
b a
daca
daca sau
daca
) , max(
sau
2
| |
) , min(
b a b a
b a
+
= , iar
2
| |
) , max(
b a b a
b a
+ +
= .
Problemele de ordonare mbrac forme variate. Unele situaii necesit metode
ingenioase pentru rezolvare. Exist cazuri cnd operaia de ordonare ajut la dovedirea
egalitii a dou numere a i b, prin stabilirea simultan a inegalitilor b a i b a .
Numerele iraionale ridic n unele situaii probleme dificile de ordonare. De
aceea se impun metode adaptate fiecrui caz n parte.

Probleme rezolvate

VII_algR11.1. Comparai numerele:
3
2 i
2
3 .
Soluie. Fie
3
2 = x i
2
3 = y , atunci avem:
3
3 2
2
3
2
2 2 2 = =
|
.
|

\
|
= x , iar
2
2 2
2
2
2
3 3 3 = =
|
.
|

\
|
= y
Atunci
( ) 8 2 2 2 ) (
3 6
2
3 2 2
= < = = x i ( ) 9 3 3 ) (
2
2
2 2 2
= = = y .
Deci
2 3
3 2 < .
R11.2. Demonstrai c numerele reale a,b,c,d care verific relaiile de mai jos
sunt egale
0 2 3 + c b a
0 2 3 + d c b
0 2 3 + a d c
0 2 3 + c a d
Soluie. Adunnd membru cu membru relaiile date obinem 00. rezult c
toate relaiile devin egaliti (nici una din inegaliti nu este strict). Deci obinem:
0 2 3 , 0 2 3 , 0 2 3 , 0 2 3 = + = + = + = + c a d a d c d c b c b a ,
57
care se mai scriu:
c a a d a d d c d c c b c b b a 2 2 , 2 2 , 2 2 , 2 2 = = = =
sau
) ( 2 ), ( 2 ), ( 2 ), ( 2 c a a d a d d c d c c b c b b a = = = = .
Atunci
) ( 2 ) ( 2 ) ( 2 ) ( 2
4 3 2
b a a d d c c b b a = = = =
de unde ) ( 2
4
b a b a = rezult 0 ) 1 2 )( (
4
= b a . Atunci obinem b a = . Analog
d c c b = = , , deci d c b a = = = .
R11.3. S se decid dac numrul
2003 2004
2004 2003
2004 2003
2004 2003
+
+
este supraunitar,
echiunitar sau subunitar.
Soluie. Scoatem factor comun pe 2004
2003
:
1 2003
2004
2003
1 2003
2004
2003
1 2003
2004
2003
2004
2004
2004
2003
2004
2003
2003
2003
2003
2003
2003
+
|
.
|

\
|
+ +
|
.
|

\
|
=
(
(

+
|
.
|

\
|
(
(

+
|
.
|

\
|

1
2004
2003
< , rezult c 1
2004
2003
2003
<
|
.
|

\
|
(orice putere natural a unui numr subunitar
este un numr subunitar. Fiindc 1
2004
2003
2003
<
|
.
|

\
|
rezult c
2003 2003
2004
2003
2003
<
|
.
|

\
|
. (Produsul dintre un numr subunitar pozitiv a i un numr
pozitiv beste mai mic ca b). Atunci
2004
2004
2003
1 2003 1 2003
2004
2003
2003 2003
+
|
.
|

\
|
< + < +
|
.
|

\
|

i obinem c
2004
2004
2003
1 2003
2004
2003
2003 2003
+
|
.
|

\
|
< +
|
.
|

\
|

Deci numitorul este mai mic dect numrtorul i atunci fracia este
supraunitar.
R11.4. Scriei n ordine cresctoare numerele:
3 2
3 2
1
,
1
,
1
, , ,
a a a
a a a , unde
} 1 , 0 , 1 { a .
Soluie. 1) Dac 1<a<0 avem:
58
2
2 3
3
1 1 1
a
a a a
a a
< < < < <
2) Dac 0<a<1 avem:
3 2
2 3
1 1 1
a a a
a a a < < < < <
3) Dac a>1 avem:
3 2
2 3
1 1 1
a a a
a a a
< < < < < .
R11.5. Fie
n n
S
2
1
1 2
1
...
4
1
3
1
2
1
1
1

+ + + =
n n n n
S
2
1
...
3
1
2
1
1
1
2
+ +
+
+
+
+
+
=
) 1 (
1
...
3 2
1
2 1
1
3
+
+ +

=
n n
S
|
.
|

\
|
+ + +
+ +
+ +
+
+
+ =
n
S
... 2 1
1
...
3 2 1
1
2 1
1
1
2
1
4

Scriei n ordine cresctoare expresiile
4 3 2 1
, , , S S S S .
Soluie. =
|
.
|

\
|
+ + + +

+ + + + =
n n n
S
2
1
...
4
1
2
1
2
2
1
1 2
1
...
3
1
2
1
1
1

=
|
.
|

\
|
+ + + +

+ + + + =
n n n
1
...
2
1
1
2
1
1 2
1
...
3
1
2
1
1
n n n 2
1
...
2
1
1
1
+ +
+
+
+
=
Deci
2 1
S S = .
1 1
1
1
1
1
1 1
...
3
1
2
1
2
1
1
1
3
+
=
+
=
|
.
|

\
|
+
+ +
|
.
|

\
|
+
|
.
|

\
|
=
n
n
n n n
S
Atunci
1
1
...
1
1
1
1
2
1
...
2
1
1
1
+
+ +
+
+
+
< + +
+
+
+ n n n n n n

Deci
3 2
S S < .
Fiindc
2
) 1 (
... 2 1
+
= + + +
n n
n
) 1 (
1
...
3 2
1
2 1
1
) 1 (
2
...
4 3
2
3 2
2
2 1
2
2
1
4
+
+ +

=
|
|
.
|

\
|
+
+ +

=
n n n n
S
Deci
4 3 2 1
S S S S = < = .


59
12. Rezolvarea ecuaiilor i inecuaiilor cu ajutorul inegalitii mediilor

12.1.Consideratii teoretice si exemple

Definiie 12.1.1.Numim ecuaie o propoziie cu una sau mai multe necunoscute,i care
este adevrat pentru anumite valori ale variabilelor,propoziia coninnd semnul de
egalitate.
Observaie 12.1.2.Dac propoziia este adevrat pentru toate valorile variabilelor
(necunoscutelor)spunem c avem o identitate.
Exemple : 1.a.3x+4y=5 x,y N este o ecuaie
1.b.3x+4y=3(x-4)+4(y+3);x,y R este o identitate
Mulimea valorilor variabilelor pentru care propoziia de acest fel este adevrat,se
numete soluia ecuaiei i se noteaz de obicei cu S.
n exemplele precedente avem S
a
=;S
b
=R.
Observaie 12.1.3.Mulimea din care este soluia trebuie precizat,altfel enunul este
incomplet.
Ecuaia : 3x+4y=5 are n ZZ o infinitate de soluii,ele sunt date de :
x
n
=4n+3;y
n
= -3n - 2 deoarece 3x
n
+4y
n
=12n+9-12n-4=5.
Observaie 12.1.4.Soluia unei ecuaii poate fi format dintr-un numr sau o
pereche,sau un triplet,dup numrul de necunoscute.Sunt probleme care conduc la
ecuaii,sau sisteme de ecuaii.
Definiie 12.1.5.Numim o inecuaie ,o propoziie care are una sau mai multe
necunoscute,i care este adevrat pentru anumite valori ale variabilelor ,propoziia
coninnd unul din semnele : < ; ; > ;
Exemplu 2.a. 3x+2y5 (x,y) NN

2.b. 3x+4y9 (x,y) RR
Observaie 12.1.6.Mulimea valorilor variabilelor pentru care o inegalitate este
adevrat se numete soluie a inecuaiei.i la inecuaie soluia conine perechi
,triplete,sau mulimi formate din numere,dup cum ea conine mai multe variabile sau
una.
3x+2y5 (x,y) NN ; S
a
={ (0,0);(0,1);(0,2);(1,0);(1,1) } iar
3x+4y9 (x,y) RR ; S
b
={ (3-4y/3+a,z);yR;a0}
Cele dou mulimi sunt una finit i una infinit.
Dac a,b R
+
; atunci urmtoarele numere ,cunoscute sub numele de medii:
M
aritmetic
= M.A.= (a+b)/2 = m
a

M
geometric
= M.G. = ab = m
g
M
armonic
= M.H. = 2/(1/a+1/b) = m
h

M
ptratic
= M.P. =
2
2 2
b a +
=m
p
; verific inegalitile din :

Teorema 12.1.7 a) m
h
m
g
m
a

b) m
h
m
g
m
a
m
p

60
Demonstraie:
m
h
m
g
2ab (a+b) ab 2 ab a+b ( a - b )
2
0 (cu egalitate
a=b).
m
g
m
a
ab (a+b)/2 ( a - b )
2
0 (evident).
m
a
m
p
(a+b)/2
2
2 2
b a +
2(a
2
+b
2
) (a+b)
2
(a-b)
2
0.
Dovedirea acestor inegaliti este mult mai interesant dac se poate face pe cale
geometric.Dm n continuare cteva astfel de demonstraii:

(a) Considerm semicercul de diametru MN.
Ip: MP=a;
PN=b;
MOON;
C: LP LO.












LO=(a+b)/2; LP
2
=ab LP= ab ab (a+b)/2.

(b) Considerm dou cercuri tangente de raze b i a.(exterior) i coarda lor comun
AB.








Fie O
2
P O
1
A
O
2
P=
2 2
) ( ) ( a b a b + = ab 4 =2 ab O
1
O
2
= a+b ab (a+b)/2
Este evident c O
2
PO
1
O
2
,adic AB||O
1
O
2
,adic O
1
ABO
2
-dreptunghi deci
L
M
a
P O
b
N
O1
P
A
b
a
O2
B

61
O
1
AO
2
B a=b.

(c) Considerm un trapez dreptunghic i un semicerc nscris n el tangent laturii
neparalele,diametrul fiind nlimea trapezului,vom obine o nou interpretare
geometric pentru m
h
m
g
m
a.













ON-linie mijlocie ; BC=b ; AD=a ; CR = 2 ab ; 2LO = AB.
Din unghiurile cu laturi perpendiculare KLO i RCD avem :
RCD~ KLO (U.U) CD/LO=RD/KO=CR/LK
KL=(LDCR)/CD=2/(1/a+1/b).
Avem inegalitile evidente :
LK LO ON m
h
m
g
m
a
.

Probleme rezolvate

R12.2.1. S se determine numerele reale x[1,), y (0,1] pentru care avem :
( x +1/ x )( y +1/ y ) 2( y x / + x y / ).
Soluie: Inegalitatea este echivalent cu :
[(y+1)/ y ] [(x+1)/ x ] 2[(x+y)/ xy ] (x+1)(y+1) 2(x+y)
xy+1 x+y x(y-1)+1-y 0 (y-1)(x-1) 0.Din condiii ns y-1 0 i
x-1 0,ca atare obinem soluia : { y=1;x [1,) } { x=1;y(0,1] }.
n rezolvarea acestei inecuaii nu este neaparat s folosim inegaliti cunoscute,deci
nu form utilizarea lor.
R12.2.2. Determinai x,y R tiind c :
1/(x
2
4x + 5) + 1/(y
2
+4y +5) + x
2
+ y
2
= 4(x-y) 6.
Soluie: Egalitatea dat este echivalent cu :
1/( x
2
4x + 5) + (x
2
4x + 5) + 1/(y
2
+4y +5) + (y
2
+4y +5) = 4
Din x
2
4x + 5 = (x-2)
2
+ 1 > 0 1/( x
2
4x + 5) + (x
2
4x + 5) 2 i analog
1/(y
2
+4y +5) + (y
2
+4y +5) 2 (inegalitatea m
a
m
g
).Egalitatea are loc dac i
numai dac (x
2
4x + 5)
2
=1, (y
2
+4y +5)
2
=1,de unde [x
2
4x + 5 { -1,1 }; x
2

C
B
L
N
O K
R
A
D

62
4x + 5>0] x
2
4x + 5=1 i analog y
2
+4y +5=1 x=2,y= -2 care este i soluia
ecuaiei.
R12.2.3. Determinai numerele reale a
1
,a
2
,,a
n
(0,) ; n N ; n2 pentru care
avem : (a
1
a
2
)/(a
1
+a
2
) + (a
2
a
3
)/(a
2
+a
3
) + + (a
n
a
1
)/(a
n
+a
1
) n/3 (a
1
3
+a
2
3
++
a
n
3
)/6
Soluie : Din m
h
m
a
, (xy)/(x+y) (x+y)/4 (a
1
a
2
)/(a
1
+a
2
) (a
1
+a
2
)/4 i
analoagele,care adunate dau:
(a
1
a
2
)/(a
1
+a
2
) + (a
2
a
3
)/(a
2
+a
3
) + + (a
n
a
1
)/(a
n
+a
1
) ( a
1
+a
2
++a
n
),dar pe de alt
parte avem: ( a
1
+a
2
++a
n
) n/3 (a
1
3
+a
2
3
++ a
n
3
)/6
(a
1
-1)
2
(a
1
+2) + (a
2
-1)
2
(a
2
+2) + + (a
n
-1)
2
(a
n
+2) 0(evident).Numerele cerute
sunt deci cele pentru care avem egalitatea,adic a
1
=a
2
==a
n
=1.
R12.2.4. Demonstrai c exist x,y R astfel nct : (x
2
+x+1)(3y
2
+2y+3)=2.
Soluie: x
2
+x+1 x
2
+x+1/40 (x+1/2)
2
0.
3y
2
+2y+3 8/3,deoarece 3y
2
+2y+3=3(y+1/3)
2
+8/3 deci lund x=-1/2 i y=
-1/3 avem egalitate.

63
13. Probleme de maxim i minim

Importana practic a problemelor de maxim i minim este de necontestat. La
gimnaziu se studiaz foarte puin asemenea probleme, deoarece ele solicit procedeee
mai deosebite pentru rezolvare.
Problemele de maxim i minim nu sunt uor de abordat fr a stpni anumite
tehnici de rezolvare. Iat cteva dintre acestea:
a) Folosim dou teoreme care ne ajut la determinarea maximului i
minimului.
Teorema 13.1. Dac suma mai multor variabile pozitive este constant,
maximul produsului are loc cnd variabilele sunt egale.
Deci dac
n
x x x P ...
2 1
= cu = + + +
n
x x x ...
2 1
constant, P este maxim cnd
n
x x x = = = ...
2 1
. Dac
n
n
x x x P

= ...
2 1
2 1
cu
i
N = ) , 1 ( n i maximul lui P are loc
cnd
n
n
x x x

= =

...
2
2
1
1
.
Pentru dou variabile
1
x i
2
x ,
2 1
x x P = cu = +
2 1
x x constant, P este maxim
cnd
2 1
x x = deoarece
2
2 2 1
2
1
2
2 1
2
2 2 1
2
1
2
2 1
2 ) ( , 2 ) ( x x x x x x x x x x x x + = + + = +

atunci ] ) ( ) [(
4
1
2
2 1
2
2 1 2 1
x x x x x x + = , deci
2 1
x x este maxim cnd 0 ) (
2
2 1
= x x
adic
2 1
x x = .
Teorema 13.2. Dac produsul mai multor variabile pozitive este constant,
atunci suma lor este minim cnd variabilele sunt egale.
Pentru dou variabile
1
x i
2
x pozitive cu
2 1
x x P = (constant)
2 1
x x S + =
este minim dac i numai dac
2 1
x x = , deoarece
2 1
2
2 1
2
2 1
4 ) ( ) ( x x x x x x + = +
cu
2 1
x x constant i atunci cea mai mic valoare a sumei se obine cnd
0 ) (
2
2 1
= x x , deci
2 1
x x = .
b) Metoda reflexiei
Metoda const n completarea figurii iniiale asociat problemei (sau a unei
pri din ea) cu simetrica ei n raport cu o dreapt (adic cu imaginea "reflectat" ntr-o
oglind).
c) Metoda liniei frnte
Dac problema ne cere s determinm minimul sumei lungimilor unor
segmente atunci folosind aceast metod trebuie s efectum o construcie care s
conduc la o linie frnt format cu segmente dintre segmentele date i cu segmente
congruente cu cele care intervin n suma al crei minim se cere. Aceast linie frnt va
avea ca extremiti puncte fixe A i B. Deoarece drumul cel mai scurt ntre dou puncte
64
este segmentul de dreapt determinat de ele este posibil ca AB s fie realmente
minimul cutat.
d) Metoda curbei de nivel
Se folosete pentru problemele n care minimul sau maximul respectiv este
condiionat de poziia unui punct M, poziie ce trebuie precizat pe o dreapt sau pe un
cerc. Se presupune c expresia din concluzia problemei are valoarea constant k i se
determin curba loc geometric C
k
, a punctului M n acest caz. Pentru a determina
poziia cerut a punctului M se alege din familia de curbe de nivel C
k
acea curb care
este tangent dreptei sau cercului pe care am cutat punctul M, poziia care realizeaz
extremul va fi chiar punctul de tangen.


Probleme rezolvate

R13.1. S se afle maximul expresiilor:
3
3
2 2
2
2 2 4
1
2
) ( ), ( ), 4 ( ) (
x
a x
x E x a x E x a x x E

= = =
unde
+
R a i
*
+
R x .
Soluie. ) (
1
x E se mai scrie: ) 4 ( ) ( ) (
2 2 2 2
1
x a x x E = . Cei doi factori au
suma constant:
2 2 2 2
4 4 a x a x = + (const.) Maximul are loc cnd
1
4
2
2 2 2
x a x
= ,
de unde obinem:
2 2
8 3 a x = , deci
3
6 2a
x = i atunci produsul maxim este
27
256
3
4
9
64
3
8
4
3
8
6 2 4 2
2
2
2
a a a a
a
a
= =

. Pentru expresia
2
E , maximul
produsului ) (
2 2
x a x are loc n acelai timp cu al ptratului su
2 2 2 2
) ( x a x . Suma
factorilor
2
x i
2 2
x a este
2
a (constant), maximul are loc cnd
2 1
2 2 2
x a x
= , de
unde
3
3
3
3
3
2
2 2 2
a a
x
a
x a x = = = =
9
3 2
3 3
3
3 2
2
max 2
a a
a
a
E =

=
Expresia ) (
3
x E se mai poate scrie:


=
x
a
x x
a
x
x
x x
a x
x
x E
2
1
1 2 1 2 1
) (
2 2 2 3

65
Condiia de maxim nu se schimb dac multiplicm expresia cu
2
4a .
Obinem:

=
x
a
x
a
x
a
x
a x E a
2
1
2 2
1
1
4 ) ( 4
2
2
2
3
2

Suma factorilor este 1
2
1
2
= +
x
a
x
a
(constant). Atunci maximul are loc cnd
1
2
1
2
2
x
a
x
a

= , de unde obinem 2
4 2
= +
x
a
x
a
, deci 2
6
=
x
a
, de unde
2
6a
x = . Deci
a x 3 = .
2 3 3
max
27
1
27 27
2 3
) (
a a
a
a
a a
x E = =

=
R13.2. S se afle minimul expresiei
2
3
1
) (
) (
b x
x
x E

= ,
*
+
R x , iar 0 > b .
Soluie. Minimul expresiei corespunde maximului expresiei inverse:
3
2
) (
x
b x
,
care se scrie:
2 2
2
2
2
1
1 1 ) ( 1

=
x
b
x x
b x
x x
b x
x
E .
nmulind cu b obinem
2
2
1

=
x
b
x
b
E b . Cei doi factori au suma constant
1 1 = +
x
b
x
b
. Deci
2
1
1
x
b
x
b

= , de unde b x 3 = . Deci
4
27
4
27
) 3 (
) 3 (
2
3
2
3
min
b
b
b
b b
b
E = =

= .
R13.3. S se afle minimul expresiei
2
4 4
x
x a
E
+
= unde 0 x , a i x sunt
numere reale pozitive.
Soluie.
2
2
4
x
x
a
E + = . Produsul termenilor este
4 2
2
4
a x
x
a
= (constant).
Minimul are loc cnd
2
2
4
x
x
a
= , de unde a x = . Minimul expresiei este
2
2
4 4
min
2a
a
a a
E =
+
= (a>0).
66
R13.3. Se consider o dreapt d i dou puncte A i B situate de aceeai parte
a ei. S se determine poziia unui punct M de pe dreapta d astfel nct suma MA+MB
s fie minim.
Demonstraie. Fie T un punct oarecare pe dreapta d. Construim simetricul B' al
punctului B fa de dreapta d.
Fie {E}=dBB'. Din congruena triunghiurilor dreptunghice TBE i TB'E
obinem c (TB)(TB'). Atunci AT+TB=AT+TB'AB' (inegalitatea triunghiului n
ATB'). Fiindc A i B' sunt fixe, poziia lui M astfel ca suma s fie minim se obine la
intersecia dreptei d cu AB'. (Problema se mai numete teorema reflexiei sau teorema
biliardului).
R13.4. n interiorul unghiului AOB se consider punctele C i D. S se
gseasc pe latura OA a unghiului dat un punct F i pe latura OB un punct E astfel ca
linia frnt DEFC s fie cea mai scurt posibil.
Demonstraie. Construim simetricul C' al punctului C fa de OA i D'
simetricul lui D fa de OB.
n loc de punctele C i D acum punctele C' i D' ndeplinesc condiiile cerute
de problem. Deci trebuie s gsim drumul cel mai scurt posibil ntre C' i D' care s
A
T
M
B'
E d
B
O
L
C'
K
A
F
C
D
T
E P
B
D'
67
ntlneasc laturile OA i OB ale unghiului AOB. Drumul cel mai scurt ntre dou
puncte este segmentul de dreapt ce le unete. Segmentul [C'D'] intersecteaz laturile
unghiului AOB n F respectiv E. Acestea sunt punctele cutate.
Din triunghiurile dreptunghice congruente KC'F i KCF obinem c
(C'F)(CF) (1)
Din triunghiurile dreptunghice congruente EDP i ED'P obinem c
(ED)(ED') (2)
Atunci CF+FE+ED=C'F+FE+ED'=C'D'. Linia frnt CEFD este cea mai
scurt. Oricare alte dou puncte luate pe laturile OB i OA, n afar de E i F gsite mai
sus ne vor da o linie frnt mai lung dect CEFD.
Fie L i T dou puncte oarecare situate respectiv pe laturile OA respectiv OB,
vrem s artm c CF+EF+FD<CL+LT+TD. Datorit simetriei putem scrie:
CF+EF+ED=C'F+EF+ED'=C'D' i CL+LT+TD=C'L+LT+TD'
n patrulaterul C'D'TL, C'D' este mai scurt dect linia frnt C'LTD'.
R13.5. Se consider triunghiul ascuitunghic ABC. S se determine triunghiul
DEF cu D(BC), E(CA), F(AB) care are perimetrul minim
Demonstraie. Fie Q simetricul punctului D fa de AC i T simetricul lui D
fa de AB. Atunci triunghiurile dreptunghice DEP i EPQ sunt congruente (P fiind la
intersecia dreptelor DQ i AC).
Din congruena acestor dou triunghiuri obinem c
(DE)(EQ) (1)
i triunghiurile dreptunghice DFR i TRF sunt congruente (R este intersecia dreptelor
AB i DT). Din congruena acestor dou triunghiuri obinem c
(FD)(FT) (2)
Folosind relaiile (1) i (2) obinem: Perimetrul triunghiului DEF este:
EF+DF+ED=EF+TF+EQ (3)
Din congruena triunghiurilor dreptunghice ART i ARD obinem c
m(RAD)=m(RAT) (4)
Din congruena triunghiurilor dreptunghice APD i APQ obinem c
m(DAP)=m(PAQ) (5)
Din relaiile (4) i (5) obinem c m(TAQ)=2m(BAC), deci m(TAQ) este
constant.
Perimetrul triunghiului DEF nu poate fi mai mic dect (TQ) iar acesta este
minim cnd AT=AQ=AD, este minim deci cnd AD este minim, adic D este piciorul
perpendicularei din D pe BC. Punctele E i F se obin ca intersecii ale dreptei TQ cu
A
F
T
R
E
Q
P
B D
C
68
AC respectiv AB. Am obinut astfel c triunghiul de perimetru minim DEF este
triunghiul ortic al triunghiului ABC.
R13.6. Se consider un triunghi ABC i M(BC). Fie E i F proieciile
ortogonale ale lui M pe AB respectiv AC. S se determine poziia lui M pe (BC) astfel
nct aria triunghiului MEF s fie maxim.
Demonstraie. Patrulaterul AEMF este inscriptibil avnd unghiurile opuse din
E i F suplementare.
Rezult c m(BAC)+m(EMF)=180.
Deci m(EMF)=180-m(BAC). Atunci
2
EMF) sin( MF ME
S
[MEF]

= (1)
Dar BAC) sin( BAC)] sin( sin[180 EMF) sin( = = . Deci
2
BAC) sin( MF ME
S
[MEF]

= (2)
Dar
[ACM] [ABM] [ABC]
S S S + = sau
2
MF AC
2
ME AB
S
[ABC]

+

= , de unde
[ABC]
S 2 MF AC ME AB = + sau
[ABC]
2S MF b ME c = + (3)
Din (2) avem c maximul ariei triunghiului MEF se atinge cnd MEMF este
maxim. Dar maximul produsului MEMF se atinge odat cu maximul lui
(cME)(bMF) adic dac cME=bMF, ceea ce implic egalitatea ariilor triunghiurilor
ABM i AMC, deci M este mijlocul lui (BC).
R13.7. Dintre toate dreptunghiurile de aceeai arie s se determine cel de
perimetru minim.
Soluie. Perimetrul drepunghiului este ) ( 2 y x + , iar aria y x S = (unde x i y
sunt dimensiunile dreptunghiului). Fiindc = y x constant, suma lor este minim cnd
y x = . Deci perimetrul dreptunghiului este x y x 4 ) ( 2 = + .
Dintre toate dreptunghiurile de aceeai arie, ptratul are perimetru minim.
R13.8. Dintre toate dreptunghiurile cu acelai perimetru care este cel de arie
maxim?
A
E
B M C
F
69
Soluie. Dac x i y sunt dimensiunile dreptunghiului, atunci perimetrul este
P y x = + ) ( 2 , rezult
2
P
y x = + , y x S = . Suma factorilor x i y este constant i
atunci produsul este maxim cnd y x = (ptrat). Deci
2
max
x S = .
Dintre toate dreptunghiurile cu acelai perimetru ptratul are arie maxim.
R13.9. Dintre toate triunghiurile dreptunghice cu aceeai ipotenuz aflai pe
cel de arie maxim.
Soluie.
2
c b
S

= unde b i c sunt lungimile catetelor. Din teorema lui Pitagora
avem c
2 2 2
a c b = + (constant). Suma factorilor
2
b i
2
c fiind constant, produsul
este maxim cnd
2 2
c b = , adic c b = . Deci triunghiul este dreptunghic isoscel.
Atunci cu teorema lui Pitagora obinem:
2 2 2
b b a + = , deci 2
2 2
= b a , de unde
2
2 a
b = . Deci
4 2
1
2
2
2
2
2
2
max
a a a c b
S = =

= .
R13.10. Dintre toate triunghiurile cu acelai perimetru, aria maxim o are
triunghiul echilateral.
Soluie. Fie a,b,c lungimile laturilor triunghiului i 2p perimetrul triunghiului.
Folosim formula lui Heron pentru aria triunghiului:
c) b)(p a)(p p(p S =
Suma factorilor variabili este p-a+p-b+p-c=3p-2p=p deci este constant.
Produsul celor trei factori este maxim cnd p-a=p-b=p-c, adic a=b=c. Deci triunghiul
de arie maxim este cel echilateral (cu perimetru constant).
4
3
2
max
a
S =
R13.11. Dintre toate patrulaterele cu acelai perimetru nscrise ntr-un cerc,
aria maxim o are ptratul.
Soluie. Dac notm cu a,b,c,d lungimile laturilor i cu 2p perimetrul, aria
patrulaterului nscris se calculeaz cu relaia:
d) c)(p b)(p a)(p p(p S =
Suma factorilor variabili este p-a+p-b+p-c=3p-2p=p (constant). Atunci S este
maxim cnd p-a=p-b=p-c=p-d a=b=c=d.
2
max
a S = .
R13.12. Dintre toate dreptunghiurile nscrise ntr-un cerc de raz dat R, care
este cel de arie maxim?
Soluie. Dac una din laturi are lungimea x, cealalt va avea lungimea
2 2
4 x R (calculat cu teorema lui Pitagora din triunghiul dreptunghic).
70
Atunci aria este: ) 4 ( 4
2 2 2 2 2
x R x x R x S = = . Suma factorilor
variabili
2
x i
2 2
4 x R este
2
4R (constant).
max
S se obine cnd factorii sunt egali
adic
2 2 2
4 x R x = , rezult 2 R x = . Cealalt latur a dreptunghiului are lungimea
2 2 4 4
2 2 2 2
R R R x R = = . Deci dreptunghiul de arie maxim este ptratul
nscris.
R13.13. Se consider un punct A' pe latura (BC) a unui triunghi ABC.
Paralela prin A' la latura AC ntlnete latura AB n punctul C' i paralela prin A' la
latura AB ntlnete latura AC n punctul B'.
a) S se exprime aria paralelogramului A'B'AC' n funcie de aria triunghiului
BA'C' i de aria triunghiului CA'B'.
b) Pentru ce poziie a punctului A' aria acestui paraloegram este maxim?
Demonstraie. Poziia punctului A' pe latura [BC] este dat de raportul
k
C A'
BA'
= 0) (k > .
A
B'
C'
B A' C

Atunci obinem
1 k
k
BA' C A'
BA'
+
=
+
, de unde
1 k
k
BC
BA'
+
= . Din k
C A'
BA'
=
mai obinem
k
1
BA'
C A'
= , de unde
1 k
1
C A' BA'
C A'
+
=
+
, deci
1 k
1
BC
C A'
+
= . Avem:
S[A'B'AC']=S[ABC]-S[BA'C']-S[CA'B'] (*)
Fiindc A'C'||AC, triunghiurile A'BC' i ABC sunt asemenea. i A'B'||AB,
atunci triunghiurile A'B'C i ABC sunt asemenea. Fiindc raportul ariilor a dou
triunghiuri asemenea este egal cu ptratul raportului de asemnare putem scrie relaiile:
2 2
k
1 k
BA'
BC
] C' S[BA'
S[ABC]

+
=

= (1)
iar
2
2
1) (k
CA'
CB
] B' S[CA'
S[ABC]
+ =

= (2)
Din relaiile (1) i (2) obinem:
71
2 2 2
2
k
1
1) (k
1
k
1) (k
] B' S[CA'
S[ABC]
:
] C' S[BA'
S[ABC]
=
+

+
= ,
deci
2
k
1
] C' S[BA'
C] B' S[A'
= (3)
Cu relaiile (1), (2), (3), relaia (*) devine:
=
+
= ] C' S[BA'
k
1
] C' S[BA' ] C' S[BA'
k
1) (k
] AC' B' S[A'
2 2
2

] C' S[BA'
k
2
k
1 k 1 2k k
] C' S[BA'
2
2 2
=

+ +
=
Deci
] C' S[BA'
k
2
] AC' B' S[A' = (4)
Cu (3), relaia (4) devine:
C] B' 2kS[A' C] B' S[A' k
k
2
] AC' B' S[A'
2
= =
Deci
S[A'B'AC']=2kS[A'B'C] (5)
Din (4) i (5) obinem: C] B' ]S[A' C' 4S[BA' ] AC' B' [A' S
2
= .
b) Din relaia (2) rezult c
S[ABC]
1) (k
1
] B' S[CA'
2
+
= (6)
Din relaiile (5) i (6) obinem:
S[ABC]
1) (k
2k
] AC' B' S[A'
2
+
= (7)
Maximul ariei S[A'B'AC'] are loc atunci cnd
2
1) (k
2k
+
este maxim (S[ABC]
constant). Dar
2
k
1
k
2
k
1 2k k
2
1) (k
2k
2 2
+ +
=
+ +
=
+
, care este maxim cnd
k
1
k +
este minim. Termenii pozitivi variabili k,
k
1
au produsul 1
k
1
k = (constant) suma
k
1
k + este minim cnd
k
1
k = , de unde 1 k
2
= 0) (k > rezult k=1. Deci A' este
mijlocul lui (BC). Atunci din (7) obinem:
72
S[ABC]
2
1
] AC' B' S[A' ariei Maximul =
R13.14. Determinai cel mai mare numr natural n pentru care numrul
123...20022003 se divide cu
11
29
+ n
.
Soluie. Numrul 29 este prim i divide pe fiecare al 29-lea numr natural.
Exist 69 (2003:29=69 rest 2) numere mai mici sau egale cu 2003 divizibile cu 29.
Exist 2 numere mai mici sau egale cu 2003 divizibile cu 29
2
(2003:29
2
=2 rest 321), iar
cu 29
3
nu exist numere mai mici sau egale cu 2003, divizibile. Deci numrul
123...20022003 se divide cu 29
69+2
=29
71
. Atunci 71=n+11 n=60. Numrul cutat
este 60.




73
14. Puncte laticiale

Definiia 14.1. Orice punct care are ambele coordonate exprimate prin numere
ntregi se numete punct laticial.


Probleme rezolvate

R14.1. S se demonstreze c nu exist dou puncte laticiale ) , ( b a A i
) , ( d c B ) ( B A care s aib aceeai distan la punctul de coordonate
|
.
|

\
|
7
5
, 3 C
) , , , ( Z d c b a .
Soluie. Folosim metoda reducerii la absurd. Presupunem c exist astfel de
puncte. Distana dintre punctele ) , (
1 1
y x P i ) , (
2 2
y x Q este dat de relaia:
2
1 2
2
1 2
) ( ) ( y y x x PQ + = . Atunci fie A, B punctele laticiale pentru care
CA=CB. Obinem:
2
2
2
2
7
5
) 3 (
7
5
) 3 (
|
.
|

\
|
+ =
|
.
|

\
|
+ d c b a ,
care se mai scrie:
) (
7
5
2 3 ) ( 2
2 2 2 2
d b b a d c a c + + = (1)
Cnd a c , n membrul stng este un numr iraional, iar n membrul drept
este un numr raional, rezult cu necesitate c 0 = a c i
0 ) (
7
10
2 2 2 2
= + + d b b a d c (2)
Din (2) pentru a c = obinem: 0 ) (
7
10
2 2
= + d b b d , care se mai scrie:
0
7
10
) ( 0 ) (
7
10
) )( ( =
|
.
|

\
|
+ = + b d b d b d b d b d
Fiindc Z d b, rezult c
7
10
+b d i atunci obinem c b d = . Fiindc
c a = i d b = rezult c punctele A i B coincid. Deci nu exist dou puncte laticiale
care s aib aceeai distan la punctul
|
.
|

\
|
7
5
, 3 C .
R14.2. S se arate c nu exist n planul raportat la dou axe ortogonale, nici
un triunghi echilateral, avnd toate cele trei vrfuri n puncte laticiale.
Soluie. Presupunem c exist un triunghi avnd vrfurile n puncte laticiale.
Putem de asemenea presupune c unul din vrfuri este n originea O, iar celelalte dou
74
sunt ) , ( b a A , ) , ( d c B . Atunci
2 2 2
b a OA + = ,
2 2 2
d c OB + = , iar
2 2 2
) ( ) ( d b c a BA + = .
Triunghiul OAB fiind echilateral avem:
x d b c a d c b a = + = + = +
2 2 2 2 2 2
) ( ) ( (1)
Fiindc a,b,c,d sunt ntregi rezult c Z x . Din (1) obinem:
x bd ac d b c a = + + + + ) ( 2
2 2 2 2

Fiindc x d c b a = + = +
2 2 2 2
, rezult c x bd ac x x = + + ) ( 2 , de unde
2
x
bd ax = + (2)
Avem relaia
2 2 2 2 2 2
) ( ) )( ( ) ( bd ac d c b a bc ad + + + = (3)
innd seama de (1) i (2), relaia (3) devine:
2
2
2
) (
|
.
|

\
|
=
x
x x bc ad ,
de unde obinem:
4
3
) (
2
2
x
bc ad = , rezult
2
) ( 2
3
(


=
x
bc ad
i deci
3 2 =
|
.
|

\
| +
x
bc ad
relaie imposibil fiindc n stnga avem un numr raional iar n
dreapta un numr iraional. Deci nu exist un triunghi care s satisfac ipoteza.
R14.3. S se demonstreze c dac pentru orice numr natural n exist n plan
un cerc de centru ) , ( b a O , care conine n interiorul su exact n puncte laticiale, atunci
a i b nu pot fi simultan numere raionale.
Demonstraie. Fie
d
c
x = i
d
e
y = cu Z d e c , , i 0 d . Punctele laticiale
) , ( c e A i ) , ( c e B au aceeai distan pn la punctul de coordonate ) , ( y x M
deoarece
2 2 2 2
|
.
|

\
|
+
|
.
|

\
|
=
|
.
|

\
|
+
|
.
|

\
|

d
e
c
d
c
e
d
e
c
d
c
e
Deci pentru orice punct de coordonate raionale exist dou puncte laticiale
distincte egal deprtate de acel punct. Dac Q a i Q b atunci exist dou puncte
laticiale distincte care sunt egal deprtate de punctul de coordonate ) , ( b a . Dac cercul
cu centrul ) , ( b a O care trece prin aceste dou puncte conine n interiorul su n puncte
laticiale, atunci orice cerc concentric cu el i de raz mai mare va conine n interiorul
su cel puin 2 + n puncte laticiale. n nici un caz nu exist un cerc cu centrul n
) , ( b a O care s conin exact 1 + n puncte. Deci sau Q a sau Q b .

75
GEOMETRIE

1. Relaii metrice n triunghiul oarecare

1.1. Teoreme ale bisectoarelor

Teorema 1.1.1. (Teorema bisectoarei interioare)
Fie triunghiul ABC, (AD bisectoarea interioar a unghiului A i D(BC),
atunci:
AC
AB
DC
DB
= .
Demonstraie. Ducem prin B o paralel la AD care ntlnete pe CA n M.
Aplicm teorema lui Thales n triunghiul CBM i obinem:
AC
AM
DC
DB
= (1)
(AD fiind bisectoarea interioar a unghiului A, avem c CADBAD, dar
BADABM (alterne interne) i BMADAC (corespondente). Deci
BMAABM. Obinem astfel c triunghiul ABM este isoscel cu
AM=AB (2)
Din relaiile (1) i (2) obinem c
AC
AB
DC
DB
= , i astfel teorema este
demonstrat.
Din
AC
AB
DC
DB
= obinem
AC
AC AB
DC
DC DB +
=
+
sau
AC
AC AB
DC
BC +
= , de
unde
AC AB
AC BC
DC
+

= , care se mai scrie


c b
ab
+
= DC (3)
Atunci
c b
ac
c b
ab
a
+
=
+
= = DC BC BD , deci
c b
ac
+
= DB (4)
M
B D
A
C
76
Observaie. Pentru un plus de precizie se poate numi aceasta "prima teorem a
bisectoarei interioare".

Teorema 1.1.2. (Teorema reciproc a teoremei bisectoarei interioare)
Fie triunghiul ABC, D(BC) astfel nct
AC
AB
DC
DB
= , atunci (AD este
bisectoarea interioar a unghiului A.
Demonstraie. Facem aceeai construcie ca la teorema direct, adic ducem
prin B o paralel la AD care ntlnete pe CA n M. Din ipotez avem c
AC
AB
DC
DB
= (1)
Fiindc AD||MB obinem
AC
AM
DC
DB
= (2)
Din (1) i (2) obinem AB=AM, deci triunghiul AMB este isoscel cu
ABMBMA (3)
Din paralelism avem:
MBABAD (alterne interne) (4)
i
BMADAC (corespondente) (5)
Din relaiile (3), (4), (5) obinem c BADDAC, adic (AD este
bisectoarea interioar a unghiului A.
Teorema 1.1.3. (Teorema bisectoarei exterioare)
Fie triunghiul ABC cu ABAC. Dac (AE este bisectoarea exterioar a
unghiului A, EBC, atunci
AC
AB
EC
EB
= .
Demonstraie. Ducem prin punctul B o paralel la AE care ntlnete pe AC n
N. Aplicm teorema lui Thales n triunghiul CAE:
AC
AN
EC
EB
= (1)
Din paralelism obinem ANBMAE (corespondente), ABNEAB
(alterne interne). Din ipotez avem c MAEEAB, deci ANBABN, adic
triunghiul ABN este isoscel cu
(AB)(AN) (2)
Din (1) i (2) obinem:
AC
AB
EC
EB
= i teorema este demonstrat.
77

Din
AC
AB
EC
EB
= obinem (n ipoteza AC>AB)
AB AC
AB
BC
EB
AB AC
AB
EB EC
EB


de unde
AB AC
AB BC
EB

= , care se mai scrie


c b
c a

= EB . Atunci
c b
ab
c b
ac ab ac
a
c b
ac

+
= +

= + = BC EB EC
Deci
c b
ab

= EC .
Observaii. 1) Condiia ABAC din teorema bisectoarei exterioare este
esenial deoarece dac AB=AC atunci bisectoarea exterioar a unghiului A este
paralel cu BC, deci nu mai exist E.
2) Teorema bisectoarei exterioare a fost demonstrat de Pappus.
3) Un plus de precizie cere s numim aceast teorem drept "prima a
bisectoarei exterioare".
Aplicaie. Dac I este centrul cercului nscris n ABC i {D}=AIBC are loc
relaia:
c b
a
+
=
IA
ID
.
Demonstraie. Cu teorema bisectoarei n ABD obinem:
BA
BD
IA
ID
= , dar
c b
ac
+
= BD , atunci
c c b
ac 1
IA
ID

+
= , deci
c b
a
+
=
IA
ID
.
M
A
N
C
B E
78

Teorema 1.1.4. (Teorema reciproc a teoremei bisectoarei exterioare)
Fie triunghiul ABC i EBC\[BC] astfel nct
AC
AB
EC
EB
= , (1)
atunci (AE este bisectoarea exterioar a unghiului A.
Demonstraie. Trebuie ca ABAC, deoarece dac AB=AC, din (1) ar rezulta EB=EC,
relaie imposibil din cauza poziiei lui E pe BC. Facem aceeai construcie ca la
teorema bisectoarei exterioare, adic ducem prin B o paralel la AE care ntlnete pe
CA n N. Cu teorema lui Thales n triunghiul CAE obinem:
AC
AN
EC
EB
= , iar din
ipotez
AC
AB
EC
EB
= , deci AN=AB, adic ABN este isoscel cu ABNANB. Dar
ABNEAB i ANBMAE, deci EABMAE, relaie ce asigur c (AE este
bisectoarea exterioar a unghiului A.

1.2. Teorema lui Pitagora generalizat

Teorema lui Pitagora din triunghiul dreptunghic admite o generalizare pentru
triunghiul oarecare.
Teorema 1.2.1. ntr-un triunghi oarecare ptratul lungimii unei laturi este egal
cu suma ptratelor lungimilor celorlalte dou laturi din care se scade sau se adun
dublul produsului lungimii uneia dintre aceste laturi cu lungimea proieciei celeilalte pe
ea
CD CB 2 CB CA AB
2 2 2
+ = (1)
CD CB 2 CB CA AB
2 2 2
+ + = (2)
Demonstraie. Fie ABC un triunghi i D proiecia lui A pe BC. Din
triunghiurile ABD i ACD dreptunghice n D obinem:
2 2 2 2 2 2
DC AD AC , BD AD AB + = + =
1) Dac m(C)<90 atunci trebuie s artm c:
CD CB 2 CB CA AB
2 2 2
+ = (1)
A
B D
C
I
79
Avem BD=|BC-CD|. ntr-adevr dac m(B)<90 atunci D[BC] i BD=BC-
CD. Dac m(B)=90 D coincide cu B i avem BD=0=BC-BC. Dac m(B)>90,
B(CD) i BD=DC-BC=|BC-DC|. Avem deci
= + = + =
2 2 2 2 2
CD) (BC AD BD AD AB
CD CB 2 CB CA CD BC 2 BC ) DC (AD
2 2 2 2 2
+ = + + =
2) Dac m(C)>90 trebuie s demonstrm c:
CD CB 2 CB CA AB
2 2 2
+ + =
n acest caz avem BD=BC+CD i obinem:
= + + = + =
2 2 2 2 2
CD) (BC AD BD AD AB
CD 2CB CB CA CD 2BC BC ) DC (AD
2 2 2 2 2
+ + = + + + =
Observaii. 1) Semnul din faa dublului produs este influenat numai de
unghiul opus laturii pe care o calculm.
2) Enunul teoremei lui Pitagora generalizat nu prevede drept caz special
alternativa m(C)=90 n care funcioneaz teorema lui Pitagora propriu zis:
2 2 2
CB CA AB + = . Acest caz poate fi inclus, relaiile (1) sau (2) conservndu-i
valabilitatea.
3) Teorema lui Pitagora generalizat ne ajut s facem o demonstraie rapid a
reciprocei teoremei lui Pitagora.
4) Teorema lui Pitagora generalizat i reciproca teoremei lui Pitagora ne
permit s precizm (dup msura unghiurilor) felul triunghiului: obtuzunghic,
ascuitunghic, dreptunghic.


1.3. Teorema lui Stewart

Teorema 1.3.1. Fie M un punct pe latura [BC] a unui triunghi ABC. Are loc
relaia:
CM BM BC MB AC MC AB BC AM
2 2 2
+ = (1)
Demonstraie. Fie L proiecia punctului A pe BC. Aplicm teorema lui
Pitagora generalizat n triunghiurile ACM i AMB. Obinem:
BM | ML 2MC MC AM AC
2 2 2
+ =
A
B D C B C D
A
80
MC | ML 2BM MB AM AB
2 2 2
+ + =
Dac nmulim cele dou egaliti, prima cu BM, iar a doua cu MC i le
adunm membru cu membru se va reduce termenul 2MCBMML i obinem:
+ + + = + MC MB MC AM BM MC BM AM MC AB BM AC
2 2 2 2 2 2

+ + + = + MC MB BM MC MC) (BM AM MC AB BM AC
2 2 2 2 2

+ + = + MB) BM(MC MC BC AM MC AB BM AC
2 2 2

+ = + BC BM MC BC AM MC AB BM AC
2 2 2

MC BM BC BM AC MC AB BC AM
2 2 2
+ =
adic obinem relaia de demonstrat.


1.4. Teorema lui Van Aubel n triunghiul dreptunghic

Teorema 1.4.1. Dac M este un punct oarecare pe ipotenuza [BC] a unui
triunghi dreptunghic ABC, are loc relaia:
2 2 2 2 2 2
BC AM AC MB AB MC = + (1*)
Demonstraie. Aplicm teorema lui Stewart:
MC BM BC BM AC MC AB BC AM
2 2 2
+ = (1)
nmulim relaia (1) cu MC i obinem:
2 2 2 2 2
MC BM BC MC BM AC MC AB MC BC AM + = (2)
nmulim relaia (1) cu MB i obinem:
MC BM BC BM AC MB MC AB MB BC AM
2 2 2 2 2
+ = (3)
Adunm membru cu membru relaiile (2) i (3) i grupnd, obinem:
A
B M L
C
A C
M
B
81
+ + = +
2 2 2 2 2
BM AC MC AB MB) (MC BC AM
MB) MC(MC BM BC MC MB AC MB MC AB
2 2
+ + +
sau
+ + + = ) AC MB(AB MC BM AC MC AB BC M
2 2 2 2 2 2 2 2
A
MC BM BC
2
(4)
Dar
2 2 2
BC AC AB = + i atunci (4) devine
MC BM BC BC MB MC BM AC MC AB BC AM
2 2 2 2 2 2 2 2
+ + =
deci
2 2 2 2 2 2
BM AC MC AB BC AM + = .
Corolar 1.4.1. Ptratul lungimii bisectoarei este medie armonic ntre
ptratele lungimilor segmentelor determinate de ea pe ipotenuz.
Dac AM este bisectoare atunci
2 2
2 2
2
MC MB
MC 2MB
AM
+

= .
Cu teorema bisectoarei obinem
MC
MB
AC
AB
= , de unde
2
2
2
2
MC
MB
AC
AB
= sau
2 2 2 2
MB AC MC AB = (5)
Cu (5) relaia lui Van Aubel (1*) se mai scrie
2 2 2 2 2 2 2 2 2
MA ) AC (AB MC AB 2 MA BC MC 2AB + = =
sau

+
=
2 2
2
2 2
2
2 2 2
2 2
2
MC AB
AC
MC AB
AB
AM
2
MC AB
AC AB
MA
2

2 2 2 2 2
2
2 2
MB
1
MC
1
AM
2
MB AC
AC
MC
1
AM
2
+ =

+ =
deci
2 2
2 2
2
MB MC
MC 2MB
AM
+

= .
Corolar 1.4.2. Dac triunghiul ABC este dreptunghic isoscel, avem:
2
MC MB
MA
2 2
2
+
=
Dac AB=AC relaia (1*) devine:
) BC AB (AB 2AB AM ) MB (MC AB
2 2 2 2 2 2 2 2
= + = +
sau
2
MB MC
AM
2 2
2
+
= .


82
1.5. Calculul lungimii bisectoarelor interioare

Teorema 1.5.1. Fie ABC un triunghi i [AD bisectoarea interioar a unghiului
BAC, cu D(BC). Atunci ) (
) (
4
AD
2
2
a p p
c b
bc

+
= sau
) (
2
AD a p bcp
c b

+
= (unde
2
c b a
p
+ +
= ) sau ) (
2
a p bcp
c b
l
a

+
= .
Demonstraie. Din teorema bisectoarei avem c
DC
BD
AC
AB
= , care se mai scrie
DC
BD
=
b
c
, de unde obinem:
DC
BC
DC
DC BD
=
+

+
=
+
b
b c
b
b c
, de unde
c b
ab
+
= DC . Atunci
c b
ac
c b
ab
a
+
=
+
= = DC BC BD . Deci
c b
ac
+
= BD . Vom
scrie relaia lui Stewart pentru cazul cnd M este n D. Obinem:
DC BD BC BD AC DC AB BC DA
2 2 2
+ =
nlocuind pe BC cu a, AB cu c, AC cu b i DC cu
c b
ab
+
, iar BD cu
c b
ac
+

obinem:
c b
ab
c b
ac
a
c b
ac
b
c b
ab
c a
+

+

+
+
+
=
2 2 2
AD ,
de unde
2
2 2 2 2
) (
AD
c b
bc
a
c b
c
b
c b
b
c
+

+
+
+
= ,
care se mai scrie:
=
(

+
+
+
=
|
|
.
|

\
|
+
+
+
=
c b
a c b
c b
bc
c b
a
b c
c b
bc
2 2 2
2
) (
AD
) 2 2 ( 2
) (
) )( (
2
a p p
c b
bc
c b
a c b a c b
c b
bc

+
=
+
+ + +

+
=
(Am inut seama c c b a p + + = 2 i atunci a c b a p + = 2 2 ).
Deci
2
2
) (
) ( 4
AD
c b
a p bcp
+

= , adic relaia de demonstrat.

1.6. Calculul lungimii nlimilor unui triunghi

Teorema 1.6.1. Dac ABC este un triunghi oarecare avem
) )( )( (
2
c p b p a p p
a
h
a
= ,
83
unde h
a
este nlimea corespunztoare laturii a, iar
2
c b a
p
+ +
= .
Demonstraie. n triunghiul ABC, ducem nlimea AA' (h
a
). Dintre cele dou
unghiuri B i C cel puin unul este ascuit. S presupunem c acesta este B. Cu teorema
lui Pitagora generalizat obinem:
C A' BC 2 AC BC AB
2 2 2
+ = ,
de unde obinem:
2BC
AB AC BC
C A' AB AC BC C A' BC 2
2 2 2
2 2 2
+
= + = ,
care se mai scrie
a
c b a
2
C A'
2 2 2
+
= .
Din triunghiul dreptunghic AA'C (m(A')=90) cu teorema lui Pitagora
obinem:
=
|
|
.
|

\
| +
= =
2
2 2 2
2 2 2 2
2
' '
a
c b a
b C A AC AA
=
|
|
.
|

\
| +
+
|
|
.
|

\
| +
=
a
c b a
b
a
c b a
b
2 2
2 2 2 2 2 2

=
+ +

+
=
a
c b a ab
a
c b a ab
2
2
2
2
2 2 2 2 2 2

=
(

+
=
a
c b a
a
ab b a c
2
) (
2
)] 2 ( [
2 2 2 2 2

=
+ + +


=
a
c b a c b a
a
b a c
2
) )( (
2
) (
2 2

=
+ + + + +
=
2
4
) )( )( )( (
a
c b a c b a b a c b a c

2 2
) )( )( ( 4
4
2 ) 2 2 )( 2 2 )( 2 2 (
a
c p b p a p p
a
p c p a p b p
=

=
) 2 2 , 2 2 , 2 2 ( c b a c p c b a b p a c b a p + = + = + =
Deci
2
2
) )( )( ( 4
' AA
a
c p b p a p p
= , de unde
) )( )( (
2
' AA c p b p a p p
a
= ,
adic tocmai relaia de demonstrat.

84
1.7. Teorema medianei

Teorema 1.7.1. ntr-un triunghi ABC avem relaia:
4
) ( 2
2 2 2
2
a c b
m
a
+
= , (1)
unde a=BC, b=CA, c=AB, m
a
=AA', A' este mijlocul lui [BC].
Demonstraie. Scriem relaia lui Stewart n cazul cnd M este mijlocul A' al
segmentului [BC]. Obinem:
+ = CA' BA' BC B A' AC C A' AB BC A A'
2 2 2

+ =
2 2 2 2
2 2 2
a a
a
a
b
a
c a m
a

4
) ( 2
2 2 2
2
a b c
m
a
+
= ,
adic relaia de demonstrat.
Fig. 1.
Probleme rezolvate

R1.7.1. n orice triunghi suma ptratelor lungimilor medianelor este trei
ptrimi din suma ptratelor lungimilor laturilor, adic:
) (
4
3
2 2 2 2 2 2
c b a m m m
c b a
+ + = + + (2)
Demonstraie. Din relaia (1) i analoagele obinem:
4
) ( 2
2 2 2
2
a c b
m
a
+
= ,
4
) ( 2
2 2 2
2
b c a
m
b
+
= ,
4
) ( 2
2 2 2
2
c b a
m
c
+
= .
Atunci
=
+
+
+
+
+
= + +
4
) ( 2
4
) ( 2
4
) ( 2
2 2 2 2 2 2 2 2 2
2 2 2
c b a b c a a c b
m m m
c b a

4
) ( 3
4
2 2 2 2 2 2
2 2 2 2 2 2 2 2 2 2 2 2
c b a c b a b c a a c b + +
=
+ + + + +
= .
A
B A' L
C
85
R1.7.2. Dac G este punctul de concuren a medianelor AA', BB', CC' al
triunghiului ABC, s se demonstreze relaia:
4
CC' ) GC' (GC BB' ) GB' (GB AA' ) GA' (GA
2 2 2
c b a + +
= + +
Soluie. inem seama c centrul de greutate al unui triunghi se gsete la
3
2
de
vrf i la
3
1
de baz, adic
a
m
3
2
GA = ,
a
m
3
1
' GA = ,
b
m
3
2
GB= ,
b
m
2
1
' GB = ,
c
m
3
2
GC = ,
c
m
3
1
' GC = , relaia de demonstrat devine:
=
+ +
=
|
.
|

\
|
+
|
.
|

\
|
+
|
.
|

\
|

3 3
1
3
2
3
1
3
2
3
1
3
2
2 2 2
2 2 2 c b a
c b a
m m m
m m m
4
) (
3
1
4
3
2 2 2
2 2 2
c b a
c b a
+ +
= + + = .
Am inut seama c ) (
4
3
2 2 2 2 2 2
c b a m m m
c b a
+ + = + + .
Fig. 2.

R1.7.3. ntr-un triunghi dreptunghic cu lungimile catetelor b i c i lungimea
ipotenuzei a avem relaia
4
5
2
2 2
a
m m
c b
= + (4)
Demonstraie. ntr-un triunghi dreptunghic mediana m
a
corespunztoare
ipotenuzei aare lungimea
2
a
. Atunci folosind relaia
) (
4
3
2 2 2 2 2 2
c b a m m m
c b a
+ + = + +
i innd seama c
2 2 2
a c b = + (teorema lui Pitagora), obinem:
B A' C
C'
B'
A
G
86
) (
4
3
2
2 2 2 2
2
a a m m
a
c b
+ = + +
|
.
|

\
|
,
de unde
4
5
2
2 2
a
m m
c b
= + , (5)
iar dac triunghiul este dreptunghic isoscel relaia (4) devine
4
5
2
2
2
a
m
b
= , de unde
8
5
2
2
a
m
b
= .
R1.7.4. Triunghiul cu dou mediane congruente este isoscel.
Soluie. Din
2 2
b a b a
m m m m = = sau
4
) ( 2
4
) ( 2
2 2 2 2 2 2
b c a a c b +
=
+
,
care se mai scrie
2 2 2 2 2 2
2 2 2 2 b c a a c b + = + , de unde
2 2
3 3 b a = , deci b a = .
R1.7.5. Diferena ptratelor lungimilor a dou laturi ale unui triunghi este egal
cu dublul produs dintre lungimea laturii a treia i lungimea proieciei medianei
corespunztoare pe acea latur.
Aplicm teorema lui Pitagora generalizat n triunghiurile ABA' i ACA'
obinem (Fig. 1):
L A' B 2A' B A' A A' AB
2 2 2
+ + =
i
L A' C 2A' C A' A A' AC
2 2 2
+ = ,
de unde prin scdere membru cu membru obinem:
L A' 2CB AC AB
2 2
=
(am inut seama c A'B=A'C) relaie care se mai scrie (fr a restrnge generalitatea
considerm AB>AC)
L A' 2
2 2
= a b c (6)
adic:
R1.7.6. Se consider triunghiul ABC cu mediana AD i nlimea AE. S se
demonstreze relaia:
DE BC
4
BC
AD AB
2
2 2
=
|
|
.
|

\
|
+
Soluie. n triunghiul ABD cu m(ADB)<90, aplicm teorema lui Pitagora
generalizat:
DE 2BD AD BD AB
2 2 2
+ = (*)
Aplicm aceeai teorem n triunghiul ADC cu m(ADC)>90 i obinem:
DE 2CD CD AD AC
2 2 2
+ + = (**)
Adunm membru cu membru relaiile (*) i (**) i obinem:
87
2 2 2 2
2AD 2BD AC AB + = + (***)
Fig. 3.

Din relaia (6) avem:
ED 2BC AC AB
2 2
=
Adunnd membru cu membru ultimele dou relaii obinem:
ED 2BC 2AD 2BD 2AB
2 2 2
+ + =
sau
ED BC AD BD AB
2 2 2
+ + = ,
care se mai scrie:
ED BC
2
BC
AD AB
2
2 2
= |
.
|

\
|
,
relaie care trebuia demonstrat.

1.8. Teorema lui Leibniz

Teorema 1.8.1. Dac M este un punct arbitrar n planul triunghiului ABC, iar
G este centrul de greutate al triunghiului, are loc relaia:
2 2 2 2 2 2 2
GC GB GA 3MG MC MB MA + + + = + + (1*)
Demonstraie. Fie A' mijlocul lui [BC]. Scriem relaia lui Stewart pentru
triunghiul MAA' i punctul G(AA') i obinem:
GA' AG AA' AG MA' G A' MA AA' MG
2 2 2
+ =
Atunci obinem:
2 2 2 2
AA'
9
2
MA'
3
2
MA
3
1
MG + = (1)
Fiindc MA' este median n triunghiul MBC, avem
4
) ( 2
'
2 2 2
2
BC MC MB
MA
+
= (2)
Cu (2), relaia (1) devine:
2
2 2 2
2 2
AA'
9
2
4
BC ) MC 2(MB
3
2
MA
3
1
MG
+
+ = ,
care se mai scrie (prin nmulirea cu 3)
A
B E D C
88
2
2
2 2 2 2
AA'
3
2
2
BC
MC MB MA 3MG + + = (3)
Fiindc GA' este median n triunghiul GBC obinem:
4
BC ) GC 2(GB
GA'
2 2 2
2
+
= ,
de unde
2 2 2 2
4GA' ) GC 2(GB BC + = (4)
Fig. 4.

Cu (4), relaia (3) devine:
2 2 2 2 2 2 2 2
AA'
3
2
2GA' GC GB MC MB MA 3MG + + + = (5)
Folosind egalitile GA
2
1
GA' = i GA
2
3
AA' = , relaia (5) devine:
2
2
2 2 2 2 2 2
GA
4
9
3
2
4
GA
2 GC GB MC MB MA 3MG + + + = ,
care se mai scrie
2 2 2 2 2 2 2
GC GB GA 3MG MC MB MA + + + = + + ,
relaie ce trebuia demonstrat.
Consecina 1.8.1. Suma ptratelor distanelor de la M la vrfurile triunghiului
este minim cnd M coincide cu G.
Fiindc ) (
4
3
2 2 2 2 2 2
c b a m m m
c b a
+ + = + + i
a
m
3
2
GA = ,
b
m
3
2
GB= ,
c
m
3
2
GC = atunci (1*) devine:
) (
3
1
3MG MC MB MA
2 2 2 2 2 2 2
c b a + + + = + + (6)
A
G
B A'
C
M
89
iar cnd MG=0 obinem
3
MC MB MA
2 2 2
2 2 2
c b a + +
= + + .
Consecina 1.8.2. Dac M coincide cu centrul cercului circumscris O, atunci
)] ( 9 [
9
1
OG
2 2 2 2 2
c b a R + + = (7)
Cnd M coincide cu O avem OA=OB=OC=R, iar (6) devine:
) (
3
1
3OG OC OB OA
2 2 2 2 2 2 2
c b a + + + = + + ,
adic ) (
3
1
OG 3 3
2 2 2 2 2
c b a R + + + = , de unde obinem:
9
) ( 9
OG
2 2 2 2
2
c b a R + +
= , adic relaia (7).
Din 0 OG
2
rezult 0 ) ( 9
2 2 2 2
+ + c b a R , de unde
9
2 2 2
2
c b a
R
+ +
.
Consecina 1.8.3.
) ( 9 OH
2 2 2 2 2
c b a R + + =
)] ( 9 [
9
4
GH
2 2 2 2 2
c b a R + + =
Se folosesc relaiile OH=3OG i GH=2OG. Pentru
3
OH
OG = , relaia (7)
devine
)] ( 9 [
9
1
9
OH
2 2 2 2
2
c b a R + + = ,
de unde
) ( 9 OH
2 2 2 2 2
c b a R + + = (8)
Cu GH=2OG, adic
2
GH
OG = , relaia (7) devine
)] ( 9 [
9
1
4
GH
2 2 2 2
2
c b a R + + = ,
de unde
)] ( 9 [
9
4
GH
2 2 2 2 2
c b a R + + =
90
Consecina 1.8.4. Dac O
9
este centrul cercului lui Euler avem
)] ( 9 [
36
1
G O
2 2 2 2 2
9
c b a R + + =
Avem relaia
6
OH
GO
9
= , de unde
2
9
2
GO 36 OH = iar din (8) obinem:
) ( 9 GO 36
2 2 2 2 2
9
c b a R + + = ,
de unde
36
) ( 9
GO
2 2 2 2
2
9
c b a R + +
= .

1.9. Aplicaii

1.9.1. Fie A, B, C, D vrfurile unui ptrat i M, N, P, Q mijloacele laturilor
lui. S se demonstreze c, dac O este un punct oarecare din planul ptratului, atunci
expresia:
) OQ OP ON (OM OD OC OB OA
2 2 2 2 2 2 2 2
+ + + + + +
are ca valoare aria ptratului.
Demonstraie. Aplicm teorema medianei n triunghiurile: AOB, BOC, COD,
DOA i obinem:
4
AB ) OB 2(OA
OM
2 2 2
2
+
= ,
4
BC ) OC 2(OB
ON
2 2 2
2
+
=
4
CD ) OD 2(OC
OP
2 2 2
2
+
= ,
4
AD ) OA 2(OD
OQ
2 2 2
2
+
=
Adunnd membru cu membru cele patru relaii de mai sus obinem:
2 2 2 2 2 2 2 2 2
4AB ) OD OC OB 4(OA ) OQ OP ON 4(OM + + + = + + +
(am inut seama c AB=BC=CD=DA), de unde obinem
2 2 2 2 2 2 2 2 2
AB OQ OP ON OM OD OC OB OA = + + + + + + ,
adic relaia cerut.
Teorema 1.9.2. (Teorema lui Euler pentru patrulater)
O
A
M B
N
C
P
D
Q
91
n orice patrulater ABCD suma ptratelor lungimilor laturilor este egal cu
suma ptratelor lungimilor diagonalelor plus de patru ori ptratul segmentului care
unete mijloacele diagonalelor, adic
2 2 2 2 2 2 2
4EF BC AC DA CD BC AB + + = + + + (1*)
unde [AC] i [BD] sunt diagonalele patrulaterului ABCD, iar E este mijlocul lui [AC],
F este mijlocul lui [BD].
Demonstraie. Cu teorema medianei n triunghiul ABC obinem:
4
AC ) AC 2(AB
BE
2 2 2
2
+
=
Din triunghiul ADC cu aceeai teorem obinem:
4
AC ) DC 2(AD
DE
2 2 2
2
+
=
Din triunghiurile ABD i CBD, pentru medianele [AF] i [CF] avem relaiile:
4
BD ) AB 2(AD
AF
2 2 2
2
+
= ,
4
DB ) CB 2(CD
CF
2 2 2
2
+
=
Adunnd membru cu membru cele patru relaii de mai sus obinem:
2
BD
2
AC
DA CD BC AB CF AF DE BE
2 2
2 2 2 2 2 2 2 2
+ + + = + + + (1)
Aplicm teorema medianei n triunghiurile BED i AFC i obinem
4
BD ) DE 2(BE
EF
2 2 2
2
+
= ,
4
AC ) CF 2(AF
EF
2 2 2
2
+
=
de unde
2
BD
2EF DE BE
2
2 2 2
+ = + (2)
i
2
AC
2EF CF AF
2
2 2 2
+ = + (3)
Folosind relaiile (2) i (3), relaia (1) devine:
D
C
B
A
F
E
92
2
BD
2
AC
DA CD BC AB
2
AC
2EF
2
BD
2EF
2 2
2 2 2 2
2
2
2
2
+ + + = + + + ,
relaie echivalent cu
2 2 2 2 2 2 2
DA CD BC AB BD AC 4EF + + + = + + ,
adic tocmai relaia de demonstrat.
Cazuri particulare
1) ABCD paralelogram (mijloacele diagonalelor coincid). Deci EF=0. Atunci
(1*) devine
2 2 2 2 2 2
BD AC DA CD BC AB + = + + + ,
relaie care se mai scrie
2 2 2 2
BD AC ) BC 2(AB + = + deci:
n orice paralelogram suma ptratelor lungimilor laturilor este egal cu suma
ptratelor lungimilor diagonalelor.
2) ABCD trapez cu bazele AB i CD (AB>CD). Atunci
2
DC AB
EF

= i
(1*) devine

|
.
|

\
|
+ + = + + +
2
2 2 2 2 2 2
2
DC AB
4 BD AC DA CD BC AB
DC 2AB CD BC BD AC
2 2 2 2
+ + = + ,
deci
ntr-un trapez suma ptratelor lungimilor diagonalelor este egal cu suma
ptratelor lungimilor laturilor neparalele plus de dou ori produsul lungimilor bazelor.
3) ABCD este dreptunghi.
Atunci relaia (1*) devine
2 2 2
2AC ) BC 2(AB = + sau
2 2 2
AC BC AB = +
adic obinem teorema lui Pitagora.

1.10. Teorema lui Menelaus

Teorema 1.10.1. Fie ABC un triunghi A', B', C' trei puncte astfel nct
A'BC, B'CA, C'AB. Dac punctele A', B', C' sunt coliniare, atunci are loc relaia:
1
B C'
A C'
A B'
C B'
C A'
B A'
=
Demonstraie. 1) Transversala C'A' intersecteaz dou laturi i prelungirea
celeilalte laturi a triunghiului.
A
C'
B1
B
A1
B'
C1
C A'
93
Proiectm vrfurile triunghiului ABC pe dreapta A'C', i obinem punctele A
1
,
B
1
, C
1
.
Fiindc CC
1
||BB
1
rezult c A'BB
1
~A'CC
1
. Atunci obinem:
1
1
CC
BB
C A'
B A'
= (1)
Triunghiurile CC
1
B' i AA
1
B' sunt asemenea (CC
1
||AA
1
) i atunci
1
1
AA
CC
A B'
C B'
= (2)
i triunghiurile AC'A
1
i BC'B
1
sunt asemenea (AA
1
||BB
1
) i avem:
1
1
BB
AA
B C'
A C'
= (3)
nmulind relaiile (1), (2) i (3) membru cu membru i simplificnd, obinem:
1
B C'
A C'
A B'
C B'
C A'
B A'
= .
2) Transversala C'A' intersecteaz prelungirile laturilor triunghiului ABC.
Proiectm vrfurile triunghiului ABC pe dreapta A'C' i obinem respectiv
punctele A
1
, B
1
, C
1
.
Fiindc CC
1
||BB
1
, triunghiurile A'BB
1
i A'CC
1
sunt asemenea. Atunci are loc
relaia:
1
1
CC
BB
C A'
B A'
= (1)
Din asemnarea triunghiurilor B'CC
1
i B'AA
1
(CC
1
||AA
1
) obinem:
1
1
AA
CC
A B'
C B'
= (2)
Acum din asemnarea triunghiurilor C'AA
1
i C'BB
1
(AA
1
||BB
1
) rezult c:
1
1
BB
AA
B C'
A C'
= (3)
Prin nmulirea relaiilor (1), (2) i (3) membru cu membru i dup simplificri,
obinem: 1
B C'
A C'
A B'
C B'
C A'
B A'
= , adic relaia ce trebuia demonstrat.
Teorema 1.10.2. (Reciproca teoremei lui Menelaus)
A
B
C'
B1
A1
C1
C
B'
A'
94
Considerm un triunghi ABC i punctele A'BC, B'AC, C'AB. Dac dou
dintre punctele A', B', C' sunt situate pe dou laturi ale triunghiului, iar al treilea punct
este situat pe prelungirea celei de-a treia laturi sau c toate punctele A', B', C' sunt pe
prelungirile laturilor triunghiului i are loc relaia:
1
B C'
A C'
A B'
C B'
C A'
B A'
= (1)
atunci punctele A', B', C' sunt coliniare.
Demonstraie. Considerm cazul cnd dou puncte sunt pe laturi i cellalt pe
prelungirea celei de-a treia laturi. Presupunem c dreapta A'B' intersecteaz latura [AB]
n punctul C''C'. Aplicm teorema lui Menelaus pentru punctele A', B', C''. Obinem:
1
B ' C'
A ' C'
A B'
C B'
C A'
B A'
= (2)
Din relaiile (1) i (2) obinem:
B ' C'
A ' C'
B C'
A C'
=
Fiindc exist un singur punct interior unui segment care mparte segmentul
ntr-un raport dat, rezult c punctul C'' coincide cu C', adic punctele A', B', C' sunt
coliniare.

1.11. Teorema lui Van Aubel

Teorema 1.11.1. Fie ABC un triunghi i punctele A'(BC), B'CA, C'AB.
Dac dreptele AA', BB', CC' sunt concurente ntr-un punct M atunci are loc relaia:
MA'
MA
B C'
A C'
C B'
A B'
= +
A
C',C''
B
C
A'
B'
95
Demonstraie. Aplicm teorema lui Menelaus pentru triunghiul AA'C i
transversala BB'. Obinem:
1
MA'
MA
A B'
C B'
BC
BA'
= (1)
Din relaia (1) obinem:
MA'
MA
BC
BA'
C B'
A B'
= (2)
Aplicm acum teorema lui Menelaus pentru triunghiul AA'B i transversala
CC'. Obinem:
1
B C'
A C'
MA
MA'
CA'
CB
= ,
de unde rezult c
MA'
MA
CB
CA'
B C'
A C'
= (3)
Adunnd relaiile (2) i (3) membru cu membru obinem:
|
.
|

\
| +
= +
BC
CA' BA'
MA'
MA
B C'
A C'
C B'
A B'
,
de unde
MA'
MA
B C'
A C'
C B'
A B'
= + .

1.12. Teorema lui Ceva

Teorema 1.12.1. (Teorema lui Ceva)
Se consider triunghiul ABC i punctele A'BC, B'CA, C'AB. Dac
dreptele AA', BB', CC' sunt concurente, atunci
1
B C'
A C'
A B'
C B'
C A'
B A'
=
Demonstraie. 1) Considerm punctul O de intersecie al dreptelor AA', BB',
CC' ca fiind situat n interiorul triunghiului. Considerm triunghiul BCC' i
transversala AA'. Cu teorema lui Menelaus obinem:
A
B
C
A'
M
C' B'
96
1
AB
AC'
OC'
OC
C A'
B A'
= (1)
Aplicm acum teorema lui Menelaus pentru triunghiul AC'C i transversala
BB'. Obinem:
1
BC'
BA
A B'
C B'
OC
OC'
= (2)
nmulind membru cu membru relaiile (1) i (2) i simplificnd obinem:
1
B C'
A C'
A B'
C B'
C A'
B A'
= , adic relaia ce trebuia demonstrat.
2) Considerm punctul de intersecie al dreptelor AA', BB', CC' ca fiind O i
care este situat n exteriorul triunghiului ABC.
Aplicm teorema lui Menelaus pentru triunghiul AA'B i transversala COC'.
Obinem:
1
B C'
A C'
OA
OA'
CA'
CB
= (1)
Aplicm acum teorema lui Menelaus pentru triunghiul AA'C i transversala
BOB'. Obinem:
1
OA'
OA
A B'
C B'
BC
BA'
= (2)
nmulim membru cu membru relaiile (1) i (2) i obinem dup simplificri
1
B C'
A C'
A B'
C B'
C A'
B A'
=
A
C'
B A' C
B'
O

C'
O
B'
B
C
A
A'
97
Teorema 1.12.2. (Reciproca teoremei lui Ceva)
Dac pe laturile unui triunghi ABC considerm punctele A', B', C' (A'BC,
B'CA, C'AB) toate pe laturi sau unul pe laturi i celelalte dou pe prelungiri, astfel
nct s avem relaia:
1
B C'
A C'
A B'
C B'
C A'
B A'
=
atunci dreptele AA', BB', CC' sunt concurente.
Demonstraie. Presupunem c dreptele AA', BB', CC' nu sunt concurente. Fie
AA'BB'={O} i COAB={C''}, C'C''. Din teorema lui Ceva obinem
1
B ' C'
A ' C'
A B'
C B'
C A'
B A'
= (1)
Din ipotez avem
1
B C'
A C'
A B'
C B'
C A'
B A'
= (2)
Din relaiile (1) i (2) rezult C' este identic cu C''.


1.13. Concurena liniilor importante n triunghi

1.13.1. Medianele unui triunghi sunt concurente n punctul G (centrul de
greutate al triunghiului).
Demonstraie. Fie AA', BB', CC' medianele triunghiului ABC. Atunci A', B', C'
sunt mijloacele laturilor [BC], [CA] i respectiv [AB].
Aplicm reciproca teoremei lui Ceva i obinem:
1
B C'
A C'
A B'
C B'
C A'
B A'
= ,
adic medianele sunt concurente.
1.13.2. Bisectoarele interioare ale unghiurilor unui triunghi sunt concurente n
I (centrul cercului nscris).
Demonstraie. Cu teorema bisectoarei interioare obinem:
AC
AB
C A'
B A'
= ,
BA
BC
A B'
C B'
= i
CB
CA
B C'
A C'
= .
A
C'
B A' C
B'
G
98

Prin nmulirea relaiilor de mai sus membru cu membru obinem:
1
CB
CA
BA
BC
AC
AB
B C'
A C'
A B'
C B'
C A'
B A'
= = ,
de unde conform reciprocei teoremei lui Ceva obinem c bisectoarele interioare ale
unghiurilor unui triunghi sunt concurente.
1.13.3. Bisectoarele exterioare a dou unghiuri a unui triunghi sunt concurente
cu bisectoarea interioar a celui de-al treilea unghi ntr-un punct I
a
(centrul cercului
exnscris).
Demonstraie. Cu teorema bisectoarei interioare pentru AA' obinem:
AC
AB
C A'
B A'
=
Cu teorema bisectoarei exterioare pentru BB' i CC' obinem:
BA
BC
A B'
C B'
= i
CB
CA
B C'
A C'
=
nmulind membru cu membru cele trei relaii de mai sus obinem:
A
C'
B'
J
B A' C
A
B C
A'
Im J
B'
C'
99
1
CB
CA
BA
BC
AC
AB
B C'
A C'
A B'
C B'
C A'
B A'
= =
de unde conform reciprocei teoremei lui Ceva rezult c cele dou bisectoare
exterioare i bisectoarea interioar sunt concurente n I
a
.
1.13.4. nlimile unui triunghi sunt concurente n punctul H (ortocentrul
triunghiului).
Demonstraie. Fie AA', BB', CC' nlimile triunghiului ABC. Din asemnarea
triunghiurilor A'AB i C'CB obinem:
BC
AB
B C'
B A'
= (1)
Din asemnarea triunghiurilor dreptunghice BB'C i AA'C obinem:
AC
BC
C A'
C B'
= (2)
Din asemnarea triunghiurilor dreptunghice C'CA i B'BA obinem:
AB
AC
A B'
A C'
= (3)
Din relaiile (1), (2) i (3) prin nmulire membru cu membru obinem:
1
AB
AC
AC
BC
BC
AB
A B'
A C'
C A'
C B'
B C'
B A'
= = ,
care se mai scrie:
1
B C'
A C'
A B'
C B'
C A'
B A'
=
i conform reciprocei teoremei lui Ceva, nlimile AA', BB', CC' sunt concurente.


1.14. Lema lui Carnot i aplicaii

Teorema 1.14.1 (Lema lui Carnot). Fie triunghiul ABC i punctele A'BC,
B'CA, C'AB. Perpendicularele n A', B', C' pe BC, CA, respectiv AB sunt
concurente dac i numai dac
0 B C' A C' A B' C B' C A' B A'
2 2 2 2 2 2
= + + (*)
A
C'
B'
B A' C
H
100

Demonstraie. Presupunem c cele trei perpendiculare sunt concurente ntr-un
punct M. Aplicm teorema lui Pitagora n triunghiul dreptunghice MBA', MCA',
MCB', AB'M, AMC', BMC' i obinem relaiile:
2 2 2
MA' BM B A' = ,
2 2 2
MA' CM C A' =
2 2 2
MB' CM C B' = ,
2 2 2
MB' AM A B' =
2 2 2
MC' AM A C' = ,
2 2 2
MC' BM B C' =
Scznd a doua relaie din prima obinem:
2 2 2 2
CM BM C A' B A' = (1)
Scznd a patra relaie din a treia obinem
2 2 2 2
AM CM A B' C B' = (2)
Scznd a asea relaie din a cincea obinem
2 2 2 2
BM AM B C' A C' = (3)
Adunnd membru cu membru relaiile (1), (2), (3) obinem relaia (*).
Reciproc, presupunem c are loc relaia din enun i prin reducere la absurd
presupunem c cele trei perpendiculare nu sunt concurente. Fie M punctul de
intersecie al perpendicularelor din B' i C' pe laturile corespunztoare i A'' piciorul
perpendicularei din M pe BC.
Din ipotez avem:
0 B C' A C' A B' C B' C A' B A'
2 2 2 2 2 2
= + +
Cu teorema direct avem
0 B C' A C' A B' C B' C ' A' B ' A'
2 2 2 2 2 2
= + +
Scznd relaiile de mai sus obinem:
= = +
2 2 2 2 2 2 2 2
C ' A' B ' A' C A' B A' 0 C ' A' C A' B ' A' B A'
A
C'
B'
M
B A' C
A
C'
B A' A'' C
M
B'
101
+ = + C) ' A' B ' C)(A' ' A' B ' (A' C) A' B C)(A' A' B (A'
= BC C) ' A' B ' (A' BC C) A' B (A'
= = ' A' A' ' A' A' C ' A' C A' B ' A' B A'
' A' A' 0 ' A' A' = = .
Observaie. Relaia (*) poate fi utilizat n rezolvarea unor probleme care cer
demonstrarea concurenei unor drepte.
Corolar 1.14.1. Pentru cazul cnd MA', MB', MC' sunt mediatoarele laturilor
concurena este evident i are loc relaia (*).

1.15. Teorema triunghiurilor ortologice

Teorema 1.15.1. Fie ABC i A'B'C' dou triunghiuri situate n acelai plan. S
se demonstreze c dac perpendicularele din A, B, C pe laturile [B'C'], [C'A'], [A'B']
sunt concurente atunci i perpendicularele din A', B', C' pe laturile [BC], [CA], [AB]
sunt concurente. (Cele dou triunghiuri se numesc ortologice.)
Demonstraie. Fie A
1
, B
1
, C
1
picioarele perpendicularelor coborte din A, B, C
pe laturile [B'C'], [C'A'], [A'B'] i
'
1
'
1
'
1
C , B , A picioarele perpendicularelor coborte din
A', B', C' pe [BC], [CA], [AB].
Cu lema lui Carnot avem:
0 B' C A' C A' B C' B C' A B' A
2
1
2
1
2
1
2
1
2
1
2
1
= + + (1*)
Cu teorema lui Pitagora n triunghiurile dreptunghice AB'A
1
i AC'A
1
obinem
2
1
2 2
1
AA AB' B' A = i
2
1
2 2
1
AA AC' C' A =
de unde prin scdere membru cu membru devine
2 2 2
1
2
1
AC' AB' C' A B' A = (1)
n triunghiurile dreptunghice BB
1
C' i BB
1
A', cu teorema lui Pitagora obinem:
2
1
2 2
1
BB BC' C' B = i
2
1
2 2
1
BB BA' A' B =
de unde prin scdere obinem:
A
B'
A1
C'1
C'
B'1
O
B1
O'
C1
C A1 B
A'
102
2 2 2
1
2
1
BA' BC' A' B C' B = (2)
Din triunghiurile A'CC
1
i B'CC
1
obinem:
2
1
2 2
1
CC CA' A' C = i
2
1
2 2
1
' ' CC C B B C =
de unde prin scdere rezult c
2 2 2
1
2
1
CB' CA' B' C A' C = (3)
Din relaiile (1*), (1), (2), (3) obinem prin adunare
0 CB' CA' BA' BC' AC' AB'
2 2 2 2 2 2
= + + (4)
ns
2 2 2 '
1
2 '
1
C A' B A' C A B A = (5)
2 2 2 '
1
2 '
1
' ' A B C B A B C B = ,
2 2 2 '
1
2 '
1
B C' A C' B C A C =
Din relaiile (4) i (5) rezult c
0 B C A C A B C B C A B A
2 '
1
2 '
1
2
1
2 '
1
2 '
1
2 '
1
= + +
deci perpendicularele din A', B', C' pe [BC], [CA], [AB] sunt concurente.
Aplicaie. Fie ABC un triunghi cu ortocentrul H i punctele A', B', C' situate
respectiv pe dreptele AH, BH, CH. S se arate c perpendicularele din A, B, C pe B'C',
C'A respectiv A'B' sunt concurente.
(Vasile Pop, 1998, Olimpiada jud. Cluj)
Demonstraie. Fiindc perpendicularele din A', B', C' pe BC, CA, AB sunt
concurente n H, cu teorema triunghiurilor ortologice obinem c i perpendicularele
din A, B, C pe B'C', C'A', A'B' sunt concurente.


1.16. Teorema ortopolului

Teorema 1.16.1. Fie ABC un triunghi i d o dreapt oarecare. Proiectm
vrfurile A, B, C pe d n punctele A', B', C'. Perpendicularele coborte din A', B', C' pe
laturile [BC], [CA], [AB] sunt concurente, iar punctul lor de intersecie poart
numele de ortopolul dreptei d fa de triunghiul ABC.
Demonstraie. Fie A
1
, B
1
, C
1
picioarele perpendicularelor coborte din A', B',
C' pe laturile triunghiului.
B C
B'
C'
A'
A
103
Cu teorema lui Pitagora n triunghiurile A'A
1
B i A'A
1
C obinem:
2
1
2 2
1
A A' B A' B A = ,
2
1
2 2
1
A A' C A' C A =
Din aceste dou relaii, prin scdere obinem:
2 2 2
1
2
1
C A' B A' C A B A =
care se mai scrie:
2 2 2 2 2
1
2
1
C C' C' A' B B' B' A' C A B A + = (1)
Analog obinem:
2 2 2 2 2 2 2
1
2
1
AA' A' B' CC' C' B' A B' C B' A B C B + = = (2)
2 2 2 2 2 2 2
1
2
1
BB' B' C' AA' A' C' B C' A C' B C A C + = = (3)
Din (1), (2) i (3) prin adunare obinem:
= + + 0 B C A C A B C B C A B A
2
1
2
1
2
1
2
1
2
1
2
1

perpendicularele n A
1
, B
1
, C
1
pe [BC], [CA], [AB] sunt concurente.


1.17. Teorema sinusurilor

Teorema 1.17.1. (Teorema sinusurilor) ntr-un triunghi, raportul dintre o
latur i sinusul unghiului opus este egal cu diametrul cercului circumscris triunghiului
sau
R
c b a
2
C sin B sin A sin
= = = (*)
unde a, b, c sunt lungimile laturilor triunghiului ABC iar R raza cercului circumscris
triunghiului.
Demonstraie. i) Considerm cazul triunghiului ascuitunghic.
Fie O centrul cercului circumscris triunghiului i A' punctul diametral opus lui A.
B' A'
C'
d
C
A1
B
C1
B1
A

104
n triunghiul dreptunghic ACA' (m(ACA')=90) avem:
R
b
2 AA'
AC
' A sin = = ,
dar AA'CABC (subntind acelai arc).
Atunci obinem:
R
b
2
B sin = ,
de unde R
b
2
B sin
= .
Analog obinem: R
a
2
A sin
= , R
c
2
C sin
= .
Deci R
c b a
2
C sin B sin A sin
= = = .
ii) Considerm cazul triunghiului obtuzunghic. (m(A)>90).
n acest caz punctul O, centrul cercului circumscris este situat n exteriorul
triunghiului.

Ducem diametrul BB', atunci m(BCB')=90. n triunghiul BCB' avem
R
a
2 BB'
BC
' B sin = = (1)
A
B
A'
C
O
A
B C
B'
O
105
Patrulaterul ACB'B fiind inscriptibil rezult c
= + 180 C) BB' m( BAC) m( ,
de unde m(BB'C)=180-m(BAC). Atunci obinem:
sin(BB'C)=sin(180-A)=sinA (2)
Deci cu (1) i (2) obinem
2R
a
A sin = , rezult c R
a
2
A sin
= . Celelalte
relaii se demonstreaz ca n cazul precedent. Rezult c i n acest caz sunt satisfcute
relaiile (*).


1.18. Teorema cosinusului

Teorema 1.18.1. Considerm un triunghi oarecare ABC. Vom demonstra mai
nti c ntre elementele triunghiului avem relaia:
B cos C cos c b a + = (1)
Mai nti presupunem c unghiurile B i C sunt ascuite i deci piciorul A' al
nlimii coborte din A este situat ntre punctele B i C.
Din triunghiurile dreptunghice AA'B i AA'C obinem:
AB
BA'
B cos = , de unde BA'=ccosB
AC
C A'
C cos = , de unde A'C=bcosC.
Dar BA'+A'C=a, deci ccosB+bcosC=a, adic tocmai relaia (1).
Dac unghiul B este obtuz, piciorul A' al nlimii din A este situat n
exteriorul segmentului [BC].
Din triunghiurile dreptunghice AA'B i AA'C obinem:
AB
B A'
ABA' cos = ,
de unde A'B=ABcos(180-B), sau A'B=ccos(180-B), iar
AC
C A'
C cos = , de unde
A'C=bcosC.
Dar cos(180-B)=-cosB i a=A'C-A'B. Atunci a=bcosC+ccosB, adic relaia
(1).
Analog se demonstreaz c
A
B A' C
A
A' B C
106
b=ccosA+acosC (2)
c=acosB+bcosA (3)
nmulind relaia (2) cu b i (3) cu c, adunnd membru cu membru obinem:
A cos 2 ) B cos C cos (
2 2
bc c b a c b + + = + (4)
innd seama de (1), relaia (4) devine:
A cos 2
2 2 2
bc a c b + = +
adic
A cos 2
2 2 2
bc c b a + = (5)
relaie care poart numele de teorema cosinusului:
Ptratul lungimii unei laturi a unui triunghi este egal cu suma ptratelor
lungimilor celorlalte dou laturi minus de dou ori produsul lungimilor lor nmulit cu
cosinusul unghiului dintre ele.
Analog cu (5) avem
B cos 2
2 2 2
ac c a b + = (6)
C cos 2
2 2 2
ab b a c + = (7)
Din (5), (6), (7) obinem:
ab
c b a
C
ac
b c a
B
bc
a c b
A
2
cos ,
2
cos ,
2
cos
2 2 2 2 2 2 2 2
+
=
+
=
+
= (8)
Relaiile (8) constituie o modalitate pentru determinarea naturii unui triunghi
(ascuitunghic, dreptunghic sau obtuzunghic).



107
2. Inegaliti geometrice

La concursurile de matematic, n revistele de matematic apar probleme de
inegaliti geometrice. n manualele colare sunt foarte puine probleme de acest tip. n
aceast tem, vom face o ordonare a tipurilor de inegaliti geometrice, la nivel
gimnazial.

2.1. Unghi exterior. Relaii ntre laturile i unghiurile unui triunghi

ntre un unghi exterior unui triunghi i unghiurile interioare neadiacente lui,
exist o relaie de inegalitate dat de teorema urmtoare.

Teorema 2.1.1. Msura unui unghi exterior unui triunghi este mai mare dect msura
oricrui unghi interior triunghiului, neadiacent lui.

Demonstraie . Fie M mijlocul lui AC i
BM N astfel nct MN BM
(fig.1). Deoarece
CNM ABM .) . . ( L U L ,
rezult c N C M B A M

= . Dar
)

( )

( )

( X C N m N C M m X C A m + = ,
deci )

( )

( B A M m X C A m > . Analog se
arat c )

( )

( C B A m X C A m > .

Consecina 2.1.1. Suma msurilor oricror dou unghiuri ale unui triunghi este mai
mic dect
o
180 .

Demonstraie. innd seama de teorema 3.1.1., avem

o
180 )

( )

( )

( )

( = + < + B C A m X C A m B C A m C A B m .

Consecina 2.1.2. Un triunghi poate avea cel mult un unghi drept sau cel mult un
unghi obtuz.

Demonstraie . n caz contrar , suma msurilor unghiurilor triunghiului este strict mai
mare dect
o
180 .

Teorema 2.1.2. ntr-un triunghi, laturii mai mari i se opune unghiul mai mare i
reciproc.


108
Demonstraie . Fie triunghiul ABC ,
AC AB < i [ ] AC M astfel nct
AD AB (fig.2). Atunci
triunghiul ABM este isoscel, deci
)

( )

( B M A m M B A m = . innd
seama de teorema 2.1.1., rezult c
)

( )

( B C A m B M A m > , deci
)

( )

( B C A m M B A m > i cum
)

( )

( M B A m C B A m > , n final
obinem c )

( )

( B C A m C B A m > .
Reciproc, presupunem c )

( )

( B C A m C B A m > i artm c AB AC > . Demonstrm


prin reducere la absurd, adic presupunem c AB AC < sau AB AC . Dac
AB AC < , conform teoremei directe ar rezulta c )

( )

( B C A m C B A m < , ceea ce este


o contradicie. Dac AB AC , rezult c triunghiul ABC este isoscel, deci
)

( )

( B C A m C B A m = , ceea ce este o contradicie. n concluzie, rezult c AB AC > .



Consecina 2.1.3. ntr-un triunghi dreptunghic, lungimea ipotenuzei este mai mare
dect lungimea oricrei catete.
Demonstraie . Rezult din teorema 2.1.2 i consecina 2.1.2.


Fiind dat o dreapt d, un punct
d A , d AB , d B , d C ,
C B (fig.3), punctul B se numete
piciorul perpendicularei din A pe
dreapta d, AC se numete oblic, iar
C se numete piciorul oblicei AC .





Consecina 2.1.4. Fie o dreapt d i un punct A, d A . Dintre dou oblice cu
picioarele pe d, inegal deprtate de piciorul perpendicularei din A pe d , oblica cu
piciorul mai ndeprtat de piciorul perpendicularei din A pe d are lungimea mai mare.


109
Demonstraie . Fie d AB , d B , d D C , distincte i diferite de B ,
BD BC < (fig.4). Din triunghiul
dreptunghic ABC rezult c unghiul
B C A

este ascuit, deci unghiul


D C A

este obtuz. Atunci


)

( )

( C D A m D C A m > , de unde,
conform teoremei 2.1.2 rezult c
AC AD > . Cazul oblicelor situate
de o parte i de alta a perpendicularei
din A se reduce la cazul de mai sus,
construind simetrica unei oblice fa
de perpendiculara din A.

2.2. Relaii de inegalitate ntre laturile unui triunghi

n acest paragraf vom da condiia necesar i suficient ca trei segmente s
poat forma un triunghi.
Teorema 2.2.1. ntr-un triunghi,
lungimea oricrei laturi este strict mai
mic dect suma lungimilor celorlalte
dou laturi.

Demonstraie . Fie triunghiul ABC .
Pe prelungirea laturii AB , construim
AC AD (fig.5).
n triunghiul isoscel ADC, avem c
D C A C D A

, deci
)

( )

( D C B m C D A m < .
Conform teoremei 2.1.2 , rezult c
BD BC < . Dar AD BA BD + = i
cum AC AD , obinem c AC BA BC + < . Analog se arat c BA CB CA + < i
CB AC AB + < .

Teorema 2.2.2. Trei numere strict pozitive pot fi lungimile laturilor unui triunghi
dac oricare dintre ele este strict mai mic dect suma celorlalte dou.

Demonstraie . Rezult din construcia triunghiului cnd se dau lungimile laturilor lui.

Consecina 2.2.1. Trei numere strict pozitive pot fi lungimile laturilor unui triunghi
dac i numai dac oricare dintre ele este strict mai mic dect suma celorlalte dou.

Demonstraie . Rezult din teorema 2.2.1. i teorema 2.2.2.

110
Teorema 2.2.3. Trei numere strict pozitive pot fi lungimile laturilor unui triunghi dac
i numai dac oricare dintre ele este strict mai mare dect modulul diferenei celorlalte
dou.

Demonstraie . Notm cu c b a , , lungimile celor trei segmente. Conform consecinei
2.2.1 avem c

> +
> +
b c a
c b a
, de unde

>
>
c b a
b c a
, sau c b a > . Analog se arat i
celelalte inegaliti. Reciproc, din c b a > se obine

>
>
c b a
b c a
,sau

> +
> +
b c a
c b a
.
Folosind i celelalte inegaliti, n final obinem c orice numr este strict mai mic
dect suma celorlalte dou.

Observaia 2.2.1. Dac trei puncte C B A , , sunt coliniare, spunem c triunghiul
ABC este degenerat. ntr-un triunghi degenerat, exact una din cele trei inegaliti
devine egalitate.

2.3. Triunghiuri care au dou laturi respectiv congruente i unghiurile cuprinse
ntre ele necongruente

Dac dou triunghiuri au dou laturi respectiv congruente i unghiurile dintre
laturile respectiv congruente sunt congruente, atunci triunghiurile sunt congruente
(cazul L.U.L.). Dac unghiurile dintre laturile respectiv congruente sunt necongruente,
atunci cele dou triunghiuri sunt necongruente.

Teorema 2.3.1. Fie triunghiurile ABC i C B A astfel nct B A AB i
C A AC . Atunci )

( )

( C A B m C A B m > C B BC > .

Demonstraie . Considerm c )

( )

( C A B m C A B m > . Construim punctul D astfel


nct C i D s fie de aceeai parte a lui AB i C B A ABD (fig.6). Fie AM
bisectoarea unghiului DAC ,
BC M .Deoarece
AMC AMD (cazul
L.U.L.), rezult c
MC MD . n triunghiul
BDM avem c
BD MD BM > + , sau
C B MC BM > + , adic
C B BC > .
Reciproc, considerm c
C B BC > i vom arta c

111
)

( )

( C A B m C A B m > . Vom demonstra aceast afirmaie prin metoda reducerii la


absurd. Negnd concluzia, avem c )

( )

( C A B m C A B m > ceea ce implic, conform


teoremei directe, c C B BC < , contradicie cu ipoteza, sau )

( )

( C A B m C A B m = .
n aceast situaie, conform cazului L.U.L. avem c C B A ABC , de unde
C B BC , ceea ce este n contradicie cu ipoteza. n concluzie, avem c
)

( )

( C A B m C A B m > .

2.4. Probleme

2.4.1. Inegaliti generale

Probleme rezolvate
R2.4.1.1. S se arate c pentru orice punct M din interiorul triunghiului ABC , au
loc inegalitile
a) AC AB MC MB + < + ;
b) p MC MB MA p 2 < + + < .
Soluie . Fie { } B AC BM = (fig.7). n triunghiurile B AB i C B M avem
B A AB B B + < sau
B A AB B M MB + < + , respectiv
C B B M MC + < . Adunnd cele dou
inegaliti se obine inegalitatea de la a).
n triunghiurile MCA MBC, i MAB avem
MC MB BC + < , MA MC CA + < ,
respectiv MB MA AB + < . Prin adunare se
obine MC MB MA p + + < . innd seama
de a) avem AC AB MC MB + < + ,
BA BC MA MC + < + i CB CA MB MA + < + , de unde rezult c
p MC MB MA 2 < + + .
R2.4.1.2. Fie triunghiul ABC i D mijlocul laturii BC . S se arate c
)

( )

( D A C m D A B m > dac i numai dac AB AC > .



Soluie . Fie E simetricul lui A fa de D(fig.8) i atunci ABEC este paralelogram.
Deoarece )

( )

( D A C m D A B m > i cum
)

( )

( D E C m D A B m = , rezult c
)

( )

( D A C m D E C m > . Atunci, n triunghiul


AEC avem c EC AC > i cum AB EC ,
rezult c AB AC > . Reciproc, dac

112
AB AC > i cum CE AB , rezult c CE AC > .
Atunci, n triunghiul ACE avem c )

( )

( D A C m C E A m > . Dar
)

( )

( D A B m C E A m = , deci )

( )

( D A C m D A B m > .


R2.4.1.3. Fie M un punct arbitrar n
interiorul triunghiului echilateral
ABC . S se arate c distanele
MC MB MA , , pot fi lungimile
laturilor unui triunghi (teorema lui
Pompeiu).

Soluie . Facem o rotaie de centru A
a triunghiului ABC i obinem
triunghiul C AC , iar punctul M se
transform n punctul M (fig.9).
Avem c M A AM , M C BM , M C CM i deoarece
o
60 )

( = M A M m ,
rezult c triunghiul M AM este echilateral. Prin aceast rotaie, se obine triunghiul
M CM , n care AM M M i BM M C , deci distanele MC MB MA , , pot fi
lungimile laturilor unui triunghi.


2.4.2. Inegaliti ntre laturile unui triunghi

Probleme rezolvate

R2.4.2.1. Fie c b a , , lungimile laturilor unui triunghi. S se arate c
) ( 4 4 ) ( 3
2
ca bc ab p ca bc ab + + < + + , unde p este semiperimetrul triunghiului.
F.E.Wood
Soluie . Prima inegalitate
2
) ( ) ( 3 c b a ca bc ab + + + + are loc 0 , , > c b a , iar
egalitatea are loc dac i numai dac c b a = = . Dac c b a , , sunt lungimile laturilor
unui triunghi, atunci a c b < , b a c < i c b a < . Prin ridicare la ptrat
2 2
) ( a c b < ,
2 2
) ( b a c < i
2 2
) ( c b a < , de unde, prin adunare se obine a
doua inegalitate.

R2.4.2.2. Dac c b a , , sunt lungimile laturilor unui triunghi, atunci
p c p b p a p
9 1 1 1

.

113
Soluie . Deoarece c b a , , sunt lungimile laturilor unui triunghi, rezult c
0
2
>
+
=
a c b
a p i analog 0 > b p , 0 > c p . Notm x a p = ,
y b p = , z c p = , unde 0 , , > z y x . Atunci z y a + = , x z b + = , y x c + = i
inegalitatea din enun devine
z y x z y x + +
+ +
9 1 1 1
, care este o inegalitate
adevrat. Egalitatea are loc dac i numai dac z y x = = , echivalent cu c b a = = ,
adic triunghiul este echilateral.

R2.4.2.3. S se arate c dac c b a , , sunt lungimile laturilor unui triunghi, atunci cu
segmentele de lungime c b a , , se poate construi un triunghi.

Soluie. Pentru ca s existe un triunghi cu lungimile laturilor c b a , , este necesar
i suficient ca c b a + < , a c b + < i b a c + < . Artm c
c b a + < . Avem ( )
2
bc 2 c b c b c b a + = + + < + < , de unde
c b a + < . Analog se demonstreaz celelalte inegaliti.


2.4.3 Inegaliti ntre elementele unui triunghi

n cele ce urmeaz vom folosi notaiile consacrate n triunghi.

Probleme rezolvate

R2.4.3.1. Fie triunghiul ABC , ( ) BC D , k
DC
DB
= . Atunci
AB
k
AC
k
k
AD
+
+
+
<
1
1
1
.

Soluie . Construim AB CE , AD E
(fig.10). Din asemnarea triunghiurilor ABD
i ECD avem
c
ED
AD
CD
BD
EC
AB
= = , de unde AB
k
EC
1
=
i AD
k
ED
1
= . n triunghiul ACE avem
CE AC AE + < , sau

114
CE AC DE AD + < + . nlocuind cu relaiile de mai sus, obinem
AB
k
AC AD
k
AD
1 1
+ < + , de unde rezult inegalitatea din enun.

R2.4.3.2. n triunghiul ABC s se arate c are loc inegalitatea
o o
90 60 <
+ +
+ +

c b a
C c B b A a


Soluie . Presupunem c c b a i atunci C B A . Avem c
( )( ) 0 B A b a , ( )( ) 0 C B c b i ( )( ) 0 A C a c . Adunnd cele trei
inegaliti avem ( ) + + cC bB aA 2
( ) ( ) ( ) C b a B a c A c b + + + + + i adunnd n ambii membri cC bB aA + + , se
obine ( ) ( )( ) C B A c b a cC bB aA + + + + + + 3 , de unde rezult prima
inegalitate.Egalitatea are loc dac i numai dac triunghiul ABC este echilateral. Din
inegalitatea triunghiului avem c a c b a 2 > + + , b c b a 2 > + + i c c b a 2 > + + .
nmulind cu B A, respectiv C i adunnd obinem
( )( ) ( ) cC bB aA C B A c b a + + > + + + + 2 , de unde rezult a doua inegalitate din
enun.

R2.4.3.3. ntr-un triunghi are loc r R 2 (inegalitatea lui Euler).

Soluie . Notm 0 > = + x a c b , 0 > = + y b a c , 0 > = + z c b a i atunci
2
z y
a
+
= ,
2
x z
b
+
= i
2
y x
c
+
= . Cu aceste notaii avem
( )( )( ) ( ) xyz z y x c p b p a p p S + + = =
4
1
,
z y x
xyz
P
S
r
+ +
= =
2
1
i
( )( )( )
( ) xyz z y x
x z z y y x
S
abc
R
+ +
+ + +
= =
8
4
. Atunci
( )( )( )
xyz
x z z y y x
r
R
8 2
+ + +
= i cum
xy y x 2 + , yz z y 2 + , zx x z 2 + , rezult inegalitatea din enun.
Egalitatea are loc dac i numai dac z y x = = , echivalent cu c b a = = , adic
triunghiul ABC este echilateral.



115
3. Locuri geometrice

3.1. Locuri geometrice uzuale

Noiunea de loc geometric n plan care se gsete i n ELEMENTELE LUI
EUCLID se pare c a fost folosit nc de PLATON (427-347) i ARISTOTEL(383-
322). Locurile geometrice reprezint unul din cele mai frumoase capitole ale
geometriei.
Definiia locului geometric poate fi gsit n mai multe formulri:
a) loc geometric este totalitatea punctelor dintr-un spaiu definite printr-o
proprietate (Dicionarul explicativ al limbii romne);
b) loc geometric este mulimea punctelor din plan sau spaiu care au o anumit
proprietate (Micul dicionar enciclopedic);
c) loc geometric este figura plan sau n spaiu ale crei puncte se definesc toate
prin aceeai proprietate (Dicionar de neologisme).
Toate aceste formulri au acelai sens: un loc geometric este o mulime de puncte
DEFINITE. n esen, problemele de loc geometric sunt probleme de gsire a unor
proprieti echivalente celor prin care este dat o anumit mulime sau altfel spus,
probleme de egalitate a dou mulimi.
n continuare dm o list care conine locuri geometrice uzuale, care pot oferi idei
i soluii n rezolvarea altor probleme de loc geometric:
3.1.1 Locul geometric al punctelor egal deprtate de extremitile unui
segment este mediatoarea acelui segment.
3.1.2 Locul geometric al punctelor din plan interioare unui unghi egal
deprtate de laturile sale este bisectoarea.
3.1.3 Locul geometric al punctelor din plan egal deprtate de dou drepte
concurente sunt bisectoarele unghiurilor formate de cele dou drepte
(bisectoarele sunt perpendiculare n punctul de intersecie al celor dou drepte).
3.1.4 Locul geometric al punctelor din plan situate la o distan dat fa de o
dreapt este reprezentat de dou drepte paralele cu o dreapt dat, situate de o parte i
de alta a ei.
3.1.5 Locul geometric al punctelor din plan situate la o distan dat fa de
un punct fix este un cerc.
3.1.6 Locul geometric al punctelor din plan egal deprtate de trei puncte
distincte, necoliniare este reprezentat de centrul cercului circumscris triunghiului
determinat de cele trei puncte.
3.1.7 Locul geometric al punctelor din plan egal deprtate de dou drepte
paralele date este o dreapt paralel cu dreptele date i situat la jumtatea distanei
dintre ele.
3.1.8 Locul geometric al punctelor din plan pentru care diferena ptratelor la
dou puncte fixe este constant, este o dreapt perpendicular pe dreapta determinat
de cele dou puncte fixe.

116
3.1.9 Locul geometric al punctelor din plan pentru care raportul distanelor la
dou drepte paralele este constant, este reprezentat de dou drepte paralele cu dreptele
date sau o dreapt (dac raportul este 1).
3.1.10 Locul geometric al punctelor din plan pentru care suma ptratelor
distanelor la dou puncte date este constant, este un cerc cu centrul n mijlocul
segmentului determinat de dou puncte.
3.1.11 Locul geometric al punctelor din plan din care un segment se vede
sub un unghi drept este cercul care are ca diametru segmentul respectiv.
3.1.12 Locul geometric al punctelor din plan, mijloace ale segmentelor
paralele cu o direcie dat i cuprinse ntre dou drepte paralele fixe, este dreapta
paralel cu dreptele date i egal deprtate de ele.
3.1.13 Locul geometric al punctelor din plan din care un segment se vede
sub un unghi dat este reprezentat de dou arce de cerc care au aceleai extremiti ca i
segmentul i sunt simetrice fa de dreapta pe care este situat segmentul.
3.1.14 Locul geometric al punctelor din plan care mpart ntr-un raport
constant segmentele determinate de un punct fix A i punctul M ce descrie o dreapt
dat (d) este o dreapt paralel cu (d) i care mparte distana de la A la (d) n acelai
raport.
3.1.15 Locul geometric al punctelor din plan pentru care raportul distanelor
la dou puncte fixe este constant (k 1) este un cerc (pentru raportul distanelor k=1 se
obine o dreapt).
3.1.16 Locul geometric al punctelor N din plan, situate pe segmentele care
unesc un punct fix A cu un punct M ce descrie o dreapt (d) dat, astfel nct AN x
AM=K este un cerc care trece prin A i are centrul pe perpendiculara dus din A pe (d).
3.1.17 Locul geometric al punctelor din plan care au puteri egale fa de
dou cercuri date este o dreapt (numit axa radical a celor dou cercuri)
perpendicular pe linia centrelor cercurilor.
3.1.18 Locul geometric al punctelor din plan de putere constant fa de un
cerc dat, este un cerc concentric cu cercul dat, un punct sau mulimea vid.


Probleme rezolvate

R3.2.1 Dou cercuri i ' sunt tangente exterioare ntr-un punct A. Fie
TT una din tangentele comune exterioare i M, M interseciile celor dou cercuri cu o
dreapt variabil ce trece prin A.
S se afle locul geometric al punctelor P de intersecie a lui MT cu MT.


117
Fig. 3.1.
Soluie. 1) Din T T OO
TT OT
TT OT
' '
' '
'

)
`

trapez dreptunghic
( ) ( )
0
180 ' ' = + A O T m TOA m ( ) ( )
0
180 ' = + A T m TA m
Dar ( ) ( ) ( ) ( ) ( ) = = + = +
0 0
90 180
2
1
'
2
1
' ' A T m TA m A M T m TMA m
( )
0
90 ' = MPM m , deci ' PTT este un triunghi dreptunghic cu ipotenuza
constant TT.
Rezult c locul geometric este cercul de diametrul TT.


R3.2.2 O proprietate simpl a punctelor mediane AA a ABC este
urmtoarea:
) ( ) (
) ' (
ACM ABM
S S AA M =
Se pune problema dac singurele puncte M din plan cu proprietatea
) ( ) ( ACM ABM
S S = sunt punctele medianei din A ?
Pentru aceasta ajungem la urmtoarea problem de loc geometric:
S se determine locul geometric al punctelor M din planul ABC pentru care
) ( ) ( ACM ABM
S S = .

Soluie. 1. Din proprietatea specificat anterior rezult c AA aparine locului
geometric.
2. Artm c n interiorul CAB nu exist alte puncte ale locului
geometric.
- presupunem c ar exista M , ' AA M


118
( )
( )
" * ' *
2
" *
2
' *
MM AC MM AB
MM AC
S
MM AB
S
MAC
MAB
=

=
=


din
1
M avem
( ) ( )
" * ' *
1 1 1
1 1
M M AC M M AB S S
ACM ABM
= = ;
dar ' '
1
M M MM > i " '
1
M M MM < . Deci
( ) ( )
1 1
" * " * ' * ' *
1 1 1 ACM ABM
S S M M AC MM AC MM AB M M AB = < = <
Rmne s cutm puncte ale locului n exteriorul acestui unghi.
Fie B simetricul fa de A.
Din
( ) ( ) M AB ABM
S S AB AB
'
' = = , deci relaia ce o verific punctele locului
este:
( ) ( ) ACM M AB
S S =
'
, care din nou conform proprietii medianei n AC B' este
mediana AA paralel cu BC.
Deci locul geometric este mediana din A i paralela prin A la BC.

Fig. 3.2.

R3.2.3. S se determine locul geometric al punctelor M din planul dreptelor
1
d i
2
d pentru care raportul:

( )
( )
k
d M d
d M d
=
2
1
,
,
, k>0.

Soluie. Rezolvm problema n cazurile:
{ } = O d d
2 1
i
2 1 2 1
|| , 0 d d d d =

119
Fig. 3.3.

Fie M situat n unul din cele patru unghiuri format de dreptele
1
d i
2
d .
Pe laturile acestui unghi lum
1
d A astfel ca 1 = OA i
2
d B astfel ca
k OB = , relaia ( ) ( ) k d M d d M d * , 1 * ,
2 1
=
2 1
* * MM OB MM OA =
( ) ( ) OBM OAM
S S = .
Deci am ajuns la problema determinrii locului geometric al punctelor din planul
triunghiului OAB cu
( ) ( ) OBM OAM
S S = care este format din dou drepte: mediana din O
i paralela prin O la AB.
n cazul
1
d ||
2
d difereniem cazul k=1 i cazul k 1 .
Dac k=1, pe fiecare dreapt perpendicular pe
1
d i
2
d avem un singur punct n
loc (mijlocul segmentului determinat de intersecia ei cu cele dou drepte).
Deci locul va fi o dreapt paralel la
1
d i
2
d egal deprtat de cele dou drepte.
Dac 1 k pe fiecare dreapt perpendicular pe
1
d i
2
d se obin cte dou
puncte, unul ntre punctele de intersecie i unul n afar, situat la distan determinat.
Deci locul geometric n acest caz va fi format din dou drepte paralele la
1
d i
2
d .

R3.2.4 Dou puncte mobile M i N se mic rectiliniu i uniform. S se
determine locul geometric al punctelor P | | MN astfel ca k
NP
MP
= (constant).



120
Fig. 3.4.

Soluie. Vom arta c locul geometric este o dreapt. Pentru aceasta este
suficient s artm c dac
0
P i
1
P sunt dou poziii ale lui P , orice alt poziie este
coliniar cu
0
P i
1
P .
Rapoartele
1
0
NN
N N
i
1
0
MM
M M
nu depind de vitezele
M
v i
N
v , ci doar de
intervalul de timp, deci

|
|
.
|

\
|
= = = =
2
1
2 2
1 2
2 1
1 1
1
0
1
0
*
t
t
t v
t v
t v
t v
x
MM
M M
NN
N N
;

Dac { } MN P P P =
1 0
, pentru a arta c P=P este suficient s artm c
k
N P
MP
=
'
'
.
Fie
'
0
N ,
'
0
M , N
'
, M
'
,
'
1
N ,
'
1
M , formnd proieciile pe dreapta
1 0
P P ale
punctelor
0
N ,
0
M , N, M,
1
N ,
1
M , avem :
'
'
'
'
NN
MN
N P
MP
= .
Din trapezul
'
0
'
1 1 0
N N N N
x
N N N N x
NN
+
+
=
1
*
'
'
0 0
'
1 1
, analog
x
M M M M x
MM
+
+
=
1
*
'
'
0 0
'
1 1


Deci = = k
P N
P M
N N
M M
o
0 0
0 0
'
0 0
'
0

=
=
'
1 1
'
1 1
'
0 0
'
0 0
*
*
N N k M M
N N k M M
. Deci .
'
'
k
N P
MP
=



121
R3.2.5 S se gseasc locul geometric al punctelor din plan pentru care
raportul distanelor la dou puncte fixe este constant.

Soluie. Fie A i B dou puncte fixe distincte. Cutm locul geometric al
punctelor M pentru care k
MB
MA
= , unde k este o constant pozitiv.
n plan pentru k=1, orice punct M pentru care 1 =
MB
MA
, aparine mediatoarei
segmentului | | AB i reciproc. De aceea n acest caz locul geometric este mediatoarea
segmentului AB.

Fig. 3.5.
Fie 1 k i M un punct care nu se afl pe dreapta AB, astfel nct k
MB
MA
= .
Bisectoarea interioar a unghiului AMB taie pe AB n C.
Deoarece 1 k implic AMB MB MA , nu este isoscel. De aceea i
bisectoarea exterioar unghiului AMB taie pe AB n D.
n proprietatea bisectoarei avem: k
DB
DA
MB
MA
CB
CA
= = = . Astfel C i D sunt
puncte fixe pe AB, care mpart segmentul | | AB n raportul k. Deoarece
( )
0
90 = CMD m rezult M aparine cercului de diametru CD.
Reciproc: fie M un punct al cercului de diametru CD, unde C i D sunt fixe pe
AB , care mparte | | AB n raportul k.
Deoarece D M C M ' ' rezult c MC i MD sunt bisectoarele unghiului M,
adic .
'
'
k
CB
CA
B M
A M
= =
Punctele C i D convin prin definiie. De aceea locul geometric al punctelor
pentru care raportul distanelor la dou puncte fixe A i B este un numr pozitiv 1 k ,
este cercul de diametru CD, punctele C, D mprind pe | | AB n raportul dat.

122
4. Patrulaterul inscriptibil i circumscriptibil

PATRULATERUL INSCRIPTIBIL

n acest paragraf se vor studia proprietile patrulaterului inscriptibil.

4.1. Arc capabil de un unghi dat

n acest paragraf vom rezolva urmtoarea problem: Se dau dou puncte fixe
diferite, A i B i un numr
0
(0,180
0
). S se determine locul geometric al punctelor
M astfel nct
0
)

( = B M A m .
Soluie. Dreapta AB mparte planul n dou semiplane S i S. Vom determina locul
geometric al punctelor M cu proprietatea din enun situate n semiplanul S. Fie
semidreapta (AX n semiplanul S astfel nct
0
)

( = B A X m . Notm cu O punctul de
intersecie dintre perpendiculara n A pe AX i mediatoarea segmentului AB.
Considerm cercul C de centru O i raz OA (fig.1).

O
N
S
S
A
X
M
B
.


Fig. 1.

Fie M un punct arbitrar al arcului B A
)
situat n semiplanul S i N un punct al arcului
B A
)
situat n semiplanul S. Deoarece AX este tangent cercului C avem c
2
) (
)

(
B N A m
B A X m
)
= . Dar
2
) (
)

(
B N A m
B M A m
)
= , deci
0
)

( )

( = = B A X m B M A m .
Dac PInt C S, atunci
0
2
) (
)

( = >
B N A m
B P A m
)
deoarece unghiul B P A

este un
unghi cu vrful n interiorul cercului C. Dac PExtCS, atunci

123
2
) (
)

(
B N A m
B P A m
)
< . Arcul B A
)
din semiplanul S, fr punctele A i B este locul
geometric al punctelor M din semiplanul S pentru care
0
)

( = B M A m i se numete
arc capabil de unghiul
0
. Locul geometric din plan este format din dou arce capabile
de unghiul
0
, unul situat n semiplanul S i cellalt n semiplanul S.
Observaia 5.1.1. Arcul deschis B A
)
situat n semiplanul S este un arc capabil de
unghiul 180
0
-
0
.

4.2 Patrulaterul inscriptibil

n acest paragraf vom defini i vom caracteriza patrulaterul inscriptibil.
Definiia 4.2.1. Un patrulater se numete nscris ntr-un cerc sau inscriptibil, dac
vrfurile patrulaterului aparin cercului.
Teorema 4.2.1. Un patrulater convex este inscriptibil dac i numai dac mediatoarele
laturilor sale sunt concurente.
Demonstraie. Fie patrulaterul convex ABCD (fig.2), cu proprietatea c mediatoarele
laturilor sale sunt concurente n O.

A
B
D
O
C
Fig. 2.

Atunci, pe baza proprietii pe care o au punctele de pe mediatoare, avem c
OA OB, OB OC, OC OD i OD OA, de unde OA OB OC OD. Deci
vrfurile patrulaterului se gsesc pe un cerc de centru O i raz R=OA, deci
patrulaterul ABCD este inscriptibil. Reciproc, dac ABCD este inscriptibil i O este
centrul cercului circumscris, atunci OA OB OC OD = R, deci O se afl pe toate
mediatoarele laturilor patrulaterului ABCD, rezult c mediatoarele sunt concurente.

Teorema 4.2.2. Un patrulater convex este inscriptibil dac i numai dac un unghi
determinat de o latur i de o diagonal este congruent cu unghiul format de latura
opus primei laturi i cealalt diagonal.

Demonstraie. Fie patrulaterul inscriptibil ABCD i C (O,R) cercul circumscris (fig.3).


124
A
B
D
M
C


Fig. 3.

Atunci avem c
2
) (
)

( )

(
C D m
C B D m C A D m
)
= = i analog pentru celelalte perechi de
unghiuri. Reciproc, dac patrulaterul este convex i dac C B D C A D

, atunci innd
seama de proprietatea arcului capabil de unghi dat i de faptul c A i B se afl n
acelai semiplan limitat de dreapta DC, rezult c punctele B i C aparin unui cerc n
care DC este coard. Deci patrulaterul ABCD este inscriptibil.

Teorema 4.2.3. Un patrulater convex este inscriptibil dac i numai dac suma a dou
unghiuri opuse ale patrulaterului este de 180
0
.

Demonstraie. Dac patrulaterul ABCD este inscriptibil i C este cercul circumscris
(fig.3), atunci
0
180
2
) (
2
) (
)

( )

( = + = +
C B A m C D A m
C D A m C B A m
) )
. Reciproc, se
ine seama de observaia 4.1.1.

Teorema 4.2.4. (Inegalitatea lui Ptolemeu) n patrulaterul convex ABCD are loc
inegalitatea ACBD ABCD+ADBC. Egalitatea are loc dac i numai dac
patrulaterul este inscriptibil.

Demonstraie. Construim triunghiul ADE asemenea cu triunghiul ABC,
ADE ABC i DAE BAC (fig. 4)


125
A B
D
E
C


Fig. 4.

Din aceast asemnare avem c
AD DE AE
AB BC AC
= = , de unde
BC AD
DE
AB

= i
AD AB
AE AC
= . innd seama de ultima relaie i de faptul c DAB EAC , rezult
c triunghiurile EAC i DAB sunt asemenea, deci
DB
EC
AB
AC
= , de
unde
AC DB
EC
AB

= . n triunghiul EDC, care poate fi i degenerat, avem c


EC ED + DC i nlocuind pe DE i EC rezult inegalitatea cerut. Egalitatea are loc
dac i numai dac punctele E, D i C sunt coliniare, adic
0
180 ) ( ) ( = + ADC m DAE m , dac i numai dac patrulaterul ABCD este
inscriptibil.

Consecina 4.2.1. (Teorema lui Pompeiu) Fie triunghiul echilateral ABC i M un
punct n planul triunghiului ce nu aparine cercului circumscris triunghiului. Atunci
distanele MA, MB, MC pot fi lungimile laturilor unui triunghi.

Demonstraie. n patrulaterul ABMC, aplicnd inegalitatea lui Ptolemeu, avem c
AMBC<CMAB+BMAC. Dar triunghiul ABC fiind echilateral avem c AM<CM+BM.
Analog obinem c BM<AM+CM i CM<AM+BM, deci AM, BM, CM pot fi lungimile
laturilor unui triunghi.

Teorema 4.2.5. (Prima teorem a lui Ptolemeu) Fie un patrulater convex ABCD.
Urmtoarele afirmaii sunt echivalente:
(i) Patrulaterul ABCD este inscriptibil;
(ii) ACBD=ABCD+ADBC.

Demonstraie. Rezult din teorema 5.2.4.

Teorema 4.2.6. (A doua teorem a lui Ptolemeu) n patrulaterul inscriptibil ABCD
are loc relaia:

126

AC AB AD CB CD
BD BA BC DA DC
+
=
+


Demonstraie. Notm ACBD= {M}(fig. 3). Din asemnarea triunghiurilor ABM i
DCM avem
MB MA AB
MC MD DC
= = , de unde
MA MD
AB AD DA DC
=

i
MB MC
BA BC CD CB
=

.
Analog, din asemnarea triunghiurilor ADM i BCM avem
MA MB
AD AB BC BA
=

.
Deci
DC DA
MD
CB CD
MC
BC BA
MB
AD AB
MA

, sau
DC DA BC BA
MD MB
CB CD AD AB
MC MA
+
+
=
+
+
, de unde rezult egalitatea ce trebuia
demonstrat.

Observaia 4.2.1. Se poate arta c dac ntr-un patrulater convex are loc relaia din
teorema 4.2.6, atunci patrulaterul este inscriptibil.

Teorema 4.2.7. (Schooten) Dac M este un punct situat pe arcul C B
)
(arcul care
nu-l conine pe A) al cercului circumscris triunghiului echilateral ABC, atunci
AM=BM+CM.

Demonstraie. Aplicnd prima teorem a lui Ptolemeu n patrulaterul inscriptibil
ABMC (fig. 5), avem AMBC=BMAC+CMAB. innd seama c triunghiul ABC este
echilateral, se obine identitatea cerut.

A
B
M
C


Fig. 5.

Lema 4.2.1. (Brahmagupta) Fie triunghiul ABC, R raza cercului circumscris
triunghiului. Dac AA este nlimea din A, ABC, atunci ABAC=2RAA.

127

Demonstraie. n cercul circumscris triunghiului ABC, fie A
1
punctul diametral opus
lui A (fig. 6). Din asemnarea triunghiurilor ABA i AA
1
C avem
1
' AB AA
AA AC
= , de
unde, innd seama c AA
1
=2R, obinem identitatea din enun.


A
B
O
A
A
1
C
.

A
M
A
B
D
D
B
C C
.
.
.
.


Fig. 6. Fig. 7.

Teorema 4.2.8. (Pappus) ntr-un patrulater inscriptibil, produsul distanelor unui
punct al cercului circumscris patrulaterului la dou laturi opuse este egal cu produsul
distanelor la celelalte dou laturi opuse.

Demonstraie. Fie patrulaterul inscriptibil ABCD, M un punct al cercului circumscris,
A,B,C,D proieciile lui M pe laturile AB, BC, CD, respectiv DA (fig.7). Conform
lemei 4.2.1 avem MAMB=2RMA, MBMC=2RMB, MCMD=2RMC i
MDMA=2RMD, de unde rezult c MAMC=MBMD.

Definiia 4.2.2. ntr-un patrulater, bimedianele sunt segmente determinatele de
mijloacele laturilor opuse sau mijloacele diagonalelor.

Lema 4.2.2. ntr-un patrulater convex, bimedianele sunt concurente.

Demonstraie. Fie A, B, C, D, E, F mijloacele laturilor AB, BC, CD, DA, respectiv
a diagonalelor AC i BD (fig.8).


128
A
O
A
B
D
D
B
C
C
F
E


Fig. 8.

Deoarece AB i CD sunt linii mijlocii n triunghiurile ABC i ADC, avem c
AB||AC, ' '
2
AC
A B = i CD||AC, ' '
2
AC
C D = . Rezult c AB||CD i ABCD,
deci patrulaterul ABCD este paralelogram. Fie O punctul de intersecie al diagonale-
lor AC i BD, deci O este mijlocul acestor segmente. Deoarece AF i CE sunt linii
mijlocii n triunghiurile ABD i ACD, avem c AFCE este paralelogram cu diago-
nalele AC i EF, de unde rezult c diagonala EF trece prin mijlocul diagonalei AC,
adic prin O. Analog, se arat i pentru celelalte bimediane c trec prin punctul O.

Teorema 4.2.9. (Mathot) ntr-un patrulater inscriptibil, perpendicularele duse din
mijloacele laturilor pe laturile opuse sunt concurente.

Demonstraie. Fie O centrul cercului circumscris patrulaterului inscriptibil ABCD i
A,B,C, D mijloacele laturilor AB, BC, CD, respectiv DA (fig. 9). Deoarece O se afl
pe mediatoarele laturilor patrulaterului, rezult c OAAB, OBBC, OCCD i
ODDA. Conform lemei 4.2.2, bimedianele patrulaterului sunt concurente i fie E
punctul de concuren. Notm cu M simetricul lui O fa de E. Patrulaterul MAOC
este paralelogram deoarece diagonalele lui se njumtesc. Rezult c MA||OC. Dar
OCCD, deci MACD. Analog MBDA, MCAB i MDBC, deci punctul de
concuren este M, simetricul lui O fa de E.

129
A
A B
D
B
D
C
O
Q
E
P
M
C
.
.
.
.
.


Fig. 9.

Observaia 4.2.2. Punctul M de concuren din teorema 4.2.9 se numete punctul lui
Mathot.

Consecina 4.2.2. ntr-un patrulater inscriptibil, perpendiculara din mijlocul unei
diagonale pe cealalt diagonal conine punctul Mathot al patrulaterului.

Demonstraie. Folosim notaiile din teorema 4.2.9. i fie P, Q mijloacele diagonalelor
BD, respectiv AC. Din demonstraiile lemei 4.2.2 i a teoremei 4.2.9, rezult c
patrulaterul OPMQ este paralelogram deoarece diagonalele lui se njumtesc, deci
MP||OQ. Dar Q fiind mijlocul coardei AC, rezult c OQAC, deci MPAC. Analog
QMBD.

Lema 4.2.3. Un trapez este incriptibil dac i numai dac este isoscel.

Demonstraie. Dac trapezul ABCD, AB||CD este inscriptibil atunci m(A)+
m(C) =180. Dar m(A)+ m(D) =180, deci C D, adic trapezul este isoscel.
Reciproc, dac trapezul este isoscel atunci CD i cum m(A)+ m(D) =180,
rezult c m(A)+m(C) =180, adic patrulaterul este inscriptibil.

Teorema 4.2.10.(Euler) Mijloacele laturilor unui triunghi, picioarele nlimilor i
mijloacele segmentelor ce unesc vrfurile triunghiului cu ortocentrul lui, sunt situate pe
un acelai cerc, numit cercul lui Euler.

Demonstraie. Fie triunghiul ABC, A, B, C mijloacele laturilor BC, CA, respectiv
AB, BC AA
1
, BC A
1
, CA BB
1
, CA B
1
, } {
1 1
H BB AA = ,
'
1
A mijlocul
segmentului AH (fig. 10).

130
A
H
B
A
1
'
B
1
A
1
C
A'
B' C'
.
.


Fig. 10.

n triunghiul dreptunghic AA
1
B, A
1
C este median, deci
2
'
1
AB
C A = . n triunghiul
ABC, AB este linie mijlocie, deci
2
' '
AB
B A = . Rezult c A
1
C AB i deoarece
BC este linie mijlocie n triunghiul ABC, deci BC||BC, n final obinem c
patrulaterul BCA
1
A este trapez isoscel. Cum orice trapez isoscel este inscriptibil,
rezult c A
1
se gsete pe cercul C determinat de punctele A, B, C. Analog, celelalte
picioare ale nlimilor sunt pe cercul C. n triunghiul ABC, AC este linie mijlocie,
deci AC||AC. n triunghiul ABH, A
1
C este linie mijlocie, deci A
1
C||BH. Dar
BHAC, deci A
1
CAC sau m(A
1
CA)=90. Analog, m(A
1
BA)=90, deci
patrulaterul A
1
CAB este inscriptibil, rezult c A
1
se gsete pe cercul C. Analog,
pentru celelalte mijloace de segmente determinate de vrfuri i ortocentru.

Consecina 4.2.3. Fie triunghiul ABC, A,B,C mijloacele laturilor BC, CA, respectiv
AB, A
1
, B
1
, C
1
mijloacele segmentelor AH, BH, respectiv CH. Atunci dreptele A
1
A,
B
1
B, C
1
C , sunt concurente, iar punctul de concuren este centrul cercului lui Euler.

Demonstraie. Din demonstraia teoremei 4.2.10 rezult c A
1
A, B
1
B, C
1
C sunt
diametre n cercul lui Euler.

Consecina 4.2.4. ntr-un triunghi ABC, ortocentrul, centrul de greutate, centrul
cercului circumscris triunghiului i centrul cercului lui Euler sunt situate pe o aceeai
dreapt, numit dreapta lui Euler.

Demonstraie. n triunghiul ABC, fie A mijlocul segmentului BC, A punctul
diametral opus lui A n cercul circumscris triunghiului ABC, O centrul acestui cerc, A
1

mijlocul segmentului AH (fig.11).


131
B
A'
A
A''
C
O
G
H
A

Fig. 11.

Deoarece BHAC i ACAC, rezult c BH||CA. Analog, CH||BA, deci BHCA
este paralelogram, rezult c A este mijlocul lui HA. n triunghiul AHA, OA este
linie mijlocie, deci AH OA
2
1
' = . Dac AAHO={G},atunci AHGAOG, raportul
de asemnare este 2, deci AG=2AG, adic G este centrul de greutate al triunghiului
ABC. nseamn c G [HO] i HG=2GO. Mai avem c OAHA
1
i OA||HA
1
, deci
patrulaterul A
1
HAO este paralelogram i atunci diagonalele acestui paralelogram se
njumtesc. Fie A
1
AHO={O
9
}. Deoarece A
1
A este diametru n cercul lui Euler,
rezult c O
9
este centrul cercului lui Euler i se gsete la mijlocul lui HO.

Teorema 4.2.11. (Simson) Proieciile unui punct de pe cercul circumscris unui
triunghi pe laturile acestuia, sunt trei puncte coliniare.

Demonstraie. Fie triunghiul ABC, M un punct pe cercul circumscris triunghiului i
A, B, C proieciile lui M pe laturile BC, CA i respectiv AB (fig. 12).

A'
A''
C
M
C'
A
B'
B
.
.
.


Fig. 12.

Unim B cu C i B cu A. Deoarece patrulaterele ABMC, CABM i ABCM sunt
inscriptibile, avem m(ABC)=m(AMC)=90
0
-m(ACM)=

132
=90
0
- m(MAC)=m(CMA)=m(ABC), ceea ce nseamn c punctele A, B, C
sunt coliniare.

Observaia 4.2.3. Dreapta pe care se afl punctele A, B, C se numete dreapta lui
Simson.

Consecina 4.2.5. n condiiile teoremei 4.2.11, dac MA mai intersecteaz cercul
circumscris triunghiului ABC n A, atunci AA este paralel cu dreapta lui Simson.

Demonstraie. Avem c m(AAM)=m(ACM)=
2
) ( M A m
)
i BAMBCM din
patrulaterul inscriptibil BACM (fig. 12). Rezult c AAMBAM , deci AA||BA.

Teorema 4.2.12. (reciproca teoremei lui Simson) Fie M un punct exterior
triunghiului ABC i A, B, C proieciile pe M pe laturile BC, CA, respectiv BC. Dac
A, B, C sunt coliniare, atunci M se afl pe cercul circumscris triunghiului ABC.

Demonstraie. Deoarece punctele A, B, C sunt coliniare, rezult c ABCABC
(fig. 13).

C'
A
B
B'
A' C
M
.
.
.


Fig. 13.

Din patrulaterele inscriptibile ABMC i CABM rezult c
m(ABC)=m(AMC)=90
0
-m(MAC),
m(ABC)=m(AMC)=90
0
-m(MCA) i innd seama de relaiile de mai sus,
obinem c MACMCA, adic patrulaterul ABCM este inscriptibil. n concluzie,
M este pe cercul circumscris triunghiului ABC.

Teorema 4.2.13. (Salmon) Fie triunghiul ABC, M este un punct arbitrar pe cercul
circumscris triunghiului. Cercurile de diametre MA, MB i MC se intersecteaz dou
cte dou n trei puncte coliniare.


133
Demonstraie. Fie C al doilea punct de intersecie a cercurilor de diametre MA i MB.
Deoarece m(ACM)=m(BCM)=90, rezult c MCAB, CAB. innd seama de
teorema lui Simson, rezult c cele trei puncte sunt coliniare.

A
B
B'
M


Fig.14.

Teorema 4.2.14. (Naghel) Fie triunghiul ABC, C(O) cercul circumscris triunghiului,
B, C proieciile vrfurilor B, C pe laturile opuse. Atunci, perpendiculara din A pe BC
trece prin O.

Demonstraie. Perpendiculara din A pe BC intersecteaz a doua oar cercul n A i
intersecteaz pe BC n D (fig. 15). Patrulaterele ABAC i BCBC sunt inscriptibile,
deci ABCAAC i ABCABC. Rezult c AACABC. Dar
triunghiurile AAC i ABD au unghiul BAD comun i AACABC, deci
ACAADB. innd seama c m(ADB)=90
0
, rezult c m(ACA)=90
0
, deci
AA este diametru n cercul C(O).

A
B
D
A
C
B
C
.
.


Fig.15.



134
4.3. Patrulaterul circumscriptibil

n acest paragraf vom studia proprietile patrulaterului circumscriptibil.

Definiia 4.3.1. Un patrulater se numete circumscris unui cerc sau
circumscriptibil, dac laturile sale sunt tangente la un cerc.

Teorema 4.3.1. Un patrulater convex este circumscriptibil dac i numai dac
bisectoarele unghiurilor sale sunt concurente.

Demonstraie. Artm c dac patrulaterul ABCD este circumscriptibil, atunci
bisectoarele unghiurilor sale sunt concurente (fig. 1). Fie I centrul cercului nscris
patrulaterului ABCD. Atunci d(I,AB)=d(I,BC)=d(I,CD)=d(I,DA)= r i innd seama de
proprietatea bisectoarei unui unghi, rezult c I se afl pe bisectoarele unghiurilor
patrulaterului ABCD. Deci avem proprietatea de concuren. Reciproc, dac I este
punctul de concuren al bisectoarelor unghiurilor patrulaterului ABCD, atunci
d(I,AB)=d(I,BC) =d(I,CD)=d(I,DA). Cercul de centru I i raz r = d (I, AB) este
tangent fiecrei laturi a patrulaterului ABCD, deci patrulaterul ABCD este
circumscriptibil.

Teorema 4.3.2. (Pithot) Dac patrulaterul ABCD este circumscriptibil,
atunci AB+CD= AD+BC.

Demonstraie. Fie M,N,P,Q punctele de contact ale cercului nscris cu laturile AB, BC,
CD , respectiv DA (fig. 2).


A
B
D
C
I

A
M
N
Q
P
B
D C
I

Fig.1. Fig.2.

Au loc urmtoarele egaliti de tangente la cerc AMAQ, BMBN, CNCP i
DPDQ, de unde
AB+CD=(AM + BM)+(CP+DP)=(AQ+DQ)+(BN+CN)=BC+AD, adic identitatea
din enun.


135
Teorema 4.3.3. Dac un patrulater convex ABCD are proprietatea c
AB+CD=AD+BC, atunci patrulaterul ABCD este circumscriptibil.

Demonstraie. Considerm un cerc C, tangent la AB, BC i CD, apoi o dreapt AD
tangent acestui cerc DCD.
A
B
D D C
fig. 3

n patrulaterul ABCD care este circumscris cercului C, conform teoremei lui Pithot,
avem c AB+CD=AD+BC. Dar din ipotez AB+CD=AD+BC, de unde, prin scderea
celor dou relaii, obinem c DD=|AD AD|. Aceast relaie are loc dac i numai
dac triunghiul ADD este degenerat, adic D coincide cu D. Rezult c patrulaterul
ABCD este circumscriptibil.

Consecina 4.3.1. Patrulaterul convex ABCD este circumscriptibil dac i
numai dac AB+CD=AD+BC.

Demonstraie. Rezult din teoremele 4.3.2 i 4.3.3.

Observaia 4.3.1. Condiia de convexitate intervine n demonstraie destul de
subtil. Figura 4 este un exemplu de patrulater ABCD cu ABAD i BCCD, deci are
loc AB+CD=AD+BC, fr a fi ns patrulater circumscriptibil.

A
B
C
D

Fig.4.

136
Propoziia 4.3.1. Dac un trapez isoscel este circumscriptibil, atunci
diametrul cercului nscris este medie geometric ntre bazele trapezului.

Demonstraie. Fie trapezul ABCD, AD||BC, C(I,r) cercul nscris i T punctul de
contact al cercului cu AB (fig.5).

B
T
C
A
D
I
.


fig. 5

Deoarece m(AIB)=90, aplicnd teorema nlimii n triunghiul AIB, avem
c IT
2
=ATBT. Dar innd seama c IT = r, AD AT
2
1
= i BC BT
2
1
= , rezult
identitatea din enun.


4.4. Relaii metrice

n cele ce urmeaz, pentru un patrulater ABCD, vom folosi urmtoarele notaii:
AB=a, BC=b, CD=c, DA=d, BD=e, AC=f i p=
2
d c b a + + +
.
Teorema 4.4.1. (Arhimede ) Aria S a unui patrulater convex ABCD este
dat de formula S
2
2
cos ) )( )( )( (
2
D B
abcd d p c p b p a p
+
= .
Demonstraie. innd seama c A[ABCD]=A[ABC]+A[ADC], rezult c
2S=absinB+cdsinD, de unde prin ridicare la ptrat avem
D B abcd D d c B b a S sin sin 2 sin sin 4
2 2 2 2 2 2 2
+ + = . Pe de alt parte din teorema
cosinusului n triunghiurile ABC i ADC avem B ab b a AB cos 2
2 2 2
+ = i
D cd d c AB cos 2
2 2 2
+ = , de unde D cd d c B ab b a cos 2 cos 2
2 2 2 2
+ = + ,
sau D cd B ab d c b a cos 2 cos 2
2 2 2 2
= + i prin ridicare la ptrat obinem
D B abcd D d c B b a d c b a cos cos 8 cos 4 cos 4 ) (
2 2 2 2 2 2 2 2 2 2 2
+ = + .
Se nmulete relaia care-l d pe 4 S
2
cu 4 i se adun cu ultima relaie obinut i
avem ) cos( 8 4 4 ) ( 16
2 2 2 2 2 2 2 2 2 2
D B abcd d c b a d c b a S + + = + + ,

137
sau
= + + + + = )] cos( 1 [ 8 ) ( ) 2 2 ( 16
2 2 2 2 2 2
D B abcd d c b a cd ab S
.
2
cos 16 ) )( )( )( (
2
cos 16 ] ) ( ) ][( ) ( ) [(
2
cos 16 ) 2 2 )( 2 2 (
2
2 2 2 2 2
2 2 2 2 2 2 2 2 2
D B
abcd d c b a d c b a b a d c b a d c
D B
abcd d c b a b a d c
D B
abcd d c b a cd ab d c b a cd ab
+
+ + + + + + + + =
=
+
+ + =
=
+
+ + + + + + =

innd seama c 2p= a+b+c+d i nlocuind mai sus, obinem formula lui Arhimede.
Propoziia 4.4.1. Fie patrulaterul inscriptibil ABCD. S se arate c
bc ad
cd ab bd ac
e
+
+ +
=
) )( (
2
i
cd ab
bc ad bd ac
f
+
+ +
=
) )( (
2
.

Demonstraie. n triunghiurile ABC, respectiv ACD avem
ab
f b a
B
2
cos
2 2 2
+
= i
cd
f d c
D
2
cos
2 2 2
+
= . Patrulaterul ABCD fiind inscriptibil avem c B+D=180, deci
cosB= -cosD, de unde
cd
f d c
ab
f b a
2 2
2 2 2 2 2 2
+
=
+
, iar dup calcule obinem
identitile din enun.

Observaia 4.4.1. Folosind relaiile demonstrate, cu ajutorul lor se pot
demonstra cele dou teoreme ale lui Ptolemeu.

Teorema 4.4.2. (Arhimede) Aria patrulaterului inscriptibil ABCD este dat
de formula ) )( )( )( ( d p c p b p a p S = .

Demonstraie. Rezult din teorema 5.4.1.

Teorema 4.4.3 Dintre toate patrulaterele convexe de laturi date, patrulaterul
inscriptibil are aria maxim.

Demonstraie. Deoarece a, b, c, d sunt date, avem
. ) )( )( )( (
2
cos ) )( )( )( (
2 2
const d p c p b p a p
D B
abcd d p c p b p a p S =
+
=
Atunci S
2
este maxim dac i numai dac 0
2
cos =
+ D B
, adic B+D=180
0
, deci
patrulaterul ABCD este inscriptibil.


138
Teorema 4.4.4. Aria patrulaterului circumscriptibil este dat de formula
2
sin
D B
abcd S
+
= .
Demonstraie. ntr-un patrulater circumscriptibil avem a+c=b+d, deci
c
d c b a
a p =
+ + +
=
2
i analoagele. Atunci
2
sin )
2
cos 1 (
2
cos
2 2 2 2
D B
abcd
D B
abcd
D B
abcd abcd S
+
=
+
=
+
= , de
unde rezult formula din enun.

Consecina 4.4.1. Aria unui patrulater ABCD inscriptibil i circumscriptibil
este dat de formula abcd S = .
Demonstraie. Rezult din teorema 4.4.4.

139
5. Construcii geometrice

Prin probleme de construcie vom nelege acele probleme de geometrie n care
se cere construirea unor figuri geometrice ce satisfac anumite proprieti, folosind
numai rigla i compasul.
nainte de a considera probleme de construcie cu rigla i compasul, Edwin
Moise n lucrarea "Geometrie elementar" face cteva precizri:
i) Cnd vorbim de rigl i compas, nelegem o "rigl ideal" i un "compas
ideal", care traseaz liniile drepte i cercurile exact.
ii) Rigla nu are un marcaj pe ea. O putem utiliza pentru a desena drepte ntre
dou puncte date, dar aceasta este tot ceea ce putem face cu ea. Nu o putem utiliza
pentru a msura distanele dintre puncte sau pentru a vedea dac dou segmente sunt
congruente.
iii) Compasul se poate utiliza astfel. Fie un punct P i un punct Q n plan.
Putem desena atunci cercul cu centrul n P i care trece prin Q. Aceasta este tot ce
putem face cu el. Altfel spus, dndu-se un al treilea punct P' nu este permis s mutm
vrful compasului n P' i apoi s desenm un cerc cu centrul n P' i de raz PQ. Din
acest motiv compasul este numit nerigid; nu i se poate muta vrful deoarece "cnd
ridici vrful de pe hrtie, compasul se nchide".


5.1. Construcia unor expresii algebrice

5.1.1. Construcia expresiilor b a + i b a ) ( b a >
Considerm numerele pozitive a i b cu b a > . Pe o dreapt d considerm un
punct O i trasm un arc de cerc de raz a, pn cnd ntlnete dreapta d n punctul A.
Cu centrul n A trasm cercul de raz b care va intersecta dreapta d n punctele B i C
(B ntre O i A). Lungimea segmentului OC reprezint expresia b a + , iar lungimea
segmentului OB reprezint expresia b a .
d
C
A
b B O
a
b
a
140
5.1.2. Construcia expresiilor b a i
b
a
) ( b a >
i) Construcia expresiei b a
Trasm dou semidrepte c i d cu originea comun O. Pe c trasm segmentul
unitate OA i segmentul OB se lungime a. Pe semideapta d cu originea n O trasm
segmentul OC de lungime b. Paralela prin B la AC ntlnete pe d n D. Cu teorema lui
Thales n OBD obinem:
OD
OC
OB
OA
= , de unde b a =

=
OA
OC OB
OD .
ii) Construcia expresiei
b
a

Trasm dou semidrepte c i d cu originea comun n O. Pe semidreapta c
lum segmentul unitate OA i segmentul OB de lungime b. Pe semidreapta d lum
segmentul OC de lungime a. Prin A ducem paralela la BC care ntlnete pe c n D. n
OBC cu teorema lui Thales obinem:
OB
OA
OC
OD
= , de unde
b
a
=

=
OB
OA OC
OD .
5.1.3. Construcia mediei aritmetice, geometrice
i) Construcia mediei aritmetice
Considerm dou segmente de lungimi a i b ) ( b a > . Construim segmentului
AB care reprezint suma b a + . Construim mediatoarea segmentului AB. Mediatoarea
determin pe segmentul AB dou segmente congruente care reprezint numrul
2
b a +
.

A O B
C
D
d
c
ab
b
1
b
a
a
A
C D
B
a b
a
b
a b +
2
141
ii) Construcia mediei geometrice
Considerm dou segmente ce au lungimile ai b ) ( b a > . Construim
segmentul AB care reprezint suma b a + . Construim un semicerc de diametru AB.
Fie C punctul de pe [AB] cu AC=a. Ridicm n C o perpendicular pe AB care
ntlnete semicercul n E. Cu teorema nlimii n triunghiul dreptunghic AEB
(m(E)=90) obinem: EC
2
=ACCB sau EC
2
=ab, de unde b a = EC .
Aplicaie. Construcia expresiei a cu a>0.
Este o aplicaie a construciei mediei geometrice pentru numerele 1 i a (a>0).
5.1.4. Determinarea a dou numere cnd se cunosc suma lor i media
geometric
Fie ai b numere pozitive cu suma b a x + = i media geometric ab y = .
Problema se reduce la construcia triunghiului dreptunghic cu ipotenuza x i
nlimea y.
Trasm diametrul AB reprezentnd numrul x. Construim semicercul de
diametru AB. Trebuie s gsim interseciile acestui semicerc cu o paralel d dus la el
de aceeai parte cu semicercul fa de AB situat la distana y.
1) Dac paralela d la AB intersecteaz semicercul n dou puncte distincte E i
K, considerm proieciile acestor puncte pe AB, adic E' i K', obinem perechile de
segmente AE' i E'B respectiv AK' i K'B, care sunt numerele cutate.
2) Dac paralela d la AB este tangent semicercului avem soluie unic,
numerele sunt egale.
3) Dac paralela d la AB nu intersecteaz semicercul nu avem soluie.

a
b
E
A C B
a b
a b
E
A C 1 a b
a
E
A E' b
K
K'
142
6. Figuri echivalente

Figurile echivalente sunt figurile geometrice plane care au aceeai arie fr a fi
neaprat congruente.

6.1. Triunghiuri echivalente
Din relaia ariei
2
inaltimea baza
S

= rezult c ariile a dou triunghiuri sunt
egale dac bazele celor dou triunghiuri au lungimi egale i nlimile de asemenea
sunt de lungimi egale.
Exemple. i) Dou triunghiuri care au un vrf fixat, iar bazele de lungimi egale
se afl pe aceeai dreapt au aceeai arie (BC=DE).
Deci S
[ABC]
=S
[ADE]
.
ii) Dou triunghiuri cu aceeai baz i vrfurile situate pe o paralel la baz, au
aceeai arie. Deci
BC] [A [ABC]
1
S S = (d||BC).
iii) n figura de mai jos avem
S
[AOD]
=S
[BOC]
(*)
S
[AOD]
=S
[ADB]
-S
[AOB]
(1)
S
[BOC]
=S
[ACB]
-S
[AOB]
(2)
Dar
2
AB) d(C, AB
2
AB) d(D, AB
S S
[ACB] [ADB]

=

= (3)
Din (1), (2) i (3) relaia (*).
A
B C D E
A A
1
B
C
d
143

6.2. Triunghi echivalent cu un paralelogram dat
Aria unui paralelogram este egal cu dublul ariei unui triunghi.
i) S
[ABCE]
=S
[ADB]
, unde C este mijlocul lui [BD].
S
[ABCE]
=BCd(A,BC)=BCAA
1

1
1 1
[ADB]
AA BC
2
AA BC
2
2
AA BD
S =

=
ii) S
[ABCE]
=S
[ABD]
, unde DF=2EL, a||AB
S
[ABCE]
=ABd(E,AB)=ABEL
EL AB
2
2EL AB
2
DF AB
S
[ABD]
=

=

6.3. Triunghi echivalent cu un trapez dat
S
[ABCD]
=S
[LBT]
, unde [DC][AT]
A B
O
D C
A E
B A1 C D
L
D a
E C
A L F B
144
[LBT] [ABCD]
S
2
AB) d(L, TB
2
AB) d(D, TB
2
AB) DC)d(D, (AB
S =

=
+
=
6.4. Triunghi echivalent cu un patrulater
S
[ABCD]
=S
[LCK]
(**)
Fie ABCD un patrulater oarecare. Ducem printr-un vrf oarecare (de exemplu
A) o dreapt d. Prin vrfurile D i B ale patrulaterului ABCD ducem paralele la AC,
care ntlnesc pe d n L respectiv K. Obinem trapezele ACDL i ACBK. Avem
S
[LCK]
=S
[ACL]
+S
[ACK]
. Atunci
S
[ABCD]
=S
[ADC]
+S
[ABC]
(1)
Dar
[LAC] [ADC]
S
2
AC) d(L, AC
2
AC) d(D, AC
S =

= (2)
[AKC] [ABC]
S
2
AC) d(K, AC
2
AC) d(B, AC
S =

= (3)
Din (1), (2) i (3) obinem relaia (**).
6.5. Triunghi echivalent cu un pentagon
S
[ABCDE]
=S
[DLM]

Considerm pentagonul ABCDE. Trasm diagonalele [DA] i [DB]. Ducem
prin E o paralel la DA care ntlnete pe AB n L, iar prin C o paralel la DB care
ntlnete pe AB n M.
Triunghiurile ADL i AED sunt echivalente deoarece:
[AED] [ADL]
S
2
AD) d(E, AD
2
AD) d(L, AD
S =

= (1)
i triunghiurile BMD i DBC sunt echivalente deoarece:
L
T
A B
C D
d
K A L
B
D
C
145
[DBC] [BMD]
S
2
DB) d(C, DB
2
DB) d(M, DB
S =

= (2)
[ADE] [BDA] [BCD] [ABCDE]
S S S S + + = (3)
Din relaiile (1), (2) i (3) obinem:
[LDM] [ADL] [BDA] [BMD] [ABCDE]
S S S S S = + + =
Deci triunghiul LDM este echivalent cu pentagonul ABCDE.
6.6. mprirea unui triunghi n pri echivalente
i) mprirea unui triunghi n n pri echivalente prin mprirea unei laturi a
triunghiului, considerat ca baz, n n segmente congruente.
Considerm triunghiul ABC cu baza [BC], pe care o mprim n n pri egale.
Obinem triunghiurile echivalente BAD
1
, D
1
AD
2
,...,D
n-1
AC. (Triunghiurile au aceeai
nlime d(A,BC)).
ii) mprirea unui triunghi n n pri echivalente prin drepte paralele cu una
din laturi.
Vom studia cazul n=3. Deci triunghiul ABC trebuie mprit n trei pri
echivalente prin paralele duse la baza [BC].
Fie (LT)||(BC) i (PQ)||(BC) cu L(AP), P(LB), T(AQ), Q(TC).
Deci
[ALT] [ABC]
3S S = i
[ABC] [APQ]
S
3
2
S = (1)
inem seama c:
Raportul ariilor a dou triunghiuri asemenea este egal cu ptratul raportului de
asemnare.
Cu (1) obinem:
3
1
AC
AT
2
=

i
3
2
AC
AQ
2
=

(2)
Din (2) obinem:

=
3
AC
AC AT
2
i

= AC
3
2
AC AQ
2
(3)
Din (3) rezult c AT i AQ sunt medii geometrice a lungimilor segmentelor
AC,
3
AC
i respectiv AC, AC
3
2
.
D
E
L A B M
C
146
Acum efectum construcia.
Construim pe latura AC ca diametru un semicerc n afara triunghiului ABC.
mprim latura AC n trei segmente congruente prin punctele H i K. Perpendicularele
n H i K pe AC ntlnesc semicercul n E respectiv F.
Conform teoremei catetei n triunghiul dreptunghic AEC avem:
3
AC
AC AH AC AE = = , iar n triunghiul dreptunghic AFC avem:
3
2AC
AC AQ = .
Arcele de cerc cu centrul n A i raze AE, respectiv AF determin pe AC
punctele T i Q care sunt punctele cutate. Ducem prin T i Q paralele la BC i
obinem mprirea ABC n trei pri echivalente.

6.7. Ptrat echivalent cu un paralelogram ABCD
S
[ABCD]
=S
ptrat
.
Considerm paralelogramul ABCD. Construim nlimea DL cu L[AB].
Lum pe prelungirea laturii [CD] un segment (DL)(DQ). Construim media
geometric a lungimii segmentelor CD i DQ, determinnd punctul de intersecie T al
semicercului de diametru CQ cu dreapta DL. Atunci DT este latura ptratului
echivalent cu paralelogramul ABCD.

A
H
T
K
Q
C B
P
L
E
F
Q D C
B L
T
A
147
6.8. Dreptunghi echivalent cu un patrulater dat

[HRPE] [HLTE] [LTPR] [ABCD]
S S S
2
1
S = = = , unde H i E sunt mijloacele segmentelor
LR respectiv PT. Fie ABCD un patrulater oarecare. Prin vrfurile B i D ducem
paralele la AC. Prin vrfurile A i C ducem perpendiculare pe AC. Obinem
dreptunghiul LTPR. Ducem din B respectiv D perpendiculare pe AC care au picioarele
n K i respectiv N.
Avem relaiile:
[DPC] [ARD] [BCK] [ABK] [ABCD]
S S S S S + + + = (*)
[ABK] [ALB]
S S = (1)
[BKC] [BTC]
S S = (2)
[DNA] [ARD]
S S = (3)
[DNC] [DPC]
S S = (4)
[DPC] [ARD] [BCK] [ABK] [ABCD] [RPTL]
S S S S S S + + + + = (5)
Din relaiile (*) i (5) obinem:
[ABCD] [RPTL]
2S S = , rezult
[RPTL] [ABCD]
S
2
1
S = (6)
Fiindc H i E sunt mijloacele laturilor [RL] respectiv [PT] avem
[HLTE] [RPEH] [RPTL]
2S 2S S = = (7)
Din (6) i (7) rezult
[HLTE] [RPEH] [ABCD]
S S S = = .

6.9. Dreptunghi echivalent cu un trapez dat

[EFGH] [ABCD]
S S = , unde ABCD este trapez, iar EFGH dreptunghi.
Fie ABCD un trapez cu AB||CD dat, cu mijloacele laturilor neparalele L i T
(L(AD), T(BC)). Prin L i T ducem perpendiculare pe baze, care ntlnesc baza
mare n E, respectiv F, iar prelungirea bazei mici n H respectiv G.
R
D
P
E
B
L
H
A
T
N
K
C
148
Din triunghiurile congruente HDL i EAL rezult c (HD)(AE), iar din
triunghiurile congruente GCT i FBT, obinem c (CG)(FB). Atunci
=
+ + +
=
+
= AB) d(D,
2
DC) FB AE (EF
AB) d(D,
2
DC) (AB
S
[ABCD]

[HEFG]
S AB) d(D, EF
2
AB) d(D, 2EF
= =

= .
6.10. Construcia unei drepte care s mpart un patrulater convex n dou pri
avnd aceeai arie
Considerm patrulaterul convex ABCD, a crui diagonale se intersecteaz n
O. Analizm situaiile:
i) (OD)(OB) (1)
Avem
[AOD] [AOB]
D
2
BD) d(A, OD
2
BD) d(A, OB
S =

=
Deci
[AOD] [AOB]
S S = (2)
[DOC] [COB]
S
2
OB) d(C, OD
2
OB) d(C, OB
S =

=
Deci
[DOC] [COB]
S S = (3)
Din (2) i (3) obinem:
[DOC] [AOD] [COB] [AOB]
S S S S + = + adic
[ADC] [ABC]
S S = ,
adic AC este dreapta cutat.
ii) (OD)(OB). Presupunem c OB>OD.
Ducem prin D o paralel la AC care ntlnete pe BC n K. Se obine triunghiul
AKB echivalent cu patrulaterul ABCD. (Un triunghi echivalent cu un patrulater). Deci
[ABCD] [AKB]
S S = . Fie T mijlocul lui [KB], atunci
[KTA] [ABT]
S S = fiindc
[KTA] [ABT]
S
2
TK) d(A, KT
2
BK) d(A, BT
S =

= . Atunci
2
S
S
[ABK]
[ABT]
= i deci
2
S
S
[ABCD]
[ABT]
= .
H D C
G
T
B F E A
L
149
Pentru a arta c dreapta AT este soluia problemei trebuie s artm c
triunghiul ABT este parte a patrulaterului ABCD, ceea ce nseamn c T(BC). Fie Q
mijlocul lui (BD). Fiindc (OB)>(OD) rezult Q(OB).
n triunghiul ABK, [TQ] este linie mijlocie deci QT||DK. Fiindc OC||DK
obinem c OC||TQ i cum Q(OB) rezult c T(BC).
Construcia unei drepte care s mpart un patrulater convex n dou pri avnd
aceeai arie

6.11. mprirea unui patrulater printr-o dreapt care trece printr-un vrf, n
pri, avnd ariile ntr-un raport dat (Fig. 1)
Fie patrulaterul ABCD i
b
a
raportul n care trebuie s mprim aria
patrulaterului printr-o dreapt care s treac prin D. Transformm patrulaterul ABCD
ntr-un triunghi echivalent cu el, care s aib o latur pe dreapta AB. Prin C ducem o
paralel la BD care ntlnete dreapta AB n L. Obinem triunghiul ADL care este
echivalent cu patrulaterul ABCD deoarece

=
2
BD) d(L, BD
2
BD) d(C, BD
S S
[BDL] [BDC]

i atunci
[ADL] [BDL] [ADB] [BCD] [ADB] [ABCD]
S S S S S S = + = + = .
mprim aria triunghiului ADL n raportul
b
a
. Pentru aceasta este suficient s
mprim latura AL n raportul
b
a
. Fie K punctul pentru care avem
b
a
KL
AK
= . Atunci
b
a
2
KL) d(D, KL
2
AK) d(D, AK
=

, adic
b
a
S
S
[DKL]
[ADK]
= (1)
K
D
C
O
Q
T
B
A
150
Ducem prin K o paralel la BD care ntlnete pe BC n Q. Unim D cu Q.
Obinem:
b
a
S
S
[DQC]
[ABQD]
= (2)
fiindc

=
2
BD) d(Q, BD
2
BD) d(K, BD
S S
[DBQ] [DBK]
, iar
[DBK] [DBL] [DKL]
S S S = sau
] [ [BDQ] [DBC] [DKL]
S S S
DQC
S = = .
Triunghiul ADK este echivalent cu patrulaterul ADQB pentru c triunghiurile
BDQ i BDK sunt echivalente. Deci
[ADQB] [ADK]
S S = .
n relaia (1) nlocuim pe
[ADK]
S cu
[ADQB]
S i
[DKL]
S cu
[DQC]
S i obinem
relaia (2).
Din construcia de mai sus rezult c n loc s construim triunghiul ADL
echivalent cu patrulaterul ABCD putem s mprim diagonala AC n raportul
b
a
. Fie
T punctul de pe AC pentru care avem
b
a
TC
AT
= . Prin T ducem o paralel la BD care
ntlnete pe BC n Q. Dreapta DQ mparte aria patrulaterului n raportul
b
a
.
6.12. mprirea unui patrulater prin drepte duse printr-un vrf n mai multe
pri echivalente
De exemplu, pentru a mpri un patrulater ABCD n trei pri echivalente prin
dou drepte duse prin D procedm astfel:
Se mparte diagonala AC n trei pri egale prin punctele R i V. Prin aceste
puncte ducem paralele la diagonala BD, care ntlnesc laturile (AB) respectiv (BC) n
punctele J i S. Dreptele DJ i DS mpart patrulaterul n trei pri echivalente.
Fig. 1.
S
Q
D
A
R
J
B
V T
K
C
L
151
6.13. mprirea unui poligon convex n pri echivalente, prin drepte duse
printr-un vrf
Considerm pentagonul EFGHT pe care dorim s-l mprim n pri
echivalente prin drepte duse prin vrful G.
Vom arta mai nti c putem mpri aria poligonului n raportul
b
a
printr-o
dreapt ce trece prin G. Vom construi un patrulater echivalent cu pentagonul
considerat. Ducem prin F o paralel la diagonala GE care ntlnete pe ET n L.
Patrulaterul LGHT este echivalent cu pentagonul EFGHT. Artm n continuare acest
lucru. Avem
[GLE] [GFE]
S S = (1)
deoarece
2
GE) d(L, GE
2
GE) d(F, GE
=

. Deci
[LGHT] [EGHT] [LGE] [EGHT] [EFG] [EFGHT]
S S S S S S = + = + =
Am redus problema la una pe care am studiat-o.
Vom mpri diagonala HL n raportul
b
a
. Fie K punctul pe HL pentru care
avem
b
a
KH
LK
= . Prin K ducem paralela KQ la GT care ntlnete pe [HT] n R, QET.
Dreapta GR mparte aria patrulaterului LGHT n raportul
b
a
. Dac prin H ducem
paralela HV la GT, VET patrulaterul GLTH este echivalent cu triunghiul LGV
deoarece

=
2
GT) d(V, GT
2
GT) d(H, GT
S S
[GTV] [GHT]

i atunci
[GLV] [GLT] [GTV] [GLT] [GHT] [GLTH]
S S S S S S = + = + =
Fiindc KQ||GT, iar HV||GT obinem c KQ||HV. Cu teorema lui Thales n
LHV avem:
b
a
KH
LK
QV
LQ
= = , de unde obinem c
b
a
2
QV) d(G, QV
2
LQ) d(G, LQ
=

,
care se mai scrie
b
a
S
S
[QGV]
[LGQ]
= (2)
152
Dar
[GTR] [GTQ]
S S = deoarece
2
GT) d(R, GT
2
GT) d(Q, GT
=


Atunci
[GTQ] [GTV] [QGV]
S S S = , care se mai poate scrie:
[GTR] [GTH] [QGV]
S S S = ,
deci
[GHR] [QGV]
S S = (3)
Atunci relaia (2) se mai poate scrie:
b
a
S
S S
[GHR]
[GTQ] [LGT]
=
+
sau
b
a
S
S S
[GHR]
[GTR] [LGT]
=
+
sau
b
a
S
S
[GHR]
[LGRT]
= (4)
innd seama de (1), relaia (4) se mai scrie
b
a
S
S
[GHR]
[GFETR]
= , deci putem mpri
aria unui poligon convex (n cazul nostru pentagon) ntr-un raport dat, printr-o dreapt
ce trece printr-un vrf al su (n cazul nostru G). Acest rezultat ne d posibilitatea s
mprim un poligon convex oarecare n pri echivalente, prin drepte duse prin unul
din vrfurile sale.
n cazul problemei noastre, se construiete patrulaterul echivalent cu poligonul
dat. Dac cerina era s mprim pentagonul n patru pri echivalente, atunci
mpream diagonala [LH] n patru pri egale prin punctele S, W, U, iar prin aceste
puncte ducem paralele la diagonala (CT) care ntlnesc laturile ET i TH n punctele C,
B, A. Unind punctele C, B, A cu G obinem dreptele GC, GB, GA care mpart
pentagonul dat n patru pri echivalente:
[GAH] [GBTA] [CRB] [GFET]
S S S S = = = .
G
F
K
H
L E T Q V
R
153
6.14. mprirea unui poligon dat n pri cu arii proporionale cu mai multe
numere date, prin drepte duse printr-un punct interior poligonului
Considerm patrulaterul ABCD pe care s-l mprim n trei pri cu ariile
proporionale cu numerele a, b, c, prin drepte duse prin punctul M din interiorul
patrulaterului. Prin M ducem dreapta ML, L(DC).
i) Unim punctul M cu D, iar prin L ducem o dreapt paralel cu MD care
ntlnete pe AD n P. Atunci
[MDL] [MDP]
S S = deoarece
2
MD) d(L, MD
2
MD) d(P, MD
=

. Atunci
[AMP] [MDP] [ADM] [MDL] [ADM] [ADLM]
S S S S S S = + = + =
Deci
[AMP] [ADLM]
S S = (1)
Ducem prin P paralela la AM care ntlnete pe BA n R. Atunci avem:
[AMR] [AMP]
S S = (2)
deoarece
2
AM) d(R, AM
2
AM) d(P, AM
=

.
[ABM] [AMR] [RMB]
S S S + = (3)
Din (2) i (3) obinem:
[ABM] [AMP] [RMB]
S S S + = (4)
Cu (1), relaia (4) se mai poate scrie
[MLDAB] [RMB]
S S = (5)
Acum ducem prin R o paralel la BM care ntlnete pe CB n T. Atunci
[MBT] [RMB]
S S = , deoarece
2
MB) d(T, MB
2
MB) d(R, MB
=

.
[TMB] [MCB] [MCT]
S S S + = (6)
Folosind relaia (5), obinem:
[MLDAB] [CMB] [MCT]
S S S + = sau
[MLDABC] [MCT]
S S = (6*)
Ducem prin T o paralel la MC care ntlnete pe DC n V, atunci avem:
G
F
U
W
S
H
A
L E C B T
154
[MCV] [MCT]
S S = (7)
deoarece
2
MC) d(V, MC
2
MC) d(T, MC
=

. Din relaiile (6*) i (7) obinem:


[MLC] [MLDABC] [MLC] [MCT]
S S S S + = + , care se mai scrie:
[ABCD] [MLC] [MCV]
S S S = + sau
[ABCD] [MLV]
S S = .
ii) mprim latura LV a triunghiului MLV n trei pri proporionale cu
numerele a, b, c prin punctele J i K, adic
c
JV
b
KJ
a
LK
= = , relaie care se mai poate
scrie:
c
2
JV) d(M, JV
b
2
KJ) d(M, KJ
a
2
LK) d(M, LK
=

(8)
deoarece d(M,LK)=d(M,KJ)=d(M,JV). Relaia (8) se mai scrie:
c
S
b
S
a
S
[MJV] [MKJ] [MLK]
= = (8*)
Ducem prin K o paralel la CM care ntlnete latura BC n G. Patrulaterul
LMGC este echivalent cu triunghiul MLK deoarece
[MCG] [MLC] [LMGC]
S S S + = sau
[CMK] [MLC] [LMGC]
S S S + = deoarece d(G,MC)=d(K,MC). Deci
[MLK] [LMGC]
S S = .
iii) Prin punctul J ducem o paralel la CM care ntlnete dreapta CB n Q, iar
din Q ducem o paralel la MB care ntlnete pe BA n I, iar din I ducem o paralel la
AM care ntlnete pe AD n W. Pentagonul GBAWM face parte din patrulaterul
ABCD i avem:
[MKJ] [GBAWM]
S S = (9)
Vom demonstra n continuare relaia (9).
[CMK] [MCJ] [MKJ]
S S S =
sau
[CMG] [MCJ] [MKJ]
S S S = (9*)
T
Q
J
A
B
R W
D
M
G
L C K J V
P
155
dar
[CMQ] [MCJ]
S S = (10)
deoarece
2
MC) d(Q, MC
2
MC) d(J, MC
=

. Scznd din ambii membri ai relaiei


(10) pe
[CMG]
S obinem:
[CMG] [CMQ] [CMG] [MCJ]
S S S S = (11)
innd seama de relaia (9*), (11) se scrie:
[QMG] [MKJ]
S S = . Dar
[MBQ] [MGB] [MGQ]
S S S + = i
[MBI] [MBQ]
S S = fiindc
2
MB) d(I, MB
2
MB) d(Q, MB
=


[MAW] [MBA] [MAI] [MBA] [IMB]
S S S S S + = + =
deoarece
[MAW] [MAI]
S S = (MAd(I,MA)=MAd(W,AM)). Atunci avem:
[MAW] [MBA] [MGB] [MKJ]
S S S S + + = ,
care se mai scrie
[MGBAW] [MKJ]
S S = . Analog gsim
[MLDW] [MJV]
S S = . Deci
[MKJ] [MGBAW] [MLK] [MLCG]
S S , S S = = i
[MJV] [MLDW]
S S = (11)
Din (8*) i (11) obinem
c
S
b
S
a
S
[MLDW] [MGBAW] [MLCG]
= = .


156
7. Metoda areolar

Vom demonstra unele relaii metrice prin consideraii areolare.
Teorema 7.1. Dac x, y, z sunt distanele unui punct M interior triunghiului
ABC, la BC, AC. AB i S
[ABC]
este aria triunghiului, avem relaia:
[ABC]
S 2 cz by ax = + +
Demonstraie. Avem relaiile:
2
x BC
S
[MBC]

= (1)
2
y AC
S
[AMC]

= (2)
2
z AB
S
[BMA]

= (3)
Atunci
2
z AB
2
y AC
2
x BC
S S S S
[BMA] [AMC] [MBC] [ABC]

+

= + + = ,
care se mai scrie z c y b x a 2S
[ABC]
+ + = , adic tocmai relaia de demonstrat.
Corolar 7.1. Dac M coincide cu I, centrul cercului nscris atunci relaia
[ABC]
2S cz by ax = + + devine
[ABC]
2S c) b r(a = + + sau
[ABC]
S p r = unde r este
raza cercului nscris n triunghi.
Corolar 7.2. Dac triunghiul este echilateral relaia
[ABC]
2S cz by ax = + +
devine
a
2S
z y x
[ABC]
= + + care se mai scrie
2
3 a
z y x = + + (am inut seama c
4
3 a
S
2
[ABC]
= ) sau:
Suma distanelor unui punct interior la laturile unui triunghi echilateral este
constant.
A
z
x
y
M
B C
157
Lema 7.1. Dac P este un punct arbitrar pe latura BC a unui triunghi oarecare
ABC atunci PB i PC sunt proporionale cu ariile triunghiurilor APB respectiv APC,
adic
[APC]
[ABP]
S
S
PC
PB
= .
Demonstraie. Avem
2
BP) d(A, BP
S
[ABP]

= , de unde
BP
2S
BP) d(A,
[ABP]
= (1)
2
PC) d(A, PC
S
[APC]

=
de unde obinem:
PC
2S
PC) d(A,
[APC]
= (2)
Dar
d(A,BP)=d(A,PC) (3)
Din (1), (2) i (3) obinem:
PC
2S
BP
2S
[APC] [ABP]
= , care se mai scrie
[APC]
[ABP]
S
S
PC
PB
=
i cu aceasta lema este demonstrat.
Teorema 7.2. (Teorema lui Ceva) Se consider un triunghi ABC i punctele
A'BC, B'CA, C'AB. Dac dreptele AA', BB', CC' sunt concurente, atunci
1
B C'
A C'
A B'
C B'
C A'
B A'
=
A
C P B
158
Demonstraie. Folosind lema obinem:
[AMC]
[AMB]
] [MCA' ] [ACA'
] [BMA' ] [ABA'
] [MCA'
] [BMA'
] [ACA'
] [ABA'
S
S
S S
S S
S
S
S
S
C A'
BA'
=

= = = ,
deci
[AMC]
[AMB]
S
S
C A'
BA'
=
[AMB]
[BMC]
MA] [B' ] [ABB'
MC] [B' C] [BB'
MA] [B'
MC] [B'
] [ABB'
C] [BB'
S
S
S S
S S
S
S
S
S
A B'
C B'
=

= = =
deci
[AMB]
[BMC]
S
S
A B'
C B'
=
[BMC]
[AMC]
] [BMC' ] [BCC'
] [AMC' ] [ACC'
] [BMC'
] [AMC'
] [BCC'
] [ACC'
S
S
S S
S S
S
S
S
S
B C'
A C'
=

= = = ,
deci
[BMC]
[AMC]
S
S
B C'
A C'
= . Atunci
1
S
S
S
S
S
S
B C'
A C'
A B'
C B'
C A'
B A'
[BMC]
[AMC]
[AMB]
[BMC]
[AMC]
[AMB]
= =
n demonstraiile unor relaii vom folosi urmtorul rezultat:
Teorema 7.3. (Teorema lui Van Aubel) Dac AA', BB', CC' sunt cevienele
unui punct M din planul triunghiului ABC avem relaia:
B C'
AC'
C B'
AB'
MA'
AM
+ =
Acum s trecem la demonstraia teoremei.
1) Punctul M este n interiorul triunghiul ABC.
A
C'
B A' C
B'
M
159
Folosim lema 7.1 demonstrat mai sus. Avem:
[BMC]
[AMB]
MC] [B' ] [BCB'
] [AMB' ] [BAB'
MC] [B'
] [AMB'
] [BCB'
] [BAB'
S
S
S S
S S
S
S
S
S
C B'
AB'
=

= = =
[BMC]
[AMC]
] [BMC' ] [CBC'
] [AMC' ] [CAC'
] [BMC'
] [AMC'
] [CBC'
] [CAC'
S
S
S S
S S
S
S
S
S
B C'
AC'
=

= = =
Atunci
[BMC]
[AMC] [AMB]
[BMC]
[AMC]
[BMC]
[AMB]
S
S S
S
S
S
S
B C'
AC'
C B'
AB'
+
= + = + (4)
Dar
[BMC]
[AMC] [AMB]
] [CMA' ] [BMA'
[ACM] [ABM]
] [CMA'
[ACM]
] [BMA'
[ABM]
S
S S
S S
S S
S
S
S
S
MA'
AM
+
=
+
+
= = = (5)
Din (4) i (5) obinem:
MA'
AM
B C'
AC'
C B'
AB'
= + .
2) Punctul M este n exteriorul triunghiului ABC.
[BMC]
[ABM]
MC] [B' CB] [B'
MA] [B' AB] [B'
MC] [B'
MA] [B'
CB] [B'
AB] [B'
S
S
S S
S S
S
S
S
S
C B'
AB'
=

= = =
A
C'
B A' C
B'
M
A
B
C
A'
M
C'
B'
160
Deci
[BMC]
[ABM]
S
S
C B'
AB'
=
[BCM]
[MAC]
BM] [C' BC] [C'
AM] [C' AC] [C'
BM] [C'
AM] [C'
BC] [C'
AC] [C'
S
S
S S
S S
S
S
S
S
B C'
AC'
=

= = =
Deci
[BMC]
[MAC]
S
S
B C'
AC'
= . Atunci
=
+
+
= = =
C] [MA' B] [MA'
[MAC] [MAB]
C] [MA'
[MAC]
B] [MA'
[MAB]
S S
S S
S
S
S
S
MA'
MA

B C'
AC'
C B'
AB'
S
S
S
S
S
S S
[BCM]
[MAC]
[BCM]
[MAB]
[BCM]
[MAC] [MAB]
+ = + =
+
= ,
adic
B C'
AC'
C B'
AB'
MA'
MA
+ = .
Corolar 7.3. Dac M esre punctul de intersecie al medianelor triunghiului
ABC, relaia lui Van Aubel devine 1 1
MA'
AM
+ = sau 2
MA'
AM
= . Deci M coincide cu G
centrul de greutate. Avem atunci 2
GA'
AG
= , de unde AG=2GA'.
Corolar 7.4. Dac M este centrul cercului nscris I, atunci
a
c
C B'
AB'
= i
a
b
B C'
AC'
= , iar relaia lui Van Aubel devine: 1
a
c b
a
b
a
c
IA'
AI
>
+
= + = deci punctul I
este mai aproape de A' dect de A (AI>IA').
Corolar 7.5. Dac M este centrul cercului nscris I avem
1
h
r
AA'
IA'
=
(r raza cercului nscris n triunghi) i analoagele
2
h
r
BB'
IB'
= i
3
h
r
CC'
IC'
= , atunci
3 2 1
h
r
h
r
h
r
CC'
IC'
BB'
IB'
AA'
IA'
+ + = + + (h
1
, h
2
, h
3
sunt nlimile triunghiului ABC).
Cu relaia lui Gergonne (cnd MI) 1
CC'
MC'
BB'
MB'
AA'
MA'
= + + obinem:
1
h
1
h
1
h
1
r
3 2 1
=

+ + , de unde
r
1
h
1
h
1
h
1
3 2 1
= + + .
161
Teorema 7.4. (Gergonne) Pe laturile [BC], [AC], [AB] ale triunghiului ABC
se consider punctele A', B', C' astfel nct dreptele AA', BB', CC' s fie concurente n
M. Atunci are loc relaia
1
CC'
MC'
BB'
MB'
AA'
MA'
= + +
Folosim n continuare lema 7.1.
[MBC]
[MAC] [MAB]
C] [MA' ] [MBA'
[MAC] [MAB]
C] [MA'
[MAC]
] [MBA'
[MAB]
S
S S
S S
S S
S
S
S
S
MA'
MA
+
=
+
+
= = =
Deci
[MBC]
[MAC] [MAB]
S
S S
MA'
MA
+
= , de unde obinem:
[MBC]
[MBC] [MAC] [MAB]
S
S S S
MA'
MA' MA
+ +
=
+
sau
[MBC]
[ABC]
S
S
MA'
AA'
=
de unde
[ABC]
[MBC]
S
S
AA'
MA'
= .
Analog obinem
[ABC]
[AMC]
S
S
BB'
MB'
= ,
[ABC]
[AMB]
S
S
CC'
MC'
= .
Adunnd membru cu membru relaiile de mai sus obinem relaia lui
Gergonne:
1
CC'
MC'
BB'
MB'
AA'
MA'
= + +
Cnd M=I relaia lui Gergonne devine:
r
1
h
1
h
1
h
1
3 2 1
= + +
Din relaia
[MBC]
[MAC] [MAB]
S
S S
MA'
MA
+
= obinem
[MAC] [MAB] [MBC]
[MAC] [MAB]
S S S
S S
MA MA'
MA
+ +
+
=
+
sau
[ABC]
[MAC] [MAB]
S
S S
AA'
MA
+
=
A
C'
B A' C
B'
M
162
Analog obinem
[ABC]
[MBC] [MAB]
S
S S
BB'
MB
+
= i
[ABC]
[MBC] [MAC]
S
S S
CC'
MC
+
= .
Atunci prin adunare obinem:
2
CC'
MC
BB'
MB
AA'
MA
= + + .
n continuare considerm o aplicaie a relaiei lui Gergonne.
Aplicaie. Se consider un patrulater convex ABCD ale crui diagonale se
intersecteaz n E. Prin E se duce o dreapt arbitrar care intersecteaz laturile AB i
CD n M respectiv N. Dreptele NA, NB. MC, MD intersecteaz diagonalele BD i AC
n punctele L, K, P, Q. S se demonstreze relaia:
1
CQ
EQ
DP
EP
AK
EK
BL
EL
= + + + .
Demonstraie. Scriem relaia lui Gergonne pentru triunghiurile MCD i NAB.
Obinem:
1
MN
EN
CQ
EQ
DP
EP
= + + i 1
NM
EM
AK
EK
BL
EL
= + +
Adunnd membru cu membru obinem
2,
MN
EM
MN
EN
AK
EK
BL
EL
CQ
EQ
DP
EP
=

+ + + + +
care se mai scrie:
1
CQ
EQ
DP
EP
AK
EK
BL
EL
= + + +
fiindc EN+EM=MN.
Cercetai dac 3
CQ
CE
DP
DE
AK
AE
BL
BE
= + + + .
Teorema 7.6. (Teorema lui Menelaus)
Fie ABC un triunghi i A', B', C' trei puncte astfel nct A'BC, B'CA,
C'AB. Dac punctele A', B', C' sunt coliniare, atunci are loc egalitatea:
A
M
B
Q
P
K
L
E
D N C
163
1
B C'
A C'
A B'
C B'
C A'
B A'
=
Demonstraie. Pentru demonstraie vom folosi urmtorul rezultat:
Lema 7.2. Raportul ariilor a dou triunghiuri despre care se cunoate c un
unghi al unuia este congruent sau suplementar cu un unghi al celuilalt triunghi, este
egal cu raportul dintre produsele laturilor care formeaz acele unghiuri.
Conform lemei avem:
C' B' A B'
B' A' CB'
S
S
,
B' A' C A'
C' A' BA'
S
S
,
A' C' B C'
B' C' AC'
S
S
] C' [AB'
] B' [CA'
] B' [CA'
] A' [BC'
] A' [BC'
] B' [AC'

=
nmulind membru cu membru cele trei relaii de mai sus obinem:
A B'
CB'
C A'
BA'
B C'
AC'
1 = i deci 1
B C'
A C'
A B'
C B'
C A'
B A'
= .

A
C'
B C
A'
B'
164
8. Triunghiuri speciale

8.1. Triunghiul ortic

Definiia 8.1.1. Triunghiul determinat de picioarele nlimilor unui triunghi
nedreptunghic se numete triunghi ortic.
Triunghiul dreptunghic nu are triunghi ortic, fiindc dou din picioarele
nlimilor coincid cu vrful unghiului drept.
Vom studia triunghiul ortic corespunztor unui triunghi ascuitunghic.
Considerm triunghiul ABC ascuitunghic cu ortocentrul n H i triunghiul
ortic corespunztor A
1
B
1
C
1
.
Fig. 1

1) Msurile unghiurilor triunghiului ortic A
1
B
1
C
1

Din patrulaterele inscriptibile A
1
BC
1
H i B
1
HA
1
C (au unghiurile din A
1
i C
1

suplementare, respectiv unghiurile din B
1
i H
1
suplementare) obinem:
m(C
1
A
1
B)=m(C
1
HB)=m(B
1
HC)=m(B
1
A
1
C)
Deci m(C
1
A
1
B)=m(B
1
A
1
C). Atunci
m(C
1
A
1
B)=180-2m(C
1
A
1
B) (1)
n patrulaterul inscriptibil AC
1
HB
1
(unghiurile din C
1
i B
1
sunt suplementare)
avem
m(C
1
AB
1
)+m(C
1
HB
1
)=180,
dar
m(BHC
1
)+m(C
1
HB
1
)=180,
deci
m(C
1
AB
1
)=m(BHC
1
)=m(C
1
A
1
B) (2)
Din (1) i (2) obinem:
m(C
1
A
1
B
1
)=180-2m(CAB)
Analog obinem:
m(A
1
C
1
B
1
)=180-2m(ACB)
m(C
1
B
1
A
1
)=180-2m(ABC)
2) Lungimile laturilor triunghiului ortic
A
C1
B1
H
B A1 C
165
Triunghiurile ABC i AB
1
C
1
sunt asemenea avnd un unghi comun A i
ABCAB
1
C
1
(patrulaterul C
1
B
1
CB este inscriptibil).
Din proporionalitatea laturilor obinem:
1 1 1
AB
AB
C B
BC
= (3)
Din triunghiul dreptunghic ABB
1
avem
A cos
AB
AB
1
= (4)
Din relaiile (3) i (4) obinem:
A cos
1
C B
BC
1 1
= , de unde B
1
C
1
=BCcosA, care
se mai scrie B
1
C
1
=acosA.
Analog obinem: A
1
B
1
=ccosC i A
1
C
1
=bcosB.
Mai folosim notaia A cos
0
a a = , B cos
0
b b = , C cos
0
c c = .
3) Perimetrul triunghiul ortic
Notm cu P
0
perimetrul triunghiul ortic. Atunci
P
0
=A
1
B
1
+B
1
C
1
+C
1
A
1
=ccosC+acosA+bcosB
Deci P
0
=acosA+bcosB+ccosC.
Se pot da pentru P
0
multe exprimri n funcie de necesiti. De exemplu, din
teorema sinusurilor R
c b a
2
C sin B sin A sin
= = = , obinem:
a=2RsinA, b=2RsinB, c=2RsinC.
Atunci P
0
=R(2sinAcosA+2sinBcosB+2sinCcosC) sau
P
0
=R(sin2A+sin2B+sin2C)
(am inut seama c 2sinxcosx=sin2x).
Dar sin2A+sin2B+sin2C=4sinAsinBsinC i atunci
2
0
2 2 2 2
4 C sin B sin A sin 4
R
abc
R
c
R
b
R
a
R R P = = = (4*)
Dar S
R
abc
=
4
, de unde abc=4RS.
Atunci putem scrie
R
S
R
RS
P
2
2
4
2
0
= = . Deci
R
S
P
2
0
= (5)
Corolar 8.1. maximul perimetrului triunghiului ortic este semiperimetrul
triunghiului de referin.
Din inegalitatea lui Euler r R 2 obinem:
r R
1 2
, de unde
r
S
R
S

2
sau
r
S
P
0
, care se mai scrie p P
0
(am inut seama c r p S = ).
166
Corolar 8.2. Dintre toate triunghiurile nscrise ntr-un triunghi ascuitunghic,
triunghiul ortic are perimetrul minim.
Dac sunt date punctele B
1
, C
1
pe laturile AC, AB, punctul A
1
de pe latura BC
pentru care suma B
1
A
1
+A
1
C
1
este minim este determinat de proprietatea c dreptele
B
1
A
1
, A
1
C
1
formeaz acelai unghi cu BC.
4) Semiperimetrul triunghiul ortic (p
0
)

R
S P
p = =
2
0
0
(am folosit relaia (5))
Deci
R
S
p =
0
.
Din relaia de mai sus rezult R p S =
0
(relaia lui ieica).
Avem relaia:
r
R
r p
p R
S
p R
R
S
p
p
p
P
P
=

= = =
2
2
2
2
0 0

Deci:
Raporul dintre perimetrul unui triunghi i perimetrul triunghiului su ortic este
r
R
.
i) Segmentele ce unesc proieciile picioarelor nlimilor pe laturile adiacente
sunt egale cu semiperimetrul triunghiului ortic.
Fie B
2
i C
2
proieciile piciorului A
1
al nlimii AA
1
pe laturile AC i AB. n
cercul de diametru AA
1
, B
2
C
2
este o coard opus unghiului A, din teorema sinusurilor
avem:
1
2 2
AA
A sin
=
C B

= R
a
2
A sin
de unde obinem:
B
2
C
2
=AA
1
sinA
n triunghiul ABA
1
,
AB
AA
B sin
1
= , de unde AA
1
=ABsinB.
Cu teorema sinusurilor n triunghiul ABC avem R 2
sinC
AB
= . Atunci
B
2
C
2
=AA
1
sinA=ABsinBsinA=2RsinAsinBsinC (6)
Din (4*) i (6) obinem
0
0
2 2
2
p
P
C B = = .
ii) n triunghiul ascuitunghic ABC cercurile construite pe AB i AC ca
diametru determin pe B
1
C
1
un segment de lungime egal cu perimetrul triunghiului
ortic.
167
Fig. 2.

5) Aria triunghiului ortic (S
0
)
2
) A 2 180 sin( B cos C cos
2
sinA C A B A
1 1 1 1 1
0

=

=
b c
S
) A 2 sin ) A 2 180 (sin( A 2 sin B cos C cos
2
=

=
c b

) A cos A sin 2 A 2 (sin A cos A sin 2 B cos C cos
2
=

=
c b

AcosBcosC cos 2 AcosBcosC cos A sin
2
2 S
bc
=

=
Deci S
0
=2ScosAcosBcosC.
Atunci AcosBcosC cos
2
0
=
S
S
.
Fiindc
8
1
AcosBcosC cos , atunci
8
1
2
0

S
S
, de unde
4
0
S
S .
Deci maximul ariei triunghiului ortic este
4
S
i se obine cnd triunghiul ABC
este echilateral.
6) Raza cercului nscris n triunghiul ortic (r
0
)
AcosBcosC cos 2
AcosBcosC cos 2
0
0
0
R
R
S
S
p
S
r = = =
Deci r
0
=2RcosAcosBcosC.
7) Raza cercului circumscris triunghiului ortic ) (
0
R este jumtate din raza
cercului circumscris triunghiului considerat (R
0
).
2 2
1
4 AcosBcosC cos 2 4
AcosBcosC cos
4
0
0 0 0
0
R
S
abc
S
abc
S
c b a
R = =

= =
(am folosit relaia abc=4RS).
A
T
T1
C1
C2
B1
B2
A1
B
C
O
168
8) Laturile triunghiului ortic sunt antiparalele cu laturile triunghiului de
referin, pe care sunt egal nclinate.
Patrulaterul BCB
1
C
1
este inscriptibil i atunci AC
1
B
1
ACB, de unde
rezult c C
1
B
1
||BC.
9) Laturile triunghiului ortic sunt paralele cu tangentele n A, B, C la cercul
circumscris triunghiului ABC.
De exemplu B
1
C
1
este paralel cu AT
1
, deoarece m(T
1
AC) este jumtate din
msura arcului AC, iar ) ( ) (
1 1
ABC m C AB m = (jumtate din msura arcului AC).
10) nlimile triunghiului ascuitunghic ABC sunt bisectoarele triunghiului
ortic A
1
B
1
C
1
.
i) Triunghiul ABC este ascuitunghic. n figura 1 patrulaterul BCB
1
C
1
fiind
inscriptibil rezult c
B
1
BCCC
1
B
1
(7)
Din patrulaterul inscriptibil AC
1
A
1
C rezult c
A
1
C
1
CA
1
AC (8)
Unghiurile A
1
BB
1
i A
1
AC sunt congruente avnd acelai complement
(90-m(C)) (8*).
Din patrulaterul inscriptibil BA
1
HC
1
avem c
A
1
BHA
1
C
1
H (9)
Din relaiile (7), (8), (9), (8*) obinem c [CC
1
este bisectoarea A
1
C
1
B
1
.
Analog se demonstreaz c i [A
1
A este bisectoarea C
1
A
1
B
1
, iar [B
1
B este
bisectoarea C
1
B
1
A
1
.
ii) n triunghiul obtuzunghic nlimea dus din vrful unghiului obtuz este
bisectoarea unghiului interior al triunghiului ortic, al crui vrf este piciorul nlimii
respective, iar nlimile duse din vrfurile celor dou unghiuri ascuite sunt bisectoare
pentru unghiurile exterioare corespunztoare, ale triunghiului ortic.
Triunghiul ABC fiind obtuzunghic s artm c [A
1
A este bisectoarea
interioar a unghiului C
1
A
1
B
1
, iar [B
1
B este bisectoarea exterioar a unghiului
C
1
B
1
A
1
.
Din patrulaterul inscriptibil B
1
AA
1
B (unghiurile opuse din A
1
i B
1
sunt
suplementare) obinem:
AA
1
B
1
ABB
1
(10)
A
H
D
1
D
B1 C
1
C A
1 B
169
Din patrulaterul inscriptibil C
1
CA
1
A (unghiurile opuse din C
1
i A
1
sunt
suplementare) obinem:
AA
1
C
1
ACC
1
(11)
Dar
C
1
BHB
1
CH (12)
avnd acelai complement (90-m(H)). Atunci obinem din relaiile (10), (11), (12)
c C
1
A
1
AB
1
A
1
A, adic [A
1
A este bisectoarea unghiului C
1
A
1
B
1
.
Din patrulaterul inscriptibil A
1
AB
1
B obinem
BB
1
A
1
A
1
AB (13)
Dar
A
1
ABC
1
AH (opuse la vrf) (14)
Din patrulaterul inscriptibil HC
1
AB
1
obinem
C
1
AHC
1
B
1
HDB
1
B (15)
Din relaiile (13), (14), (15) rezult c A
1
B
1
BBB
1
D, deci [B
1
B este
bisectoarea A
1
B
1
D.


8.2. Triunghiul tangenial

Definiie 8.2.1. Triunghiul tangenial este triunghiul format de cele trei
tangente duse prin vrfurile unui triunghi nedreptunghic la cercul su circumscris.
n cazul triunghiului dreptunghic dou tangente devin paralele i deci nu exist
triunghi tangenial.
1) Msurile unghiurilor
i) Triunghiul ABC este ascuitunghic.

Fig. 1.

Triunghiul A'B'C' este triunghiul tangenial.
B
C'
A
B'
C
H
A''
A'
C''
B''
170
Tangentele din A' la cerc sunt congruente i deci triunghiul BA'C este isoscel.
Avem c msura BAC este jumtate din msura arcului BC, iar msura
unghiului CBA' este jumtate din msura arcului BC, deci unghiurile BAC i CBA'
sunt congruente.
n triunghiul A'BC avem
m(BA'C)+m(A'BC)+m(A'CB)=180,
care se mai scrie m(BA'C)+2m(BAC)=180 de unde
m(BA'C)=180-2m(BAC)
sau m(A')=180-2m(A).
Analog obinem m(B')=180-2m(B), m(C')=180-2m(C).
ii) Triunghiul ABC este obtuzunghic cu unghiul obtuz n A. Tangentele dintr-
un punct exterior la cerc fiind congruente, rezult c triunghiurile B'AC i C'AB sunt
isoscele i unghiurile A'B'C' respectiv A'C'B' sunt exterioare acestor dou
triunghiuri. Atunci BA) C' 2m( ) B' C' A' m( = (msura arcului AB) =2m(BAC).
Deci m(C')=2m(C).

Fig. 2.

Analog obinem m(B')=2m(B).
Atunci
m(A')=180-(m(B')+m(C'))=180-2(m(B)+m(C))=
=180-2(180-m(A))=2m(A)-180
Deci m(A')=2m(A)-180.

O
B
C
A
C'
B'
A'
171
Fig. 3

2) Lungimile laturilor triunghiului tangenial
Fie ABC triunghi ascuitunghic cu A'B'C' triunghiul tangenial (Fig.1).
Triunghiul A'BC este isoscel cu (A'B)(A'C). Cu teorema sinusurilor n
triunghiul AB'C obinem:
sinB
AB'
sinB'
AC
= care se mai scrie:
sinB
AB'
) B 2 180 sin(
=

b
, de unde
) B 2 180 sin(
B sin
' AB

=
b
.
Dar sin(180-2B)=sin2B=2sinBcosB.
Atunci
B cos 2 BcosB sin 2
B sin
' AB
b b
= = , deci
B cos 2
' AB
b
= i
B cos 2
C B'
b
= .
Cu teorema sinusurilor n triunghiul isoscel C'AB obinem:
sinC
AC'
sinC'
AB
= ,
de unde
C cos 2 C cos C sin 2
C sin
C 2 sin
C sin
) C 2 180 sin(
C sin
' C sin
C sin
' AC
c c c c c
= = =

= =
Deci
C cos 2
' AC
c
= (1)
Atunci
C cos 2 B cos 2
AC' A B' C' B'
c b
+ = + = .
Din teorema cosinusului avem:
B
C
A'
B'
C'
A
O
172
ac
b c a
2
B cos
2 2 2
+
= i
ab
c b a
2
C cos
2 2 2
+
=
Atunci
=

+
+

+
= + =
ab
c b a
c
ac
b c a
b c b
2
2
2
2
C cos 2 B cos 2
C' B'
2 2 2 2 2 2

=

+
+
+
=
+
+
+
=
2 2 2 2 2 2 2 2 2 2 2 2
1 1
c b a b c a
abc
c b a
abc
b c a
abc

=
+ +

=

+ +
+ + +
=
) )( (
2
) )( (
2 2 2 2 2 2
2
2 2 2 2 2 2
2 2 2 2 2 2
c b a b c a
a abc
c b a b c a
b c a c b a
abc
BcosC cos 2
4
2 2
2
2
2 2 2 2 2 2
3
a
bc a
ab
c b a
ac
b c a
bc a
=

+
= .
Deci
BcosC cos 2
C' B'
a
= .
Analog obinem
AcosC cos 2
C' A'
b
= ,
AcosB cos 2
B' A'
c
= .
Dm i alte exprimri pentru lungimile laturilor triunghiului tangenial.
Folosim teorema sinusului.
Din R
b
2
B sin
= obinem: B sin 2R b = .
Atunci tgB
B cos 2
B sin 2
B cos 2
C B' R
R b
= = = .
Analog obinem tgC
C cos 2
C sin 2
C cos 2
AC' R
R c
= = = .
Atunci B'C'=B'A+C'A=RtgB+RtgC adic B'C'=R(tgB+tgC).
Analog obinem: A'C'=R(tgA+tgC) i A'B'=R(tgA+tgB).
3) Perimetrul triunghiului tangenial (P')
P'=A'B'+B'C'+A'C'=2R(tgA+tgB+tgC)
Dar tgA+tgB+tgC=tgAtgBtgC, atunci P'=2RtgAtgBtgC, relaie care se mai
scrie
C cos B cos A cos
C sin B sin A sin
R 2 P' = i cu teorema sinusurilor ( R
a
2
A sin
= , R
b
2
B sin
= ,
R
c
2
C sin
= ) obinem:
AcosBcosC cos 4 AcosBcosC cos
1
2 2 2
2 '
2
R
abc
R
c
R
b
R
a
R P = = .
173
Dar abc=4RS i atunci
sAcosBcosC sC cosAcosBco 4
4
'
2
Rco
S
R
RS
P = = .
Deci
sC cosAcosBco
'
R
S
P = , atunci semiperimetrul
sC cosAcosBco 2
'
R
S
p = sau
p'=RtgAtgBtgC.
4) Raza cercului nscris n triunghiul tangenial (r')
Laturile triunghiului tangenial sunt tangente n A, B, C la cercul circumscris
triunghiului ABC, deci raza cercului nscris n triunghiul tangenial este
r'=R.
5) Aria triunghiului tangenial ' S
Fiindc ' ' ' r p S = obinem tgC tgB tgA '
2
= R S .
6) Raza cercului circumscris triunghiului tangenial (R')
Cu teorema sinusurilor n triunghiul tangenial obinem:
' 2
' A sin
'
R
a
= sau ' 2
) A 2 180 sin(
C' B'
R =

,
de unde
= =

=
n2A cosBcosCsi 4 ) A 2 180 sin( 2
C' B'
'
a
R
sC cosAcosBco 4 nAcosA cosBcosCsi 8
A sin 2 R R
= =
(Am inut seama c R
a
2
A sin
= i sin2A=2sinAcosA).
Deci
sC cosAcosBco 4
'
R
R = .
7) nlimile triunghiului tangenial (
'
'
'
'
'
'
, ,
c b a
h h h )
Din
2
'
'
'
' a
h a
S

= obinem
= =

= =
a
R
a
R
a
S
h
a
cosBcosC
C cos
C sin
B cos
B sin
A cos
A sin
4
cosBcosC 2
tgC tgB tgA 2
'
' 2
2
2
'
'

A sin 2
sinBsinC
A cos
A sin 4 sinBsinC
A cos
A sin
4
2
2
R
R
a
R = =
Deci
A cos
sinBsinC 2
'
'
R
h
a
= .
Analog obinem
B cos
sinAsinC 2
'
'
R
h
b
= i
C cos
sinAsinB 2
'
'
R
h
c
= .
174
8) Distanele de la centrul cercului circumscris triunghiului ABC la vrfurile
triunghiului tangenial A'B'C'
n patrulaterul inscriptibil AOBC', cu prima teorem a lui Ptolemeu obinem:
C'OAB=AC'BO+AOC'B (O centrul cercului circumscris triunghiului ABC).
Dar AB=c, BO=AO=R i AC'=C'B, deci C'Oc=2AC'R, din (1) avem
C cos 2
AC'
c
= i atunci
C cos 2
2
O C'
R c
c

= , de unde
C cos
O C'
R
= .
Analog obinem:
A cos
O A'
R
= ,
B cos
O B'
R
= .
9) Distanele ntre vrfurile triunghiului considerat i vrfurile corespunztoare
ale triunghiului tangenial: AA', BB', CC'.
Aplicm teorema cosinusului n triunghiul ABA'
AA'
2
=AB
2
+A'B
2
-2ABA'Bcos(ABA')
n triunghiul A'BC cu teorema sinusului obinem:
sinA'
BC
) BCA' sin(
B A'
=

sau
2A) sin(180
BC
sinA
B A'

=
care se mai scrie:
sin2A
BC
sinA
B A'
= sau
2sincosA
BC
sinA
B A'
= ,
de unde
A cos 2
B A'
a
= . Atunci
) ' ABA cos(
A cos 2
2
A cos 2
' AA
2
2 2

+ =
ca a
c
Obinem
A cos
' AA
2
2
2 a
m
= , de unde
2 2 2
A cos '
a
m AA =
Atunci avem i
2 2 2
B cos '
b
m BB = ,
2 2 2
C cos '
c
m CC = .
Adunnd relaiile de mai sus i innd seama c
) (
4
3
2 2 2 2 2 2
c b a m m m
c b a
+ + = + +
obinem
) (
4
3
C cos ' CC B cos ' BB A cos ' AA
2 2 2 2 2 2 2 2 2
c b a + + = + +
10) Aria unui triunghi nedreptunghic este medie proporional ntre aria
triunghiului su tangenial i aria triunghiului su ortic, adic ' ' '
2
S S S = , unde ' S i
' ' S sunt ariile triunghiurilor tangenial i respectiv ortic.
tgC tgB tgA '
2
= R S iar sC cosAcosBco 2 ' ' S S = ,
relaie demonstrat la triunghiul ortic. Atunci
175
sC cosAcosBco 2
cosC
sinC
cosB
sinB
cosA
sinA
' ' '
2
S R S S =
de unde obinem:
nC sinAsinBsi 2 ' ' '
2
SR S S =
Din teorema sinusurilor
R
c b a
2
C sin B sin A sin
= = =
avem:
R
c
R
b
R
a
2
C sin ,
2
B sin ,
2
A sin = = =
Atunci
R R R
c b a
SR S S
2 2 2
2 ' ' '
2


= , de unde
R
abc S
S S
4
' ' '

= , iar cu
SR abc 4 = obinem
2
4
4
' ' ' S
R
SR S
S S =

= , adic
2
' ' ' S S S = .
11) Produsul distanelor unui punct de pe cercul circumscris unui triunghi la
laturile triunghiului su tangenial este egal cu produsul distanelor aceluiai punct la
laturile triunghiului considerat.
Pentru demonstraie folosim o propoziie ajuttoare.
Lema 8.2. Distana unui punct oarecare de pe un cerc la o coard este medie
proporional ntre distanele aceluiai punct la tangentele duse prin extremitile
coardei.
Demonstraie. Considerm un punct P pe cerc i PD, PE distanele lui P la
tangentele duse prin extremitile coardei AB.

Fig. 4.

Triunghiurile dreptunghice PBE i PAM sunt asemenea fiindc unghiurile
PAM i PBE au ca msur jumtate din msura arcului PB.
Din proporionalitatea laturilor obinem:
D
P
E
A
B
M
176
PA
PB
PM
PE
=
i triunghiurile dreptunghice PBM i PAD sunt asemenea avnd unghiurile
PBA i PAD congruente.
Scriem proporionalitatea laturilor
PD
PM
PA
PB
=
Atunci obinem:
PD
PM
PM
PE
= de unde PM
2
=PDPE. i lema este demonstrat.
n figura 5 se consider punctul P pe cercul circumscris triunghiului ABC i
PR, PM, PN distanele lui P la laturile triunghiului ABC, iar PD, PE, PF distanele lui P
la laturile triunghiului tangenial. Cu lema considerat obinem:
PM
2
=PFPD, PR
2
=PFPE, PN
2
=PEPD
nmulind membru cu membru relaiile de mai sus obinem:
PMPRPN=PFPDPE.

Fig. 5





Tiparul executat prin comanda nr. 549, 2003, la
CARTPRINT Baia Mare, cod 4800
cartier Ssar, str. Victoriei nr. 146
tel./fax: 0262/218923
Romnia
P
R
C' B D A'
C
N
E
M
A
F
B'

Vous aimerez peut-être aussi